You are on page 1of 76

Copyright © 2014 Delhi Academy of Medical Sciences, All Rights Reserved.

1/146
Copyright © 2014 Delhi Academy of Medical Sciences, All Rights Reserved. 2/146
Test Information
Test Name AIIMS MOCK 2015 MD/MS Total Questions 200
Test Type Examination Difficulty Level Difficult
Total Marks 600 Duration 180minutes
Test Question Language:- ENGLISH
(1). Dilator pupillae is supplied by –
a. Post-ganglionic parasympathetic fibres from Edinger-Westphal nucleus
b. Post-ganglionic sympathetic fibres from the cervical sympathetic chain
c. III nerve
d. Sympathetic fibres from the fronto-orbital branch of V nerve
Solution. Ans-1: (b) Post-ganglionic sympathetic fibres from the cervical sympathetic chain
Ref: Gray’s anatomy, 39/e, pg 710; Parson’s ‘Diseases of the Eye’; 20/e, pg 33
Sol :
• Dilator pupilae is supplied with nonmyelinated post ganglionic fibers from the superior sympathetic ganglion. (Gray’s
anatomy)
• Dilator pupillae is supplied by adrenergic fibers of cervical sympathetic nerves.The dilator tract probably commences in
hypothalamus not far from constrictor centre and also has connection with cerebral cortex. (Parson’s)
• Sphincter ppillae is supplied by myelinated post ganglionic fibers from parasympathetic system.
• The sphincter papillae is supplied by cholinergic nerves of parasympathetic system through the third canal nerve.The fibers
start
in E W nucleus near the third nerve nucleus in floor of aqueduct of sylvius.
• Sympathetic supply of eye:
• First order neuron : from hypothalamus till ciliospinal centre of budge in intermediolateral columns of c8, t1 and t2
• Second order neuron : from ciliospinal centre of budge till superior cervical ganglion in neck
• Third order neuron: post ganglionic fibres arise from cervical ganglion, ascends along internal carotid artery to enter skull,
where
post ganglionoic fibres join Nasocilliary branch of trigeminal nerve. Ultimately, postganglionic sympathetic fibres reach ciliary
body and
dilator papillae long ciliary nerve.
• Light reflex pathway(pupillary pathway)-
• Retina →optic nerve(Afferent) → chiasma →optic tract →Pretectal Nucleus→EW nucleus(in Mid Brain)→3 rd
Nucleus(occulomotor
Nu. i.e. efferent→lower div.→nerve to inferior oblique →cilliary ganglion →short ciliary nerve→ Sphincter Pupillae.
• Short cilliary nerve are only post ganglionic nerve which is myelinated
• No LGB & No cortex in this pathway.
• As cortex is not invoved so not affected by cortical lesion.
• Accomodation reflex
• Retina →optic nerve(Afferent) → chiasma →optic tract→ LGB →Optic radiation(occipital cortex)→occipitomesencephalic
tract→Pons→3rd nerve Nu.→Accessory ganglion (axenfield ganglion)
Your Answer. b
Correct Answer. b
Copyright © 2014 Delhi Academy of Medical Sciences, All Rights Reserved. 3/146
(2). All of the following cross the right ureter anteriorly except –
a. Terminal ileum
b. Genitofemoral nerve
c. Right colic artery and ileocolic artery
d. Vas deferens
Solution. Ans-2: (b) Genitofemoral nerve
Ref: Snell’s ‘Clinical anatomy’, 7/e, pg 226,241,284
Sol :
• Ureter is a 25 cm long structure on each side.
• The upper half of its length lies in posterior abdominal wall and the lower half lies in the true pelvis.
• The relations of abdominal part of ureter are different on each side while its similar on both the sides for the pelvic part.
Relations of Abdominal part-
1. Right ureter-
• The abdominal part of right ureter is overlapped at its upper end by the descending part of duodenum.
• Lower down it is crossed by terminal ileum & by root of mesentry.
• Genitofemoral nerve crosses behind the ureter.
• Crossing in front of ureter, there are the testicular & ovarian vessels, right colic & ileocolic branches of superior mesenteric
artery
& terminal part of superior mesenteric artery itself (in the root of mesentry). The arteries are accompanied by corresponding
veins.
• The inferior vena cava lies a short distance medial to right ureter.
2. Left ureter-
• Abdominal part of left ureter is crossed (near the brim of the pelvis) by sigmoid colon. The ureter passes deep to the apex
of the Vshaped
attachment of the sigmoid mesocolon.
• As on the right side, the ureter is crossed, posteriorly by the genitofemoral nerve and in front by testicular or ovarian
vessels.
• It is also crossed by left colic branches of inferior mesenteric artery.
• The inferior mesenteric vein is placed parallel to left ureter, a little to its medial side.
Relations of the pelvic part of ureter-
• These are similar on right & left side.
• As the ureter runs backwards & laterally on the lateral wall of pelvis, it lies on the fascia covering the obturator internus.
• Here the ureter crosses several structures that lie between it & lateral pelvic wall.
• In the male, these are, superior vesical artery,obturator nerve,artery & vein,inferior vesical artery.
Your Answer. b
Correct Answer. b
(3). In case of perforation of ulcer in the posterior wall of 1 st part of duodenum, which of the following structure is likely to be
involved?
a. Inferior vena cava
b. Gastroduodenal artery
c. Bile duct
d. Portal vein
Solution. Ans-3: (b) Gastroduodenal artery
Ref: Bailey & Love’s ‘Short Practice of Surgery’, 25/e, Michels, N. A.: Blood Supply and Anatomy of the Upper Abdominal
Organs with a
Descriptive Atlas.
Sol :
• A highly lethal form of exigent hemorrhage due to peptic ulcer is that which occurs consequent to perforation of the
gastroduodenal
artery, its bifurcation, or either major branch, in the base of a large juxtapyloric ulcer.
• Perforations involving the gastroduodenal artery complex occur as a sequel to transmural ulceration of the posterior
duodenal wall.
• Such erosion excavates the pancreas on each side of the more resistant artery, leaving a portion of the circumference of a
segment
of the vessel protruding from the relatively non-fibrotic pancreatic base of the usually large ulcer.
Your Answer. b
Correct Answer. b
Copyright © 2014 Delhi Academy of Medical Sciences, All Rights Reserved. 4/146
(4). About sternocleidomastoid tumor all are true except –
a. Always associated with breech
b. Spontaenous resolution in most cases
c. Two-third have palpable neck mass at birth
d. Uncorrected cases develops plagiocephaly
Solution. Ans-4: (d) Uncorrected cases develops plagiocephaly
Ref:Read the text below
Sol:
• Torticollis presents with a hard, nontender, fibrotic mass within the sternocleidomastoid muscle.
• It may be present at birth but is usually not noticed until the second to sixth weeks of life.
• The mass appears with equal frequency in both sexes and on each side of the neck.
• Rarely, there is more than one mass in the muscle or both sternocleidomastoid muscles are involved.
• A history of breech delivery is present in 20–30% of these children.
Clinical Findings
• Torticollis is manifested when the sternocleidomastoid muscle is shortened and the mastoid process on the involved side is
pulled
down toward the clavicle and manubrium.
• As a result, the head is abducted to the ipsilateral side and rotated to the contralateral side (toward the opposite shoulder).
• The shoulder on the affected side is raised, and there may be cervical and thoracic scoliosis.
• Passive rotation of the head to the side of the involved muscle will be resisted and limited to varying degrees, and the
muscle will
appear as a protuberant band
• Because of persistent pressure when the patient is recumbent, the ipsilateral face and contralateral occiput will be
flattened.
• Facial hemihypoplasia and plagiocephaly (flattening of the ipsilateral posterior skull) occurs in untreated cases, usually
within 6
months.
Treatment
• Surgery is rarely necessary for this disorder.
• Torticollis is treated with active range of motion exercises.
• The child's shoulders are held flat to a table and the head is tilted and rotated in a full range of motion.
• This procedure should be performed at least four times a day, usually for 2–3 months.
• The firm "tumor" often disappears well before the torticollis is cured. If the muscle continues to become progressively
shortened,
with facial and occipital skull deformity, both heads of the sternocleidomastoid muscle should be divided through a small
transverse
incision just above the clavicle.
This procedure does not reverse the bony changes that have already developed but prevents progression of the process.
Your Answer. b
Correct Answer. d
(5). About Paneth cells – True is
a. Lack rough endoplasmic reticulum
b. High zinc content
c. Foamy cytoplasm
d. None
Solution. Ans-5: (b) High zinc content
Ref: Gray’s 40 th ed 1139
Sol:
Paneth cells
• Paneth cells are numerous in the deeper parts of the intestinal crypts, particularly in the duodenum.
• They are rich in zinc and contain large acidophilic granules that stain strongly with eosin or phosphotungstic haematoxylin.
• Paneth cells secrete lysozyme, a highly specific antibacterial enzyme, and other defensive proteins (defensins) such as
tumour
necrosis factor alpha (TNF-α), which protect the intestinal luminal surface.
Your Answer. b
Correct Answer. b
Copyright © 2014 Delhi Academy of Medical Sciences, All Rights Reserved. 5/146
(6). The function of 8th cranial nerve is related to
a. Smell
b. Taste
c. Touch
d. Balance
Solution. Ans-6: (d) Balance
Ref:Read the text below
Sol:
• The vestibulocochlear nerve (also known as the auditory or acoustic nerve) is the eighth of twelve cranial nerves, and is
responsible
for transmitting sound and equilibrium (balance) information from the inner earto the brain
Your Answer. d
Correct Answer. d
(7). Following structures are present above the perineal membrane in females except :
a. Sphincter urethrae
b. Compressor urethrae
c. Sphincter urethrovaginalis
d. Bartholin's gland
Solution. Ans-7 : (d) Bartholin's gland
Ref:Read the text below
Sol:
• The glands in females which are homologous to bulbourethral glands in males, are the greater vestibular glands or
Bartholin's
glands.
• They are round or oval, reddish-yellow bodies flanking the vaginal orifice.
• They are situated inferior to the perineal membrane in females.
Your Answer. d
Correct Answer. d
Copyright © 2014 Delhi Academy of Medical Sciences, All Rights Reserved. 6/146
(8). All of the following are derivatives of the pharyngeal arches except –
a. Orbicularis oculi
b. Levator palpebrae superioris
c. Palatine tonsil
d. Tensor tympani
Solution. Ans-8: (b) Levator palpebrae superioris
Ref: Read the text below
Sol :
Gross Anatomy (Board Review)by Hagerstown, MD: Lippincott Williams & Wilkins
• There are six pharyngeal arches, but in humans the fifth arch only exists transiently during embryologic growth and
development.
• Since no human structures result from the fifth arch, the arches in humans are I, II, III, IV, and VI.
• More is known about the fate of the first arch than the remaining four.
• The first three contribute to structures above the larynx, while the last two contribute to the larynx and trachea.
Pharyngeal
arch Muscular contributions Skeletal contributions Nerve Artery
1st (also
called
"mandibular
arch")
Muscles of mastication, anterior belly of
the digastric, mylohyoid, tensor tympani, tensor veli
palatini
Maxilla, mandible (only
as a model for mandible
not actual formation of
mandible),
the incus and malleus of
the middle ear,
also Meckel's cartilage
Trigeminal
nerve(V2 and V3)
Maxillary
artery,external
carotid artery
2nd (also
called the
"hyoid
arch")
Muscles of facial expression,
buccinator, platysma, stapedius,stylohyoid, posterior
belly of the digastric
Stapes, styloid
process, hyoid (lesser
horn and upper part of
body), Reichert's
cartilage
Facial nerve (VII)
Stapedial
artery,hyoid
artery
3rd Stylopharyngeus
Hyoid (greater horn and
lower part of
body), thymus, inferior
parathyroids
Glossopharyngeal
nerve (IX)
Common
carotid,internal
carotid
4th Cricothyroid muscle, all intrinsic muscles of soft
palate including levator veli palatini
Thyroid cartilage,
superior
parathyroids, epiglottic
cartilage
Vagus
nerve (X),superior
laryngeal nerve
Right 4th aortic
arch: subclavian
artery
Left 4th aortic
arch:aortic arch
6th All intrinsic muscles of larynx except the cricothyroid
muscle
Cricoid
cartilage, arytenoid
cartilages, corniculate
cartilage
Vagus
nerve (X),recurrent
laryngeal nerve
Right 6th aortic
arch: pulmonary
artery
Left 6th aortic
arch:pulmonary
arteryand ductus
arteriosus
Orbicularis oculi is muscle of facial expression which is derivative of 2 nd pharyngeal arch.
Structures developing from Pharyngeal pouches-
First pouch
The endoderm lines the future auditory tube (Pharyngotympanic " Eustachian " tube) , middle ear, mastoid antrum, and inner
layer of the
tympanic membrane & origin of Mandibular nerve & Maxillary artery.
Second pouch
Contributes to the middle ear, palatine tonsils, supplied by the facial nerve.
Third pouch
The third pouch possesses Dorsal and Ventral wings. Derivatives of the dorsal wings include the inferior parathyroid glands,
while the
ventral wings fuse to form the cytoreticular cells of the thymus. The main nerve supply to the derivatives of this pouch is
Cranial Nerve
IX, glossopharyngeal nerve.
Fourth pouch
Superior parathyroid glands and ultimobranchial body which forms the parafollicular C-Cells of the thyroid gland.
Musculature and cartilage of larynx (along with the sixth pharyngeal pouch).
Fifth pouch
Rudimentary structure, becomes part of the fourth pouch contributing to thyroid C-cells
Sixth pouch
Along with the fourth pouch, contributes to the formation of the musculature and cartilage of the larynx
Your Answer. b
Correct Answer. b
Copyright © 2014 Delhi Academy of Medical Sciences, All Rights Reserved. 7/146
(9). Which of the following about the valves of Houston is true?
a. They disappear after mobilization of the rectum
b. The middle valve folds towards the right side
c. The upper valve corresponds to the anterior peritoneal reflection
d. The valves contain all layers of the muscle wall
Solution. Ans-9: (a) They disappear after mobilization of the rectum
Ref: Sabiston’s ‘Text book of Surgery’ 18/e
Sol :
The rectum possesses three involutions or curves known as the valves of Houston.
The middle valve folds to the left (option ‘2’), and the proximal and distal valves fold to the right.
These valves are more properly called folds because they have no specific function as impediments to flow.
They are lost after full surgical mobilization of the rectum, a maneuver that may provide about 5 cm of additional length to
the rectum,
greatly facilitating the surgeon's ability to fashion an anastomosis deep in the pelvis.
These folds are about 12 mm. in width and are composed of the circular muscle coat of the rectum (option ‘4’). They are
usually three in
number; sometimes a fourth is found, and occasionally only two are present.
• One is situated near the commencement of the rectum, on the right side.
• The second extends inward from the left side of the tube, opposite the middle of the sacrum.
• The third, the largest and most constant, projects backward from the forepart of the rectum, opposite the fundus of the
urinary
bladder.
• When a fourth is present, it is situated nearly 2.5 cm above the anus on the left and posterior wall of the tube.
Your Answer. c
Correct Answer. a
(10). A patient presents with a penetrating chest wound associated with intrathoracic haemorrhage, A thoracotomy is done by taking
an
incision in the 4th/5th intercostals space starting 1 cm away from the lateral margin of the sternum. This is done to avoid injury to the –
a. Pleura
b. Intercostal artery
c. Internal thoracic artery
d. Intercostal nerve
Solution. Ans-10: (c) Internal thoracic artery
Ref: Guillermo Parra Sanchez, Edward W.K. Peng, Richard Marks, Pradip K. Sarkar. 'Sccop and Run' strategy for a resuscitative
sternotomy following unstable penetrating chest injury. Interactive Cardiovascular and Thoracic Surgery.
Sol :
Thoracotomy in Penetrating Chest Wound-
• The anatomy of the thoracic cage is well-known and encompasses the area beneath the clavicles and superior to the diaphragm,
bound laterally by the rib cage, anteriorly by the sternum and ribs, and posteriorly by the rib and vertebral bodies.
• Entry into the thorax may be made by sternotomy; thoracotomy (incising between selected ribs, most commonly the fourth and
fifth)
on either the right or left side; or a clamshell incision, consisting of left and right thoracotomy incisions traversing the sternum to join
the
two.
• Particular care must be exercised laterally near the sternum, where the internal thoracic (mammary) artery lies 2-4 cm on either
side.
• Similarly, remember that immediately inferior to each rib body are the intercostal artery, vein, and nerve, from which voluminous
bleeding can occur.
• Patients have required reexploration for injuries to these various vessels and have exsanguinated as a result of missed injuries to
these vessels.
Your Answer. c
Correct Answer. c
Copyright © 2014 Delhi Academy of Medical Sciences, All Rights Reserved. 8/146
(11). The current flow across a single ion channel can be measured by –
a. Voltage clamp test
b. Patch clamp test
c. Multimeter
d. Iontophoresis
Solution. Ans-11: (b) Patch clamp test
Ref: Ganong’s ‘Review of Medical Physiology’;
24/e,http://en.wikipedia.org/wiki/Patch_clamp
Sol :
MEMBRANE PERMEABILITY & MEMBRANE TRANSPORT PROTEINS
• An important technique that has permitted major advances in our knowledge about transport proteins is patch clamping.
• It is an electrophysiological test to study single or multiple ion channels in cells.
• It is a refinement of the voltage clamp method.
• It was developed by Erwin Neher and Bert Sakmann in the late 1970s and early 1980s.
• This discovery made it possible to record the currents of single ion channel molecules for the first time, which improved
understanding of the involvement of channels in fundamental cell processes such as action potentials and nerve activity.
• Neher and Sakmann received the Nobel Prize in Physiology or Medicine in 1991 for this work.
Procedure –
• A micropipette is placed on the membrane of a cell and forms a tight seal to the membrane.
• The patch of membrane under the pipette tip usually contains only a few transport proteins, allowing for their detailed biophysical
study.
• The cell can be left intact (cell-attached patch clamp).
• Alternatively, the patch can be pulled loose from the cell, forming an inside-out patch.
• A third alternative is to suck out the patch with the micropipette still attached to the rest of the cell membrane, providing direct
access to the interior of the cell (whole cell recording)
Your Answer. Not Attempted
Correct Answer. b
Copyright © 2014 Delhi Academy of Medical Sciences, All Rights Reserved. 9/146
(12). Sleep spindles and K complexes are characteristic of –
a. NREM sleep Stage I
b. NREM sleep Stage II
c. NREM sleep stage III
d. REM sleep
Solution. Ans-12: (b) NREM sleep Stage II
Ref: Ganong’s ‘Review of Medical Physiology’; 24/e; Guyton and Hall’s ‘Text Book of Medical Physiology’; 12/e
Sol :
• During each night, a person goes through stages of two types of sleep that alternate with eachother.
• They are – Non-rapid eye movement (NREM) sleep and Rapid eye movement (REM) sleep.
• Most sleep during each night is NREM sleep whereas, episodes of REM sleep occupy about 25 percent of the sleep time in young
adults.
• Each episode normally recurs about every 90 minutes.
NREM SLEEP –
• It is also known as slow wave sleep.
• It is the deep, restful sleep that the person experiences during the first hour of sleep after having been awake for many hours.
• It is divided into four stages –
o As a person begins to fall asleep and enters stage 1, the EEG shows a low-voltage, mixed frequency pattern. A theta rhythm (4–7
Hz)
can be seen at this early stage of slow-wave sleep. Throughout NREM sleep, there is some activity of skeletal muscle but no eye
movements occur.
o Stage 2 of NREM sleep is marked by the appearance of sinusoidal waves called sleep spindles (12–14 Hz) and occasional high
voltage
biphasic waves called K complexes.
o In stage 3 of NREM sleep, a high-amplitude delta rhythm (0.5–4 Hz) dominates the EEG waves.
o Maximum slowing with large waves is seen in stage 4 of NREM sleep.
REM SLEEP –
• It is so named because of the characteristic eye movements that occur during this stage of sleep.
• Eyes undergo rapid movements despite the fact that the person is still asleep.
• It is also known as paradoxical sleep / desynchronized sleep.
• This type of sleep is not so restful, and it is usually associated with vivid dreaming.
REM sleep has several important characteristics:
1. It is an active form of sleep usually associated with dreaming and active bodily muscle movements.
2. The person is even more difficult to arouse by sensory stimuli than during deep slow-wave sleep, and yet people usually awaken
spontaneously in the morning during an episode of REM sleep.
3. Muscle tone throughout the body is exceedingly depressed, indicating strong inhibition of the spinal muscle control areas.
4. Heart rate and respiratory rate usually become irregular, which is characteristic of the dream state.
5. Despite the extreme inhibition of the peripheral muscles, irregular muscle movements do occur. These are in addition to the rapid
movements of the eyes.
6. The brain is highly active in REM sleep, and overall brain metabolism may be increased as much as 20 percent. The
electroencephalogram (EEG) shows a pattern of brain waves similar to those that occur during wakefulness. This type of sleep is
also
called paradoxical sleep because it is a paradox that a person can still be asleep despite marked activity in the brain.
Your Answer. b
Correct Answer. b
Copyright © 2014 Delhi Academy of Medical Sciences, All Rights Reserved. 10/146
(13). The centre of satiety is located in which part of hypothalamus?
a. Dorsomedial nucleus
b. Ventromedial nucleus
c. Arcuate nucleus
d. Medial preoptic nucleus
Solution. Ans-13: (b) Ventromedial nucleus
Ref: Guyton and Hall’s ‘Text Book of Medical Physiology’; 10/e, pg 682-684, 838
Sol :
FUNCTIONS OF HYPOTHALAMUS –
1. Cardiovascular regulation –
• Stimulation of different areas throughout the hypothalamus can cause every known type of neurogenic effect on the cardiovascular
system; including increased arterial pressure, decreased arterial pressure, increased heart rate and decreased heart rate.
• In general, stimulation in the posterior and lateral hypothalamus increases the arterial pressure and heart rate; whereas stimulation
in the preoptic area has opposite effects, causing a decrease in both heart rate and arterial pressure.
• These effects are transmitted mainly through specific cardiovascular control centres in the reticular regions of the medulla and pons.
2. Regulation of body temperature –
• The anterior portion of the hypothalamus, especially the preoptic area, is concerned with the regulation of body temperature.
• An increase in the temperature of the blood flowing through this area increases the activity of this temperature-sensitive neurons,
whereas a decrease in temperature decreases their activity.
• In turn, these neurons control mechanisms for increasing or decreaing body temperature.
3. Regulation of body water –
• The hypothalamus regulates body water in two ways –
o By creating the sensation of thirst, which makes the person or animal drink water; and
o By controlling the excretion of water into the urine.
• An area called the thirst centre is located in the lateral hypothalamus.
• When the fluid electrolytes in either this centre or allied areas of the hypothalamus become too concentrated, the person or animal
develops an intense desire to drink water.
• It will search out the nearest source of water and drink enough to return the electrolyte concentration of the thirst centre to normal.
• Control of renal excretion of water is vested mainly in the supraoptic nucleus.
• When the body fluids become too concentrated, the neurons of this area become stimulated.
• The nerve fibres from these neurons project downward through the infundibulum of the hypothalamus into the posterior pituitary
gland, where the nerve endings secrete a hormone called antidiuretic hormone (also called vasopressin).
• This hormone is then absorbed into the blood and acts on the collecting ducts of the kidneys to cause massive reabsorption of water,
thereby decreasing the loss of water into the urine but allowing continuing excretion of electrolytes; thus decreasing the concentration of
the body fluids back towards normal.
4. Regulation of uterine contractility and milk ejection from the breast –
• Stimulation of the paraventricular nucleus causes its neuronal cells to secrete the hormone oxytocin.
• This in turn causes increased contractility of the uterus as well as contraction of the myoepithelial cells that surround the alveoli of
the breasts, which then causes the alveoli to empty the milk through the nipples.
• At the end of pregnancy, especially large quantities of oxytocin are secreted, and this secretion helps to promote labour contractions
that expel the baby.
• Also, when a baby suckles the mother’s breast, a reflex signal from the nipple to the posterior hypothalamus causes oxytocin release,
and the oxytocin then performs the necessary function of contracting the ductules of the breast, thereby expressing milk through the
nipples so that the baby can nourish itself.
5. Gastrointestinal and feeding regulation –
• Stimulation of several areas of the hypothalamus causes an animal to experience extreme hunger, a voracious appetitie and an
intense desire to search for food.
• The area most associated with hunger is the lateral hypothalamic area.
• Conversely, damage to this area on bith sides of the hypothalamus causes the animal to lose desire for food, sometimes causing
lethal starvation.
• A centre the opposes the desire for food, called the satiety centre, is located in the ventromedial nucleus.
• When this centre is stimulated electrically, an animal that is eating food suddenly stops eating and shows complete indifference to
food.
• However, if this area is destroyed bilaterally, the animal cannot be satiated.
• Instead, its hypothalamic hunger centres become overactive; so that it has a voracious appetite, resulting eventually in tremendous
obesity.
• Another area of the hypothalamus that enters into the overall control of gastrointestinal activity is the mamillary bodies.
• These control at least partially, the patterns of many feeding reflexes, such as licking the lips and swallowing.
6. Endocrine hormone secretion by the anterior pituitary gland –
• Stimulation of certain areas of the hypothalamus also causes the anterior pituitary gland to secrete its endocrine hormones.
• The anterior pituitary gland receives its blood supply mainly from the blood that flows first through bthe lower part of hypothalamus
and then through the anterior pituitary vascular sinuses.
• As the blood courses through the hypothalamus before reaching the anterior pituitary, releasing and inhibitory hormones are
secreted into the blood by various hypothalamic nuclei.
• These hormones are then transported via the blood to the anterior pituitary, where they act on the glandular cells to control release
of specific anterior pituitary hormones.
Examples of hypothalamic releasing factors and the anterior pituitary hormones regulated by them –
Hypothalamic releasing factor Function on anterior pituitary
Corticotropin-releasing hormone (CRH) Causes release of ACTH
Growth hormone releasing hormone (GHRH) Causes release of growth hormone
Growth hormone inhibitory hormone (GHIH) Inhibits release of growth hormone
Gonadotropin releasing hormone (GnRH) Causes release of FSH and LH
Thyrotropin releasing hormone (TRH) Stimulates secretion of TSH and prolactin
Prolactin inhibiting factor (PIF) = Dopamine Inhibits release of prolactin
Your Answer. b
Correct Answer. b
Copyright © 2014 Delhi Academy of Medical Sciences, All Rights Reserved. 11/146
(14). Maximum osmotic gradient in kidney is found in :
a. Outer medulla
b. Inner medulla
c. Outer cortex
d. Inner cortex
Solution. Ans-14: (b) Inner medulla
Ref.: Read the text below
Sol :
• The kidney layer of the kidney is isotonic with the blood : -300 milliosmoles/liter.
• The innermost layer (medulla) is very hypertonic : ~ 1200 milliosmoles/liter.
• Osmotic gradient is produced by a countercurrent mechanism located in the loop of Henle.
• The countercurrent mechanism is based upon the Na pump; by pumping large quantities of Na into the interstitial fluid in the
medulla a very high concentration is built up.
Your Answer. b
Correct Answer. b
(15). Source of progesterone during normal menstrual cycle :
a. Corpus luteum
b. Stroma
c. Surface epithelium of ovary
d. None of these
Solution. Ans-15: (a) Corpus luteum
Ref.: Read the text below
Sol :
• Follicles in the ovary begin developing under the influence of a complex interplay of hormones, and after several days one or
occasionally two become dominant (non-dominant follicles atrophy and die).
• Approximately mid-cycle, 24–36 hours after the Luteinizing Hormone (LH) surges, the dominant follicle releases an ovum, or egg,
in an event called ovulation.
• After ovulation, the egg only lives for 24 hours or less without fertilization while the remains of the dominant follicle in the ovary
become a corpus luteum; this body has a primary function of producing large amounts of progesterone.
• Under the influence of progesterone, the endometrium (uterine lining) changes to prepare for potential implantation of an embryo
to establish a pregnancy.
• If implantation does not occur within approximately two weeks, the corpus luteum will involute, causing sharp drops in levels of
both progesterone and estrogen.
• The hormone drop causes the uterus to shed its lining and egg in a process termed menstruation.
Your Answer. a
Correct Answer. a
Copyright © 2014 Delhi Academy of Medical Sciences, All Rights Reserved. 12/146
(16). GFR decreases with the following:
a. Hypotension
b. Hypoproteinemia
c. Blood loss
d. All the above
Solution. Ans-16 : (d) All the above
Ref.: Read the text below
Sol :
• Glomerular filtration rate (GFR) is the volume of fluid filtered from the renal glomerular capillaries into the Bowman’s capsule per
unit
time. Clinically, this is often measured to determine renal function.
• Hypotension causes decreased blood supply to the kidneys thereby causing decrease in GFR.
• Hypoproteinemia causes decrease in the plasma oncotic pressure leading to decreased GFR.
• Medications like ACE inhibitors after renal perfusion by dilating the efferent arteriole of the glomerulus reducing renal perfusion
pressure and decreasing GFR.
Your Answer. d
Correct Answer. d
(17). Function of Kupffer cells in liver is :
a. Phagocytic
b. Lytic
c. Secretory
d. Excretory
Solution. Ans-17: (a) Phagocytic
Ref.: Read the text below
Sol :
• The tissue macrophages include the Kupffer cells of liver, pulmonary alveolar macrophages, osteoclasts, and microglia in the
brain,
and come from the circulation.
• In the past, they have been called the reticuloendothelial system, but the general term tissue macrophage system seems more
appropriate.
• The macrophages become activated by lymphokines from T lymphocytes
The activated macrophages migrate in response to chemotactic stimulus and engulf and kill bacteria by processes generally similar
to
those occurring in neutrophils.
Your Answer. a
Correct Answer. a
Copyright © 2014 Delhi Academy of Medical Sciences, All Rights Reserved. 13/146
(18). Atrial natriuretic polypeptide (ANP) acts by :
a. Decreasing aldosterone production
b. Decreasing sodium reabsorption from renal tubules
c. Inhibiting release of renin
d. All of the above
Solution. Ans-18: (d) All of the above
Ref.: Read the text below
Sol :
ANP binds to a specific set of receptors – ANP receptors. Receptor-agonist binding causes a reduction in blood volume and,
therefore, a
reduction in cardiac output and systemic blood pressure. Lipolysis is increased and renal sodium reabsorption is decreased. The
overall
effect of ANP on the body is to counter increases in blood pressure and volume caused by the renin-angiotensin system.
Renal
· Dilates the afferent glomerular arteriole, constricts the efferent glomerular arteriole, and relaxes the mesangial cells.
· Increases blood flow through the vasa recta, which will wash the solutes (NaCl and urea) out of the medullary interstitium.
· Decreases sodium reabsorption in the distal convoluted tubule (interaction with NCC ) and cortical collecting duct of the nephron via
guanosine 3',5'-cyclic monophosphate (cGMP) dependent phosphorylation of ENaC
· Inhibits renin secretion, thereby inhibiting the renin-angiotensin-aldosterone system.
· Reduces aldosterone secretion by the adrenal cortex.
· Atrial natriuretic peptide (ANP) increases Na+ excretion by decreasing the amount of Na+ reabsorbed from the inner medullary
collecting duct via a decrease in the permeability of the apical membrane of the collecting duct epithelial cells.
Vascular
Relaxes vascular smooth muscle in arterioles and venules by:
· Membrane Receptor-mediated elevation of vascular smooth muscle cGMP
· Inhibition of the effects of catecholamines
Cardiac
· Inhibits maladaptive cardiac hypertrophy
· Mice lacking cardiac NPRA develop increased cardiac mass and severe fibrosis and die suddenly
· Re-expression of NPRA rescues the phenotype.
It may be associated with isolated atrial amyloidosis.
Adipose tissue
· Increases the release of free fatty acids from adipose tissue. Plasma concentrations of glycerol and nonesterified fatty acids are
increased by i.v. infusion of ANP in humans.
· Activates adipocyte plasma membrane type A guanylyl cyclase receptors NPR-A
· Increases intracellular cGMP levels that induce the phosphorylation of a hormone-sensitive lipase and perilipin A via the activation
of
a cGMP-dependent protein kinase-I (cGK-I)
Does not modulate cAMP production or PKA activity
Your Answer. d
Correct Answer. d
Copyright © 2014 Delhi Academy of Medical Sciences, All Rights Reserved. 14/146
(19). Patellar tendon reflex is due to stimulation of –
Copyright © 2014 Delhi Academy of Medical Sciences, All Rights Reserved. 15/146
a. Golgi tendon organ
b. Dynamic muscle fibre
c. Static nuclear chain fibre
d. Both muscle spindle and Golgi tendon organ
Solution. Ans-19: (b) Dynamic muscle fibre
Ref: Ganong’s ‘Review of Medical Physiology’; 24/e; Guyton and Hall’s ‘Text Book of Medical Physiology’; 12
Sol :
· When a skeletal muscle with an intact nerve supply is stretched, it contracts.
· This response is called the stretch reflex or myotatic reflex.
· The stimulus that initiates this reflex is stretch of the muscle, and the response is contraction of the muscle being stretched.
· The sense organ is a small encapsulated spindlelike or fusiform shaped structure called the muscle spindle, located within the
fleshy
part of the muscle.
· The impulses originating from the spindle are transmitted to the CNS by fast sensory fibers that pass directly to the motor neurons
that supply the same muscle.
· The neurotransmitter at the central synapse is glutamate.
· The stretch reflex is the best known and studied monosynaptic reflex and is typified by the knee jerk reflex.
Each muscle spindle has three essential elements –
· A group of specialized intrafusal muscle fibres with contractile polar ends and a non-contractile centre,
· Large diameter myelinated afferent nerves (Types Ia and II) originating in the central portion of the intrafusal fibres, and
· Small diameter myelinated efferent nerves supplying the polar contractile regions of the intrafusal fibres.
1. Intrafusal fibres –
· The intrafusal fibres are positioned parallel to the extrafusal fibres (the regular contractile units of the muscle) with the ends of the
spindle capsule attached to the tendons at either end of the muscle.
· The intrafusal fibres do not contribute to the overall contractile force of the muscle, but rather serve a pure sensory function.
· Changes in muscle length are associated with changes in joint angle; thus muscle spindles provide information on position (i.e.
proprioception).
· There are two types of intrafusal fibres in mammalian muscle spindles –
o The first type contains many nuclei in a dilated central area and is called a nuclear bag fibre;
o The second type, the nuclear chain fibre, is thinner and shorter and lacks a definite bag.
· There are two subtypes of nuclear bag fibres – static and dynamic.
· Typically, there are two or three nuclear bag fibres and about five nuclear chain fibres per spindle.
2. Afferent nerve endings –
· There are two kinds of sensory endings in each spindle –
o A single primary (Group Ia) ending and
o Up to eight secondary (Group II) endings
· The Ia afferent fibre wraps around the centre of the dynamic and static nuclear bag fibres and nuclear chain fibres.
· Group II sensory fibres are located adjacent to the centres of the static nuclear bag and nuclear chain fibres; these fibres do not
innervate the dynamic nuclear bag fibres.
· Ia afferents are very sensitive to the velocity of change in muscle length during a stretch (dynamic response); thus they provide
information about the speed of movements and allow for quick corrective movements.
· The steady state (tonic) activity of Group Ia and II afferents provide information on the steady state length of the muscle (static
response).
3. Efferent nerves –
· The spindles have a motor nerve supply of their own.
· These nerves are 3-6 μm in diameter.
· They constitute about 30% of the fibres in the ventral roots, and are called γ-motor neurons.
· There are two types of γ-motor neurons –
o Dynamic – which supply the dynamic nuclear bag fibres and
o Static – which supply the static nuclear bag and nuclear chain fibres.
· Activation of the dynamic γ-motor neurons increases the dynamic sensitivity of the Group Ia nerve endings.
Activation of the static γ-motor neurons increases the tonic level of activity in both Group Ia and II nerve endings, decreases the
dynamic
sensitivity of the Group Ia afferents, and can prevent silencing of Ia afferents during muscle stretch
Your Answer. a
Correct Answer. b
Copyright © 2014 Delhi Academy of Medical Sciences, All Rights Reserved. 16/146
Copyright © 2014 Delhi Academy of Medical Sciences, All Rights Reserved. 17/146
(20). Concentration and attention are the functions of –
Copyright © 2014 Delhi Academy of Medical Sciences, All Rights Reserved. 18/146
a. Frontal lobe
b. Parietal lobe
Copyright © 2014 Delhi Academy of Medical Sciences, All Rights Reserved. 19/146
c. Basal ganglia
Copyright © 2014 Delhi Academy of Medical Sciences, All Rights Reserved. 20/146
d. Thalamus
Copyright © 2014 Delhi Academy of Medical Sciences, All Rights Reserved. 21/146
Solution. Ans-20: (a) Frontal lobe
Ref: http://biau.org/about-brain-injuries/cognitive-skills-of-the-brain/
Sol :
COGNITIVE SKILLS OF THE BRAIN –
There are six functional components inside of the brain –
· Frontal Lobe
· Parietal Lobe
· Occipital Lobe
· Temporal Lobe
· Brain Stem
· Cerebellum
FRONTAL LOBE – The frontal lobe links and integrates all components of behavior at the highest level. Emotion and social adjustment
and impulse control are also localized here. Injury to parts of the frontal lobe may cause an inability to move part of the body or the
whole side of the body. Speech may become halting, disorganized or be stopped except for single explosive words. Personality may
change. Social rules of behavior may be disregarded. The executive functions, planning, abstract reasoning, impulse control, sustained
attention and insight are all located here. The frontal lobe is highly susceptible to injury.
Functions –
· Initiation
· Problem solving
· Judgment
· Inhibition of behavior
· Planning/anticipation
· Self-monitoring
· Motor planning
· Personality/emotions
· Awareness of abilities / limitations
· Organization
· Attention/concentration
· Mental flexibility
· Speaking (expressive language)
Observed problems –
· Emotion (i.e., depression, anxiety, personality changes,
aggression, acting out, and social inappropriateness)
PARIETAL LOBE - The parietal lobe is largely responsible for construction ability and language. Injury to the front parts of this lobe
may cause someone to lose sensation on parts of the body. With an injury in this area, one may become disoriented. Recall of long term
memories may be mixed up in time or sequencing. They may become easily lost or confuse left and right. They may have difficulty
recognizing or naming what they see. Injury may also produce disorders in the ability to read, write or perform math calculations. This
area also includes conscious sensation and voluntary motion.
Functions –
· Sense of touch
· Differentiation: size, shape, color
· Spatial perception
· Visual perception
· Academic skills (reading)
Observed problems –
· Sensation (i.e., touch, taste, and smell)
OCCIPITAL LOBE – Injury to this area usually results in “blindness” to part or all of the visual field. Usually people experience “holes”
or “blind spots” in what they see. There may be problems picking things out of space or they may misperceive pictures or objects.
Recognition of colors may also be disturbed.
Functions –
· Vision
· Reading (perception and recognition of printed words)
Observed problems –
· Depth perception
· Color perception
· Difficulty tracking moving objects
· Partial or total blindness
TEMPORAL LOBE – The temporal lobe perceives and recognizes verbal material. It is among the most frequently injured parts of the
brain during head injury. A person may have difficulty screening out distractions. Injury to the upper temporal area can cause someone
to misunderstand what is said. They may make sounds like words but which are not recognizable as words at all. They may also
misunderstand body language. Emotional changes such as unexplained panic or unexpected tearfulness may be noted. Left temporal
area includes production of speech, naming and verbal memory. The right temporal area includes musical abilities, foreign languages,
visual memory, and comprehension of the environment.
Functions –
· Memory
· Hearing
· Understanding language (receptive language)
· Organization and sequencing
· Musical awareness
Observed problems –
· Thinking (i.e., memory and reasoning)
· Language (i.e., communication, expression, and
understanding)
CEREBELLUM –
Functions –
· Coordination of voluntary movement
· Balance and equilibrium
· Some memory for reflex motor acts
Observed problems –
· Loss of ability to coordinate fine movements
· Loss of ability to walk
· Inability to reach out and grab objects
· Tremors
· Dizziness (vertigo)
· Slurred speech (scanning speech)
· Inability to make rapid movements
BRAIN STEM – The brain stem plays a vital role in basic attention, arousal, and consciousness. All information to and from our body
passes through the brain stem on the way to or from our brain. Like the frontal and temporal lobes, the brain stem is located in an area
near bony protrusions making it vulnerable to damage during trauma.
Functions –
· Breathing
· Heart Rate
· Swallowing
· Reflexes to seeing and hearing (startling response)
· Controls sweating, blood pressure, digestion, temperature
(autonomic nervous system)
· Affects level of alertness
· Ability to sleep
· Sense of balance (vestibular function)
Observed problems –
· Decreased vital capacity in breathing, important for speech
· Swallowing food and water (dysphasia)
· Difficulty with organization/perception of the environment
· Problems with balance and movement
· Dizziness and nausea (vertigo)
· Sleeping difficulties (insomnia, sleep apnea)
Copyright © 2014 Delhi Academy of Medical Sciences, All Rights Reserved. 22/146
Your Answer. c
Correct Answer. a
(21). The biosynthesis of the enzyme pyruvate carboxylase is repressed by ?
a. Insulin
b. Cortisol
c. Glucagon
d. Epinephrine
Solution. Ans-21: (a) Insulin
Ref: Read the text below
Sol :
• Pyruvate carboxylase is gluconeogenic hormone.
• Insulin inhibits gluconeogenesis and has got inhibitory effect on pyruvate carboxylase.
Your Answer. a
Correct Answer. a
(22). Protein purification and separation can be done by all except ?
a. Chromatography
b. Centrifugation
c. Electrophoresis
d. Densitometery
Solution. Ans-22: (d) Densitometery
Ref: Read the text below
Sol :
• Remember : For purification and separation following methods are used.
1. Chromatography
2. Dialysis
3. Centrifugation
4. Electrophoresis
5. Precipitation
Your Answer. b
Correct Answer. d
(23). A 32 years old man is starving for the last 2 days. Which of the following enzymes will be activated in his body?
a. Glucokinase
b. Pyrovate dehydrogenase
c. Glycogen phosphorylase
d. Glucose 6 phosphatase
Solution. Ans-23: (c) Glycogen phosphorylase
Ref:Read the text below
Sol:
· When the body begins to starve it needs to break down its glycogen reserves in order to neurish the body so Glycogen
phosphorylase
will be activated
Your Answer. c
Correct Answer. c
Copyright © 2014 Delhi Academy of Medical Sciences, All Rights Reserved. 23/146
(24). Which of the following statement regarding rothera’s test is untrue:
a. Detects acetone
b. Brown colour is positive
c. May be used in diabetes mellitus patients to detect complications
d. Sodium nitoprusside is required
Solution. Ans-24: (b) Brown colour is positive
Ref:Read the text below
Sol:
· Rotheras test is a method of testing urine for the presence of acetone or acetoacetic acid: a sign of diabetes mellitus.
· Strong ammonia is added to a sample of urine saturated with ammonium sulphate crystals and containing a small quantity of
sodium
nitroprusside.
· A purple colour confirms the presence of acetone or acetoacetic acid
Your Answer. d
Correct Answer. b
(25). Alkaptonurea is characterized by:
a. Increased urinary homocystine
b. Increased conjugated bilirubin
c. Increased serum phenylalanine
d. Excessive homogentisic acid in urine
Solution. Ans-25: (d) Excessive homogentisic acid in urine
Ref:Read the text below
Sol:
· Alkaptonuria (black urine disease or alcaptonuria) is a rare inherited genetic disorder of phenylalanine and tyrosine metabolism.
· This is an autosomalrecessive condition that is due to a defect in the enzyme homogentisate 1,2-dioxygenase ,which participates
in the degradation oftyrosine.
· As a result, a toxic tyrosine byproduct called homogentisic acid (or alkapton) accumulates in the blood and is excreted in urine in
large amounts(hence -uria).
· Excessive homogentisic acid causes damage to cartilage (ochronosis, leading to osteoarthritis) and heart valves as well as
precipitating as kidney stones.
Your Answer. d
Correct Answer. d
(26). Microsatellite sequence
a. Can determine paternity
b. Consists of 10-15 base pair sequence
c. Is irreproducible by PCR
d. Are neutral regions of DNA
Solution. Ans-26: (a) Can determine paternity
Ref:Read the text below
Sol:
Can determine paternity
· Microsatellites, Simple Sequence Repeats (SSRs), or tandem repeats, are repeating sequences of 1-6 base pairs of DNA.
· Microsatellites can be amplified for identification by the polymerase chain reaction (PCR) process, using the unique sequences of
flanking regions as primers so they can be reproduced.
· As there are often many alleles present at a microsatellite locus,genotypes within pedigrees are often fully informative, in that
the progenitor of a particular allele can often be identified. In this way, microsatellites are ideal for determining paternity, population
genetic studies and recombination mapping.
Microsatellites owe their variability to an increased rate of mutation compared to other neutral regions of DNA.
Your Answer. b
Correct Answer. a
Copyright © 2014 Delhi Academy of Medical Sciences, All Rights Reserved. 24/146
(27). Replication occurs during
a. G1 phase
b. S phase
c. G2 phase
d. M phase
Solution. Ans-27: (b) S phase
Ref:Read the text below
Sol:
· The sequence of cell cycle is, Go-G1-G2-M M is mitosis phase, where cell division occurs.
· Before cell division, nuclear division should occur,
· For nuclear division to occur, we require the synthesis of two ds DNA from single dsDNA (ie) replication has to occur.This occurs
during S phase/synthetic phase.
Your Answer. d
Correct Answer. b
(28). VMA is excreted excessively in the urine. What may be the diagnosis ?
a. Carcinoid syndrome
b. Phaeochromocytoma
c. Cushing’s syndrome
d. None
Solution. Ans-28: (b) Phaeochromocytoma
Ref: Read the text below
Sol :
• VMA is end product of catabolism of catecholamines.
• In phaeochromocytoma and neuroblastoma there is excessive synthesis of catecholamines which causes enhanced synthesis of
VMA
and it’s excretion in the urine.
Your Answer. b
Correct Answer. b
(29). FIGLU is a metabolite of ?
a. Folic acid
b. Tyrosine
c. Histidine
d. Alanine
Solution. Ans-29: (c) Histidine
Ref: Read the text below
Sol :
• FIGLU is a intermediate metabolite of histidine catabolism.
• It is excreted in urine in increased quantity when there is folic acid deficiency as folic acid is required for further metabolism of
FIGLU.
Your Answer. a
Correct Answer. c
Copyright © 2014 Delhi Academy of Medical Sciences, All Rights Reserved. 25/146
(30). False about tryptophan ?
a. Nonessential amino acid
b. Involved in serotonin synthesis
c. Involved in niacin synthesis
d. Involved in melatonin synthesis
Solution. Ans-30: (a) Nonessential amino acid
Ref: Read the text below
Sol :
• Tryptophan is an essential amino acid, as it’s synthesis in body does not take place.
• Tryptophan is involved in synthesis of melationin, serotonin, and niacin by various pathways.
Your Answer. a
Correct Answer. a
(31). Which of the following is true about diazoxide except
a. k+ channel opener
b. Can be used as antihypertensive agent
c. Causes severe hypoglycemia
d. Used in insulinoma
Solution. Ans 31: (c) Causes severe hypoglycemia
Ref– Harrison -1473, 2228
Sol
· Diazoxide is a benzothiadiazide whose hyperglycemic effect is attributed to inhibition of insulin release. Other agents effective in
some patients to controlthe hypoglycemia include verapamil and diphenylhydantoin. Long-actingsomatostatin analogues such as
octreotide are acutely effective in 40% ofpatients..
· Diazoxide is similar chemically to the thiazide diuretics but has no diuretic activity.
· Mechanism of Diazoxide: Hyperpolarization of smooth muscle membrane through opening of potassium channels
· Diazoxide inhibits insulin release from the pancreas (probably by opening potassium channels in the beta cell membrane) and is
used
to treat hypoglycemia secondary to insulinoma. .
· In contrast to the structurally related thiazide diuretics, Diazoxide causes renal salt and water retention.
Your Answer. c
Correct Answer. c
Copyright © 2014 Delhi Academy of Medical Sciences, All Rights Reserved. 26/146
(32). A patient develops hypoglycemia. He was on insulin and acarbose. For treatment of above what is to be given
a. Glucose
b. Maltose
c. Sucrose
d. Starch
Solution. Ans 32: (a) Glucose
Ref– Read the text below
Sol
Urgent treatment of hypoglycemia.
· Oral treatment with glucose tablets or glucose-containing fluids, candy, or food is appropriate if the patient is able and willing to
take these.
· A reasonable initial dose is 20 g of glucose. If the patient is unable or unwilling, because of neuroglycopenia, to take carbohydrates
orally, parenteral therapy is necessary. Intravenous glucose (25 g) should be given and followed by a glucose infusion guided by
serial
plasma glucose measurements.
· If intravenous therapy is notpractical, subcutaneous or intramuscular glucagon (1.0 mg in adults) can be used, particularly in
patients with T1DM. It also stimulates insulin secretion and is therefore less useful in T2DM.
· These treatments raise plasma glucose concentrations only transiently, and patients should therefore be urged to eat as soon as is
practical to replete glycogen stores. Since the patient is taking acarbose, polysaccharide and disaccharides would not be effective in
increasing blood glucose
Your Answer. a
Correct Answer. a
(33). Which does not cause hypoglycemia
a. Insulin
b. Glimipiride
c. Nateglinide
d. Acarbose
Solution. Ans 33: (d) Acarbose
Ref– Read the text below
Sol
· Nateglinide, a D-phenylalanine derivative, is the latest insulin secretagogue to become clinically available. Incidence of
hypoglycemia with nateglinide may be the lowest of all the secretagogues.
· Prominent adverse effects of acarbose include flatulence, diarrhea, and abdominal pain and result from the appearance of
undigested carbohydrate in the colon that is then fermented into short-chain fatty acids, releasing gas.
· Althoughnot a problem with monotherapy or combination therapy with a biguanide, hypoglycemia may occur with concurrent
sulfonylurea treatment.
Acarbose as a monotherapy dose not cause hypoglycemia while nateglinidecan still cause it
Your Answer. d
Correct Answer. d
Copyright © 2014 Delhi Academy of Medical Sciences, All Rights Reserved. 27/146
(34). Which does not causes megaloblastic anemia
a. Phenytoin
b. Chloroquine
c. Sulfasalazine
d. Alcohol
Solution. Ans 34: (b) Chloroquine
Ref– Katzung 11th edition & Harrison 17th EditionPg.601,605
Sol
· Low folate levels and megaloblastic anemia have been reported with phenytoin usage.
· Sulfasalazine can also impair folate absorption, and patients should be given folic acid supplements.
· Alcohol is a well known cause of megaloblastic anemia.
· Chloroquine is usually very well tolerated, even with prolonged use. Pruritus is a frequent side effect. Nausea, vomiting, abdominal
pain, headache, anorexia, malaise, blurring of vision, and urticaria are uncommon.
· Rare reactions include hemolysis in glucose-6-phosphate dehydrogenase (G6PD)-deficient persons, impaired hearing, confusion,
psychosis, seizures, agranulocytosis, exfoliative dermatitis, alopecia, bleaching of hair, hypotension, and electrocardiographic
changes
(QRS widening, T-wave abnormalities).
The long-term administration of high doses of chloroquine for rheumatologic diseases can result in irreversible ototoxicity,
retinopathy,
myopathy, and peripheral neuropathy
Your Answer. b
Correct Answer. b
(35). Which of the following drug is used in sickle cell anemia
a. Hydroxyurea
b. Carmustine
c. Paclitaxel
d. Bleomycin
Solution. Ans 35: (a) Hydroxyurea
Ref– Harrison 17th EditionPg. 594-97
Sol
· The most significant advance in the therapy of sickle cell anemia has been the introduction of hydroxyurea as a mainstay of
therapy
for patients with severe symptoms.
· Hydroxyurea (10–30 mg/kg per day) increases fetal hemoglobinand may also exert beneficial affects on RBC hydration, vascular
wall
adherence, and suppression of the granulocyte and reticulocyte counts.
· Hydroxyurea should be considered in patients experiencing repeated episodes of acute chest syndrome or with more than three
crises per year requiring hospitalization. HbF levels increase in most patients within a few months.
· Bone marrow transplantation can provide definitive cures but is known to be effective and safe only in children.
· Gene therapy for sickle cell anemia is being intensively pursued, but no safe measures are currently available.
· Agents blocking RBC dehydration or vascular adhesion, such as clotrimazole or magnesium, may have value as an adjunct to
hydroxyurea therapy, pending the completion of ongoing trials. Combinations of clotrimazole and magnesium are being evaluated.
Your Answer. a
Correct Answer. a
Copyright © 2014 Delhi Academy of Medical Sciences, All Rights Reserved. 28/146
(36). Which of the following is not a side effect of naloxone
a. Seizure
b. Hypertension
c. Pulmonary edema
d. Ventricular dysrrthymia
Solution. Ans 36: (a) Seizure
Ref– http://www.netdoctor.co.uk/medicines/100003478.htm
Sol
SIDE EFFECTS OF NALOXONE
• Nausea and vomiting
• Sweating
• Increased heart rate (tachycardia)
• Hyperventilation
• Increased blood pressure
• Reversal of pain relief if larger than necessary doses are given
• Irregular heart beat (ventricular arrhythmias)
• Accumulation of fluid in the lungs (pulmonary oedema)
Your Answer. Not Attempted
Correct Answer. a
(37). 30 yrs old female was diagnose sputum smear positive tuberculosis. Her 3 yrs old child is healthy. what chemoprophylaxis is to
be given
a. INH 3 mg/kg for 3 months
b. INH 5 mg/kg for 3 months
c. INH 3 mg/kg for 6 months
d. INH 5 mg/kg for 6 months
Solution. Ans 37: (d) INH 5 mg/kg for 6 months
Ref– http://www.health.gov.bz/www/attachments/569_TUBERCULOSIS%20GUID ELINE.
Sol
· Current international guidelines recommend 6-9 months of Isoniazid (INH) preventive chemotherapy to prevent the development of
active tuberculosis in children exposed to smear positive tuberculosis patients.
· The purpose ofthis study was to determine adherence to six months of supervised INHprophylaxis and outcome in children with
household exposure to an adultpulmonary tuberculosis index case in VaraminTehran, Iran.
· The dose of INH in children is same as that of adults (5mg/kg)
Your Answer. b
Correct Answer. d
Copyright © 2014 Delhi Academy of Medical Sciences, All Rights Reserved. 29/146
(38). Which of the following is not an indication of cotrimoxazole
a. Lower UTI
b. Prostatis
c. Chancroid
d. Typhoid
Solution. Ans 38: (c) Chancroid
Ref– Katzung 11th Edition Pg.816
Sol
Oral Trimethoprim-Sulfamethoxazole (Tmp-Smz )
· A combination of trimethoprim-sulfamethoxazole is effective treatment for a wide variety of infections including P jiroveci
pneumonia, shigellosis, systemic salmonella infections, urinary tract infections, prostatitis, and some nontuberculous mycobacterial
infections.
· It is active against most Staphylococcus aureus strains, both methicillin-susceptible and methicillinresistant, and against respiratory
tract pathogens such as the pneumococcus, Haemophilus sp, Moraxella catarrhalis, and Klebsiella pneumoniae (but not
Mycoplasma
pneumoniae).
· However, the increasing prevalence of strains of E coli (up to 30% or more) and pneumococci that are resistant to
trimethoprimsulfamethoxazole must be considered before using this combination for empirical therapy of upper urinary tract
infections
or pneumonia.
Your Answer. c
Correct Answer. c
(39). Bleomycin toxicity is characterized by destruction of
a. Endothelial cells
b. Type I pneumocytes
c. Type II pneumocyutes
d. Alveolar macrophages
Solution. Ans 39: (b) Type I pneumocytes
Ref– Fishman pulmonary disease 4th ed,pg1069
Sol
· Type I pneumocytes destruction is followed by Type II pneumocyutes hyperplasia & dysplasia leading to activation of fibroblast
,collagen deposition & fibrosis
Your Answer. a
Correct Answer. b
Copyright © 2014 Delhi Academy of Medical Sciences, All Rights Reserved. 30/146
(40). Use of tamoxifen for breast cancer can cause all of the following adverse effects, except
a. Thromboembolism
b. Endometrial carcinoma
c. Carcinoma in contralateral breast
d. Cataract
Solution. Ans 40: (c) Carcinoma in contralateral breast
Ref– Katzung 11th Edition Pg.716,958
Sol
· Tamoxifen, a competitive partial agonist inhibitor of estradiol at the estrogen receptor (Figure 40–5), was the first selective
estrogen
receptor modulator (SERM) to be introduced. It is extensively used in the palliative treatment of breast cancer in postmenopausal
women
and is approved for chemoprevention of breast cancer in high-risk women .
· Studies of patients treated with tamoxifen as adjuvant therapy for early breast cancer have shown a 35% decrease in contralateral
breast cancer. Studies indicate that following unilateral excision of breast cancer, women receiving tamoxifen (an estrogen partial
agonist, see below) show a 35% decrease in contralateral breast cancer compared with controls. Many studies show an increased
risk of
endometrial carcinoma in patients taking estrogens alone.
· The risk seems to vary with the dose and duration of treatment: 15 times greater in patients taking large doses of estrogen for 5 or
more years, in contrast with two to four times greater in patients receiving lower doses for short periods.
However, as noted above, the concomitant use of a progestin prevents this increased risk and may in fact reduce the incidence of
endometrial cancer to less than that in the general population
Your Answer. c
Correct Answer. c
(41). Regarding ACE inhibitor which of the following is true
a. Inhibits conversion of angiotensinogen to angiotensin 1
b. T ½ of enalapril is more than lisinopril
c. Omission of prior diuretic dose decreases the risk of postural hypotension
d. It is effective only with left ventricular systolic dysfunction
Solution. Ans 41: (c) Omission of prior diuretic dose decreases the risk of postural hypotension
Ref– KDT- 450
Sol
Adverse effect of ACE inhibitors-
• Hypotension-Steep fall in blood pressure may occur following first dose of Ace inhibitor in Patients with elevated plasma rennin
activity
so the doses of ACE inhibitors should be initiated un low doses followed by gradual increment,if lower doses have been well
tolerated
Pharmacokinetics
• Enalapril half life 1.3 hrs, Enalaprilate half life 11 hrs
• Lisinopril half life 12 hrs
Your Answer. c
Correct Answer. c
Copyright © 2014 Delhi Academy of Medical Sciences, All Rights Reserved. 31/146
(42). All are true about ESBL excepta.
Sensitive to carbapenems
b. Ambler classification is based on molecular structure
c. classificatrion is based on 3rd g. cefalosporin sensivity
d. 3rd & 4th g. cefalosporin sensivity testing is must to confirm the ESBL
Solution. Ans 42: (d) 3rd & 4th g. cefalosporin sensivity testing is must to confirm the ESBL
Ref– ://en.wikipedia.org/wiki/Beta-lactamase
Sol
· In general, an isolate is suspected to be an ESBL producer when it shows in vitro susceptibility to the second-generation
cephalosporins (cefoxitin, cefotetan) but resistance to the third-generation cephalosporins and to aztreonam.
· Beta-lactamase inhibitors such as clavulanate, sulbactam, and tazobactam in vitro inhibit most ESBLs, but the clinical
effectiveness
of betalactam/ beta-lactamase inhibitor combinations cannot be relied on consistently for therapy.
· Cephamycins (cefoxitin and cefotetan) are not hydrolyzed by majority of ESBLs, but are hydrolyzed by associated AmpC-type
âlactamase.
Currently, carbapenems are, in general, regarded as the preferred agent for treatment of infections due to ESBL-producing
organisms.
· Carbapenems are resistant to ESBL-mediated hydrolysis and exhibit excellent in vitro activity against strains of Enterobacteriaceae
expressing ESBLs
Your Answer. c
Correct Answer. d
(43). Methotrexate is used in all except
a. Sickle cell anemia
b. Psoriasis
c. Rheumatoid arthritis
d. Ankylosing spondylitis
Solution. Ans 43: (a) Sickle cell anemia
Ref– Read the text below
Sol
Indications of methotrexate
· Although the most common methotrexate dosing regimen for the treatment of rheumatoid arthritis is 15–25 mg weekly, there is an
increased effect up to 30–35 mg weekly.
· Evidence supports its use in juvenile chronic arthritis, and it has been used in psoriasis, psoriatic arthritis, ankylosing spondylitis,
polymyositis, dermatomyositis, Wegener's granulomatosis, giant cell arteritis, systemic lupus erythematosus, and vasculitis.
Your Answer. a
Correct Answer. a
Copyright © 2014 Delhi Academy of Medical Sciences, All Rights Reserved. 32/146
(44). False about pioglitazone
a. It PPARã agonist
b. Is metabolized in liver
c. Not given in case of diastolic dysfunction
d. It acts on insulin gene and even in absence of insulin helps in metabolism of carbohydrate
Solution. Ans 44: (d) It acts on insulin gene and even in absence of insulin helps in metabolism of carbohydrate
Ref– Harrrison -2176,2177
Sol
· Thiazolidinediones reduce insulin resistance. These drugs bind to the PPAR-ã (peroxisome proliferator-activated receptor-ã)
nuclear
receptor. Agonists of this receptor regulate a large number of genes, promote adipocytedifferentiation, reduce hepatic fat
accumulation,
and appear to reduce insulinresistance indirectly by enhancing fatty acid storage and possibly by increasingadiponectin levels.
Circulating insulin levels decrease with use of thethiazolidinediones, indicating a reduction in insulin resistance.
· The US FDA recommends measurement of liver function tests prior to initiating therapy with a thiazolidinedione and at regular
intervals. Rosiglitazone raises LDL, HDL, and triglycerides slightly. Pioglitazone raisesHDL to a greater degree and LDL a lesser
degree
but lowers triglycerides.Peripheral edema and CHF may occur and is more common in individuals alsotreated with insulin. These
agents
are contraindicated in patients with liver disease or CHF (class III or IV). Recent metanalyses have suggested thatrosiglitazone is
associated with an increased risk of myocardial infarction.
· TheFDA has issued an alert that rare patients taking these agents may experience aworsening of diabetic macular edema. An
increased risk of fractures has beennoted in women taking these agents. Thiazolidinediones have been shown toinduce ovulation in
premenopausal women with PCOS.
Your Answer. d
Correct Answer. d
(45). Which of the following statement is true about bacteriophage
a. It is a bacterium
b. It helps in transformation
c. It imparts toxigenicity to bacteria
d. It transfers only chromosomal gene
Solution. Ans 45: (c) It imparts toxigenicity to bacteria
Ref– Read the text below
Sol
· Bacteriophages are viruses that infect bacteria.It helps in transduction and not transformation.
· Transformation is bacterial gene transfer through the agency of free DNA as shown by Griffith in the experiments of Pneumococci
with mice. Transformation plays no role in transfer of drug resistance.
· It imparts toxigenicity by lysogenic conversion e.g. toxin production by diphtheria bacillus due to presence of prophage beta.
· Penicillinase resistance in Staphylococcus is a classical example of plasmid mediated drug resistance transmitted by transduction
or
bacteriophage.
Your Answer. c
Correct Answer. c
Copyright © 2014 Delhi Academy of Medical Sciences, All Rights Reserved. 33/146
(46). Lambda phage which is true
a. It causes mad cow disease
b. Lysogenic to lytic conversion cant occur
c. Lysogenic inform incorporates in host DNA & remain dormant
d. Lytic phase incorporates in host DNA, proliferate & causes rupture of cell
Solution. Ans 46: (c) Lysogenic inform incorporates in host DNA & remain dormant
Ref– Ananthnarayan Microbiology,8th ed pg 456-458
Sol
· In Lysogenic pathway viral DNA becomes integrated into Bacterial chromosome where it replicates passively as the bacterial DNA
&
cell divides. This dormant genomically integrated virus is called prophage & cell that harbours it is called a Lysogen.
· In Lytic pathway the viral DNA replicates itself & directs synthesis of biral proteins.THE proliferating viruses induce lysis of cells.
Stages of lytic Cycle-
• Adsorption
• Penetration
• Synthesis of phage components
• Assembly
• Maturation
• Release of progeny phage particles
A prophage can be Induced by a DNA damaging agent such as ultraviolet radiation.The inducing agent throughs a switch,so that a
different set of genes is turned on.Viral DNa loops out of Chromosome & replicates & the virus proceeds along Lytic pathway i.e.
lysogenic to lytic canversion does occur
Your Answer. d
Correct Answer. c
(47). Best medium to grow anaerobic bacteria exclusively is :
a. Blood agar
b. Robertson’s cooked meat medium
c. Thioglycollate medium
d. Sabouraud Dextrose agar
Solution. Ans-47: (b) Robertson’s cooked meat medium
Ref: Ananthanarayan’s - 43
Sol :
• An anaerobic bacteria culture is a method used to grow anaerobes from a clinical specimen.
• Robertson’s cooked meat medium is probably the most widely used fluid medium for the culture of anaerobes.
• Obligate anaerobes are bacteria that can live only in the absence of oxygen. Obligate anaerobes are destroyed when exposed to
the
atmosphere for as briefly as 10 minutes.
Facultative anaerobes are those organisms that will grow with or without oxygen. Many anaerobes have characteristic microscopic
morphology. For example, Bacteroides spp are gram-negative rods that are pleomorphic (variable in size and shape) and exhibit
irregular
bipolar staining
Your Answer. b
Correct Answer. b
Copyright © 2014 Delhi Academy of Medical Sciences, All Rights Reserved. 34/146
(48). Which of the following is type III hypersensitivity reaction :
a. Prausnitz-kustner reaction
b. Contact dermatitis
c. Arthus reaction
d. Rh incompatibility
Solution. Ans-48: (c) Arthus reaction
Ref: Read the text below
Sol :
· In immunology, the Arthus reaction is a type of local type III hypersensitivity reaction. Type III hypersensitivity reactions are
immune complex-mediated, and involve the deposition of antigen/antibody complexes mainly in the vascular walls, serosa (pleura,
pericardium, synovium), and glomeruli.
Your Answer. c
Correct Answer. c
(49). Half life of IgG :
a. 5 days
b. 6 days
c. 8 days
d. 23 days
Solution. Ans-49: (d) 23 days
Ref: Ananthanaryan’s Microbiology, 8th ed.,p 98
Sol :
• IgG is the most abundant antibody in the blood, as well as the most common antibody produced in late primary and in secondary
immune response.
• IgG is a 150 kD monomer with a half-life of 23 days.
• It is also distributed in extracellular fluid and is present in milk, and maternal IgG is the only IgG that normally crosses the placenta.
• IgG binds to the surface of somatic and microbial cells, which allows those cells to be phagocytosed of killed by cytotoxic cells.
Your Answer. c
Correct Answer. d
(50). Venkataraman-Ramakrishan media is used for :
a. Streptococcus pneumonia
b. Staphylococcus aureus
c. Vibrio cholera
d. Clostridium tetani
Solution. Ans-50: (c) Vibrio cholera
Ref: Ananthanarayan’s --303
Sol :
Transport media for cholera vibrios are :
• Alternative transport media for vibrio cholera are Venkataraman-Ramakirshnan medium (V-rR fluid) or alkaline peptone water. VR
fluid should be prepared in 30 ml (1 oz) screw capped bottles (MacCartney bottles). It preserves vibrios for more than 6 weeks and
has
also proved to be very convenient medium for transportation as it can be kept at room temperature after collection of specimen.
• If none of the transport medium is available, collect watery stool from suspected cholera case by soaking blotting paper strip in the
stool, sealing the strip in a polythene bag or an airtight container where the strip does not dry.
• The moist blotting paper strip can then be transported to the laboratory. Blotting papers have been found to be superior to the filter
paper as the former can soak more moisture and retain the same over longer periods thus maintaining the viability of vibrios
absorbed on
it.
Your Answer. c
Correct Answer. c
Copyright © 2014 Delhi Academy of Medical Sciences, All Rights Reserved. 35/146
(51). Chinese letter arrangement is seen in :
a. M. tuberculosis
b. Corynobacterium diphtheria
c. Chlamydia trachomatis
d. M. leprae
Solution. Ans-51: (b) Corynebnacterium diphtheria
Ref: Ananthanarayan’s - 232
Sol :
• Nonpathogens that resemble corynebacterium diptheriae in appearance are termed “diptheroids.”
• The arrangement of the bacteria provide a clue to distinguish the pathogen corynebacterium diptheriae from the nopathogenic
diptheroids.
• C. diptheriae bacteria form 90 degree angles to one another, forming “X’x”, “Y’s”, or “Chinese letter”. In contrast, diptheroids are
often in parallel arrangements called “palisades”.
• The diagnostic significance of metachromatic granules arises from the fact that the causative agent of diphtheria.
Corynebacterium diptheriae, characteristically produces metachromatic granules.
Your Answer. b
Correct Answer. b
(52). Which of the following is seen in dark field microscopy :
a. Vibrio
b. Spirochaetes
c. Chlamydia
d. All of the above
Solution. Ans-52: (b) Spirochaetes
Ref: Read the text below
Sol :
· A clinical diagnosis of syphilis is confirmed by using darkfield microscopy to demonstrateTreponema pallidum in material from
suspected lesions or regional lymph nodes.
· A positivedarkfield result is an almost certain diagnosis of primary, secondary, or early congenital syphilis.
· For patients with early primary syphilis or for patients with syphilitic lesions and advanced acquired immunodeficiency syndrome
(AIDS), the darkfield examination may identify the etiologicagent of syphilis and help diagnose the disease even when antibodies to
T.
pallidum cannot be detected.
Your Answer. d
Correct Answer. b
(53). Swimming pool granuloma is caused by :
a. Mycobacterium chelone
b. Mycobacterium kansasii
c. Mycobacterium marinum
d. Mycobacterium ulcerans
Solution. Ans-53: (c) Mycobacterium marinum
Ref: Ananthanarayan’s Microbiology, 8th ed.,p. 362
Sol :
• Aquarium granuloma/Fishtank granuloma/swimming pool granuloma results when water containing an infectious organism,
Mycobacterium marinum, enters a break in the skin. A localized infection called a granuloma results after about 3 weeks.
• The person usually has a history of exposure to swimming pools , salt water aquarium, or ocean fish.
• The lesions appear as reddish bumps (papules) that slowly into purplish nodules.
Your Answer. c
Correct Answer. c
Copyright © 2014 Delhi Academy of Medical Sciences, All Rights Reserved. 36/146
(54). EBV (ebstein barr virus) causes all of the following except :
a. Glandular fever
b. Burkitt’s lymphoma
c. Pancreatic carcinoma
d. Nasopharyngeal carcinoma
Solution. Ans-54: (c) Pancreatic carcinoma
Ref: Ananthanarayan’s - 476
Sol :
• Epstein-Barr virus (EBV) causes infectious mononucleosis as primary disease. The virus infects more than 90% of the average
population and persists lifelong in peripheral B-lymphocytes. The virus is produced in the parotid and spread via the oral route.
• Serology suggests that the Epstein-Barr virus might be involved in the causation of two neoplastic diseases of humans : African
Burkitt’s Iymphoma and nasopharyngeal carcinoma.
• Whereas the development of the lymphoma has an even better linkage with chromosomal rearrangements, nasopharyngeal
carcinoma
shows a unique association with Epstein-Barr virus.
Your Answer. c
Correct Answer. c
(55). All are true about brucella except
a. B. abortus is capnophilic
b. Transmitted by aerosol can occur occasionally
c. Paesturisation destroys it
d. 2 Mercaptoethanol is used to detect IgA
Solution. Ans 55: (d) 2 Mercaptoethanol is used to detect IgA
Ref– Read the text below
Sol
· B. abortus is capnophilic i.e. requires 5-10% CO2 to grow Can be transmitted by aerosol occasionally Brucellosis may be
acquirewd
byingestion ,inhalation or mucosal or percutaneous exposure.Pasteurisation destroys it
· 2 Mercaptoethanol (a reducing agent) reduces IgM a pentamer into a monomer. With immunoelectrophoresis, 2-ME treatment
improvedidentification of IgM M-proteins because IgM monomers migrate more readilythrough the agarose than the IgM pentamer.
Your Answer. a
Correct Answer. d
(56). All are true about non typhoid salmonella except
a. Poultry is source
b. Can cause invasive disease in neonates
c. Blood culture is more sensitive than stool culture in gastroenteritis in adults
d. Resistance to FQ has emerged
Solution. Ans 56: (c) Blood culture is more sensitive than stool culture in gastroenteritis in adults
Ref– Ananthnarayan Microbiology 298-300
Sol
· Unlike S. Typhi and S. Paratyphi, whose only reservoir is humans, NTS can be acquired from multiple animal reservoirs.
Transmission is most commonly associated with animal food products, especially eggs, poultry, undercooked ground meat, and
dairy
products and fresh produce contaminated with animal waste.
· Known to cause invasive disease in neonates, elderly and immunocompromised individuals
· Resistance to nalidixic acid and fluoroquinolones also has begun to emerge and is most commonly associated with point mutations
in
the DNA gyrase genes gyrA and gyrB. Nalidixic acid resistance is a good predictor of reduced susceptibility to clinically useful
fluoroquinolones The diagnosis of NTS infection is based on the isolation of the organism from freshly passed stool or from blood or
another ordinarily sterile body fluid.
· All salmonellae isolated in clinical laboratories should be sent to local public health departments for serotyping. Blood cultures
should be done whenever a patient has prolonged or recurrent fever.
Your Answer. a
Correct Answer. c
Copyright © 2014 Delhi Academy of Medical Sciences, All Rights Reserved. 37/146
(57). Which of the following statements about vaccines is false
a. Thiomersal is preservative in DPT vaccine
b. Magnesium chloride is stabilizer used in OPV
c. Kanamycin is preservative in measles
d. Neomycin is preservative in BCG vaccine
Solution. Ans 57: (d) Neomycin is preservative in BCG vaccine
Ref– Read the text below
Sol
· In OPV the magnesium chloride stabilised vaccine will maintain adequate immunogenicity for 18 months when kept in a
refrigerator
at +2°C to +8°C, for six weeks at +25°C and for three days at +37°C.
· Neomycin is used as a preservative in measles,mumps,rubella,varicella & poliovirus but not in BCG.
· MEASLes- it is an attenuated live virus vaccine.each dose contain not less than 1000 CCID 50 of viral strain CAM 70& not more
than
36mcg residual kanamycin sulphate & 11mcg Residual Erythromycin lactobionate.
Your Answer. d
Correct Answer. d
(58). Undersection 304 A of IPC, for a proved case of medical negligence.the maximum punishment of imprisonment isa.
2 years
b. 5 years
c. 7 years
d. 10 years
Solution. Ans 58 : (a)2 years
Ref:Read the text below
Sol:
Section offence Punishment
Sec 299 IPC – Culpable homicide not amounting to murder
Sec 300 IPC – Defines Murder
Sec 302 IPC – Punishment for murder Death or imprisonment for life
Sec 303 IPC – Punishment for murder by life Convict (death)
Sec 304 IPC – Punishment for culpable homicide Imprisonment for life or 10 yrs
not amounting to murder -
Sec 304A IPC – Causing death by Negligence 2 years or fine or both
Sec 304B – Dowry death 7 years to imprisonment for life
Sec 305 IPC- Abetment of suicide of a child and insane person 10 years imprisonment
Sec 306 IPC – Abetment to suicide Imprisonment for 10 yrs
Sec 307 IPC – Attempt to murder Death/ Imprisonment for life/ Imprisonment for10 yrs
Sec 308 IPC – Attempt to commit culpable homicide Imprisonment for 3 yrs/ Imprisonment for 7 yrs
Sec 309 IPC – Attempt to commit suicide 1 year or fine or both
S.320 IPC- Defines Grievous injury
S.321IPC- Voluntarily causing hurt
S.322 IPC- Voluntarily causing Grievous hurt
S.323 IPC-Punishment for Voluntarily causing hurt
S.324 IPC- Voluntarily causing hurt by dangerous weapon
S.325 IPC-Punishment for Voluntarily causing Grievous hurt
S.326 IPC - Voluntarily causing Grievous hurt by dangerous weapons or means.
S.340 IPC-Wrongful Confinement
S.351 IPC-Assault
S.362 IPC-Abduction
Your Answer. c
Correct Answer. a
Copyright © 2014 Delhi Academy of Medical Sciences, All Rights Reserved. 38/146
(59). According to Human organ transplant act 1994 provision for punishment of imprisonment to erring doctor is -
a. Less than 1 year
b. Less than 2 year
c. 2 to 5 year
d. More than 5 year
Solution. Ans 59 : (c)2 to 5 year
Ref:Read the text below
Sol:
Sec 18.Punishment for removal of human organ without authority
(1) Any person who renders his services to or at any hospital and whom for purposes of transplantation, conducts, associates with,
or
helps in any manner in, the removal of any human organ without authority, shall be punishable with imprisonment for a term which
may
extend to five years and with fine which may extend to a ten thousand rupees.
(2) Where any person convicted under sub-section (1) is a registered medical practitioner, his name shall be reported by the
Appropriate
Authority to the respective state Medical Council for taking necessary action including the removal of his name from the register of
the
Council for a period of two years for the first offence and permanently for the subsequent offence.
So, punishment for erring doctor can be imprisoned from 2 to 5 years (maximum of 5 years) and fine of ten thousand rupees, while
the
name of the doctor can be removed from the medical register for a period of 2 years for first offence and permanently for
subsequent
offence
Your Answer. c
Correct Answer. c
(60). Flaying is seen in which type of laceration –
a. Split
b. Stretch
c. Avulsion
d. Tears
Solution. Ans 60 : (c)Avulsion
Ref:The Essentials of Forensic medicine-Dr.K.S.Narayan Reddy,23rd edition,page 152
Sol:
Laceration- lacerations are tears or spilts of skin,mucus membranes,muscle or internal organs produced by application of blunt force
to
broad area of body,which crushed or stretched tissues beyond the limits of their elasticity.They are also called tears or ruptures.
Typesa.
Split Lacerationsb.
Stretch Lacerations
c.Avulsion
d.Tears
e.Cut lacerations
The Shearing & grinding force by a weight,such as lorry wheel passing over alimb may produce separation of skin from the
underlying
tissues (avulsion) over a relatively large area is known as flaying – characteristically seen in avulsion laceration
Your Answer. Not Attempted
Correct Answer. c
Copyright © 2014 Delhi Academy of Medical Sciences, All Rights Reserved. 39/146
(61). Hostile witness is a person who conceals a part of the truth or wilfully give completely false information under oath in court of
law and is
punishable under which section of IPC
a. 141
b. 151
c. 171
d. 191
Solution. Ans 61 : (d)191
Ref:The Essentials of Forensic medicine-Dr.K.S.Narayan Reddy,23rd edition,page 11, 42 Indian penal codes
Sol:
Sec 191 IPC - Giving false evidence — Perjury
Sec 192 IPC- Fabricating false evidence.
Sec 193 IPC - Punishment for false evidence – Punishment for Perjury – 7 months imprisonment and fine.
Hostile witness is one who is supposed to have some interest or motive for concealing part of the truth,or for giving completely false
evidence.
A hostile witness is punished for Perjury.
Your Answer. c
Correct Answer. d
(62). A 45 year old chronic alcoholic male is brought to emergency department with an alleged history of assaulting his
neighbours.On
examination he is found to have irrelevant talk.He hadf not consumed alcohol since last 4 days.He is diagnosed as delirium
tremens.In
this condition the person isa.
Not responsible for his act under section 84 IPC
b. Partially responsible for his act
c. Criminally responsible for his act
d. Has Diminished responsibility for his act
Solution. Ans 62 : (a)Not responsible for his act under section 84 IPC
Ref:The Essentials of Forensic medicine-Dr.K.S.Narayan Reddy,23rd edition,page 400
Sol:
• McNaughten’s Rule – Sec 84 IPC - Act of a person of unsound mind
“Nothing is an offence which is done by a person ,who at the time of doingit,by reason of unsoundness of mind,is incapable of
knowing
the nature of the act or that he is doing what is either wrong or contrary to law”
• Sec 85 IPC - Act of a person incapable of judgment by reason of intoxication caused against his will.
• Sec 86 IPC - Offence requiring a particular intent or knowledge committed by one who is intoxicated.
A person who is intoxicated and commits a crime the criminal responsibility will lie on the fact whether he has consumed the
intoxicant
with the intention of committing the crime or not.
A person suffering from Delirium tremens is free from criminal responsibility.
Diminished responsibility– term used for borderline mental states – bordering upon but not amounting to insanity. Eg certain organic
depressions, obsessional states and some paranoid states.
Your Answer. a
Correct Answer. a
Copyright © 2014 Delhi Academy of Medical Sciences, All Rights Reserved. 40/146
(63). During autopsy most preferred organ for DNA extraction isa.
Liver
b. Spleen
c. Kidney
d. Brain
Solution. Ans 63 : (b)Spleen
Ref:Read the text below
Sol:
· The best samples are bone and teeth.
Since they are not amongst the options the other best answer is Spleen
Your Answer. a
Correct Answer. b
Copyright © 2014 Delhi Academy of Medical Sciences, All Rights Reserved. 41/146
(64). Which of the following factors is responsible for apoptosis?
a. Oleic acid
b. Isoprenoids
c. Myristic acid
d. Glucocorticoids
Solution. Ans-64: (d) Glucocorticoids
Ref:
Sol:
This is a highly controversial question. Glucocorticoids, oleic acid and isoprenoids all of them induce apoptosis. However, if you
want to
choose a single most appropriate answer, then the best option is glucocorticoids, as they have a proven therapeutic role in
treatment of
human malignancies due to induction of apoptosis of cancer cells; while the others have induced apoptosis in experimental studies.
· Although glucocorticoids (GCs) have been used for their immunosuppressive, anti-inflammatory and cytotoxic effects for many
years,
their precise mechanism of action has not been fully elucidated.
· Evidence indicates that GCs induce apoptosis in hematological cells, thus supporting their use as chemotherapeutic agents for
leukemias, lymphomas, and myeloma.
· Although much research has been focused on investigating the mechanism of action responsible for GC-mediated cell death, the
signaling pathways remain unclear.
· Two schools of thought have developed to account for GC-induced apoptosis. One supports the hypothesis that apoptosis is
achieved
via activation of death-inducing genes.
· The theory is that ligand-activated GRs directly bind to cis-acting sequences of DNA (GREs), which function as inducible enhancer
elements.
· This induction of transcription leads to increased expression of an apoptosis-inducing gene(s) that generates an apoptotic signal
and
initiates the cascade.
· However, to date no proapoptotic genes have been identified as targets of GC-mediated transcription.
· Current thinking is that GC-induced genes are responsible for the myriad of side effects associated with GC treatment, rather than
the cytotoxic and immunosuppressive actions.
· The second theory states that GCs induce apoptosis via repression of transcription factor activity, thereby inhibiting the
transcription of growth / survival genes.
· Additionally, there is some evidence that GC-induced apoptosis is merely a consequence of cell cycle arrest.
Option ‘1’: Oleic acid
Ref: Zhua Y, Schwarza S, Ahlemeyera B, Grzeschika S, Klumppb S and Krieglsteina J. Oleic acid causes apoptosis and
dephosphorylates
Bad. Neurochemistry International. 2005;46(2):127-135.
· There is increasing evidence showing the involvement of unsaturated free fatty acids in cell death pathways, particularly in the
context of apoptotic signalling. Previous in vitro study has demonstrated that oleic acid, a monounsaturated fatty acid, reduces
phosphorylation of proapoptotic Bad through activation of protein phosphatase type 2Cβ.
· As revealed by nuclear staining, oleic acid caused a concentration- and time-dependent damage with typical apoptotic features in
cortical and hippocampal cultures from embryonic and neonatal rats, respectively, as well as in human neuroblastoma SH-SY5Y
cells.
· In mixed hippocampal cultures, nearly all neurons were damaged at 24 h after the treatment, while damage of astrocytes was
detected 48 h after adding this fatty acid, suggesting that neurons were more vulnerable than astrocytes.
Option ‘2’: Isoprenoids
Ref: Mo H and Elson C. Apoptosis and cell-cycle arrest in human and murine tumor cells are initiated by isoprenoids. J Nutr.
1999;129(4):804-13.
· Diverse classes of phytochemicals initiate biological responses that effectively lower cancer risk.
· One class of phytochemicals, broadly defined as pure and mixed isoprenoids, encompasses an estimated 22,000 individual
components.
· A representative mixed isoprenoid, gamma-tocotrienol, suppresses the growth of murine B16(F10) melanoma cells, and with
greater
potency, the growth of human breast adenocarcinoma (MCF-7) and human leukemic (HL-60) cells.
· Beta-Ionone, a pure isoprenoid, suppresses the growth of B16 cells and with greater potency, the growth of MCF-7, HL-60 and
human colon adenocarcinoma (Caco-2) cells.
· Results obtained with diverse cell lines differing in ras and p53 status showed that the isoprenoid-mediated suppression of growth
is
independent of mutated ras and p53 functions.
· The isoprenoids initiated apoptosis and, concomitantly arrested cells in the G1 phase of the cell cycle.
· Beta-Ionone interfered with the posttranslational processing of lamin B, an activity essential to assembly of daughter nuclei.
· This interference, renders neosynthesized DNA available to the endonuclease activities leading to apoptotic cell death.
The additive and potentially synergistic actions of these isoprenoids in the suppression of tumor cell proliferation and initiation of
apoptosis coupled with the mass action of the diverse isoprenoid constituents of plant products may explain, in part, the impact of
fruit,
vegetable and grain consumption on cancer risk
Your Answer. b
Correct Answer. d
Copyright © 2014 Delhi Academy of Medical Sciences, All Rights Reserved. 42/146
(65). A 36-year old female underwent cholecystectomy for acute abdomen. The wound was closed with sutures. Which of the
following steps is
responsible for increasing the tensile strength of the wound?
a. Macrophage infiltration
b. Replacement of type III collagen
c. Cross-linking of tropocollagen
d. Granulation tissue formation
Solution. Ans-65: (c) Cross-linking of tropocollagen
Ref: Robbins and Cotran’s ‘Pathological Basis of Disease’; 7/e, pg 113-114
Sol:
WOUND STRENGTH –
· When sutures are removed, usually at the end of the first week, wound strength is approximately 10% that of unwounded skin, but
increases rapidly over the next 4 weeks.
· This rate of incease then slows at approximately the third month after the original incision, and reaches a plateau at about 70% to
80% of tensile strength of unwounded skin, a condition that may persist for life.
· The recovery of tensile strength results from the excess of collagen synthesis over collagen degradation over during the first two
months of healing; and at later times, from structural modifications of collagen fibres (cross-linking, increased fibre size) after excess
collagen synthesis.
(For details about collagen structure and cross-linking, please refer Q. 16.)
FACTORS THAT RETARD WOUND HEALING –
Local factors Systemic factors
· Inadequate blood supply – (Arteriosclerosis, Varicose veins)
· Denervation
· Local infection
· Foreign body hematoma (Unnecessary sutures; fragments of
glass, steel or bone)
· Mechanical stress (Early motion of wounds)
· Necrotic tissue
· Protection (Dressings)
· Faulty or improper surgical techniques
· Type of tissue (poorly vascularized areas such as foot)
· Age
· Anemia
· Drugs (Steroids, cytotoxic medications, intensive antibiotic
therapy)
· Genetic disorders (Osteogenesis imperfacta, Ehlers-Danlos
syndrome, Marfan syndrome)
· Hormones (Glucocorticoids)
· Diabetes
· Malignant disease
· Malnutrition
· Obesity
· Systemic infections
· Temperature
· Trauma
· Hypovolemia
· Hypoxia
· Uremia
· Vitamin deficiencies (Vit. C)
· Trace metal deficiencies (Zinc, copper)
Your Answer. b
Correct Answer. c
Copyright © 2014 Delhi Academy of Medical Sciences, All Rights Reserved. 43/146
(66). Epigenetics is the study of genetic changes without involving DNA sequences. All of the following molecular techniques are
used in
epigenetic research except –
a. HPLC
b. Bisulfite sequencing
c. Methylation sensitive – RE analysis
d. ChIP-on-chip
Solution. Ans-66: (a) HPLC
Ref: http://en.wikipedia.org/wiki/Epigenetics
Sol:
EPIGENETICS –
· Epigenetics is the study of cellular and physiological traits that are heritable by daughter cells and not caused by changes in the
DNA sequence.
· Robin Holliday defined epigenetics as "the study of the mechanisms of temporal and spatial control of gene activity during the
development of complex organisms".
· Thus epigenetic can be used to describe anything other than DNA sequence that influences the development of an organism.
· Unlike simple genetics based on changes to the DNA sequence (the genotype), the changes in gene expression or cellular
phenotype
of epigenetics have other causes, thus use of the term epi- (Greek: επί- over, outside of, around) -genetics.
· It describes the study of stable, long-term alterations in the transcriptional potential of a cell.
· These alterations may or may not be heritable, although the use of the term epigenetic to describe processes that are not heritable
is
controversial.
MOLECULAR BASIS FOR EPIGENETICS –
· Epigenetic changes can modify the activation of certain genes, but not the sequence of DNA.
· Additionally, the chromatin proteins associated with DNA may be activated or silenced.
· This is why the differentiated cells in a multi-cellular organism express only the genes that are necessary for their own activity.
· Epigenetic changes are preserved when cells divide.
· Most epigenetic changes only occur within the course of one individual organism's lifetime.
· Specific epigenetic processes include –
o Paramutation
o Bookmarking
o Imprinting
o Gene silencing
o X chromosome inactivation
o Position effect
o Reprogramming
o Transvection
o Maternal effects
o The progress of carcinogenesis
o Many effects of teratogens
o Regulation of histone modifications and heterochromatin and
o Technical limitations affecting parthenogenesis and cloning
· DNA damage can also cause epigenetic changes.
· DNA damages are very frequent, occurring on average about 10,000 times a day per cell of the human body.
· These damages are largely repaired, but at the site of a DNA repair, epigenetic changes can remain.
· In particular, a double strand break in DNA can initiate unprogrammed epigenetic gene silencing both by causing DNA methylation
as well as by promoting silencing types of histone modifications (chromatin remodeling).
· In addition, the enzyme Parp1 (poly(ADP)-ribose polymerase) and its product poly(ADP)-ribose (PAR) accumulate at sites of DNA
damage as part of a repair process.
· This accumulation, in turn, directs recruitment and activation of the chromatin remodeling protein ALC1 that can cause nucleosome
remodeling.
· Nucleosome remodeling has been found to cause, for instance, epigenetic silencing of DNA repair gene MLH1.
· DNA damaging chemicals, such as benzene, hydroquinone, styrene, carbon tetrachloride and trichloroethylene, cause
considerable
hypomethylation of DNA, some through the activation of oxidative stress pathways.
· Foods are known to alter the epigenetics of rats on different diets.
· Some food components epigenetically increase the levels of DNA repair enzymes such as MGMT and MLH1 and p53.
· Other food components can reduce DNA damage, such as soy isoflavones and bilberry anthocyanins.
MOLECULAR STUDIES TO DETECT EPIGENETIC CHANGES –
· Epigenetic research uses a wide range of molecular biologic techniques to further our understanding of epigenetic phenomena,
including –
o Chromatin immunoprecipitation (together with its large-scale variants ChIP-on-chip and ChIP-Seq)
o Fluorescent in situ hybridization (FISH)
o Methylation-sensitive restriction enzyme analysis
o DNA adenine methyltransferase identification (DamID) and
o Bisulfite sequencing.
· Furthermore, the use of bioinformatic methods is playing an increasing role (computational epigenetics).
Your Answer. Not Attempted
Correct Answer. a
Copyright © 2014 Delhi Academy of Medical Sciences, All Rights Reserved. 44/146
Copyright © 2014 Delhi Academy of Medical Sciences, All Rights Reserved. 45/146
(67). A patient presented with wheeze, pallor and fatigue. He was administered penicillin for an infection 48 hours earlier. There is
no history
of penicillin allergy. Laboratory examination revealed antibodies to penicillin and evidence of hemolytic anemia. He has most likely
developed whch type of hypersensitivity reaction?
Copyright © 2014 Delhi Academy of Medical Sciences, All Rights Reserved. 46/146
a. Type I
b. Type II
Copyright © 2014 Delhi Academy of Medical Sciences, All Rights Reserved. 47/146
c. Type III
Copyright © 2014 Delhi Academy of Medical Sciences, All Rights Reserved. 48/146
d. Type IV
Copyright © 2014 Delhi Academy of Medical Sciences, All Rights Reserved. 49/146
Solution. Ans-67: (b) Type II
Ref: Robbins and Cotran’s ‘Pathological Basis of Disease’; 7/e, pg 210, 212, 215
Sol:
Clinically, antibody-mediated cell destruction (Type II hypersensitivity reactions) occur in the following situations –
1. Transfusion reactions – Cells from an incompatible donor react with and are opsonized by preformed antibodies in the host.
2. Erythroblastosis fetalis – There is an antigenic differences between the mother and the fetus, and the antibodies (of the IgG class)
from the mother cross the placenta and cause destruction of fetal red cells.
3. Autoimmune hemolytic anemia and thrombocytopenia – Individuals produce antibodies to their own blood cells, which are then
destroyed.
4. Certain drug reactions – Antibodies are produced that react with the drug, which may be attached to the surface of erythrocytes
and
other cells.
EXAMPLES OF ANTIBODY-MEDIATED DISEASES (TYPE II HYPERSENSITIVITY) –
Disease Target antigen Mechanism Clinico-pathologic
manifestations
Autoimmune hemolytic anemia
Erythrocyte membrane proteins
(Rh blood group antigens, I
antigen)
Opsonization and phagocytosis of
erythrocytes Hemolysis, Anemia
Autoimmune thrombocytopenic
purpura
Platelet membrane proteins
(GpIIb/IIIa, Integrins)
Opsonization and phagocytosis of
platelets Bleeding
Pemphigus vulgaris
Proteins in intercellular junctions
of epidermal cells (epidermal
cadherin)
Antibody-mediated activation of
proteases, Disruption of
intercellular adhesions
Skin vesicles (bullae)
Vasculitis caused by ANCA
Neutrophil granule proteins,
presumably released from
activated neutrophils
Neutrophil degranulation and
inflammation Vasculitis
Goodpasture syndrome
Non-collagenous protein in
basement membranes of renal
glomeruli and lung alveoli
Complement- and Fc receptormediated
inflammation
Nephritis, Pulmonary
hemorrhages
Acute rheumatic fever
Streptococcal cell wall antigen;
antibody cross reacts with
myocardial antigen
Inflammation, macrophage
activation Myocarditis, Arthritis
Myasthenia gravis Acetylcholine receptor
Antibody inhibits acetylcholine
binding, down-modulates
receptors
Muscle weakness,
Paralysis
Graves’ disease
(hyperthyroidism) TSH receptor Antibody-mediated stimulation of
TSH receptors Hyperthyroidism
Insulin-resistant diabetes Insulin receptor Antibody inhibits binding of
insulin Hyperglycemia, Ketoacidosis
Pernicious anemia Intrinsic factor of gastric parietal
cells
Neutralization of intrinsic factor,
decreased absorption of vitamin
B12
Abnormal erythropoiesis,
Anemia
EXAMPLES OF IMMUNE-COMPLEX MEDIATED DISEASES (TYPE III HYPERSENSITIVITY) –
Disease Antigen involved Clinico-pathologic manifestations
Systemic lupus erythematosus DNA, nucleoproteins, others Nephritis, Arthritis, Vasculitis
Polyarteritis nodosa Hepatitis B virus surface antigen (in some cases) Vasculitis
Post-streptococcal glomerulonephritis Streptococcal cell wall antigen(s); may be ‘planted’
in glomerular basement membrane Nephritis
Acute glomerulonephritis Bacterial antigens (Treponema); Parasitic antigens
(Malaria, Schistosomes); Tumour antigens Nephritis
Reactive arthritis Bacterial antigens (Yersinia) Acute arthritis
Arthus reaction Various foreign proteins Cutaneous vasculitis
Serum sickness Various proteins; e.g. foreign serum (antithymocyte
globulin) Arthritis, Vasculitis, Nephritis
EXAMPLES OF T-CELL MEDIATED (TYPE IV) HYPERSENSITIVITY –
Disease Specificity of Pathogenic T cells Clinico-pathologic manifestations
Type 1 diabetes mellitus Antigens of pancreatic islet β cells (insulin,
glutamic acid decarboxylase, others)
Insulitis (chronic inflammation in islets),
Destruction of β cells, Diabetes
Multiple sclerosis
Protein antigens in central nervous system
myelin (myelin basic protein, proteolipid
protein)
Demyelination in CNS with perivascular
inflammation; Paralysis; Ocular lesions
Rheumatoid arthritis Unknown antigen in joint synovium (Type II
collagen?)
Chronic arthritis with inflammation,
destruction of articular cartilage and bone
Peripheral neuropathy (Guillian-Barre
syndrome?) Protein antigens of peripheral nerve myelin Neuritis; Paralysis
Copyright © 2014 Delhi Academy of Medical Sciences, All Rights Reserved. 50/146
Your Answer. b
Correct Answer. b
(68). The role played by major histocompatibility complex proteins (MHC-1 and MHC-2) is to –
a. Transduce the signals to T cells following antigen recognition
b. Mediate immunoglobulin class-switching
c. Present antigens for recognition by T cell antigen rceeptors
d. Enhance the secretion of cytokines
Solution. Ans-68: (c) Present antigens for recognition by T cell antigen rceeptors
Ref: Robbins & Cotran’s ‘Pathologic Basis of Disease’; 8/e, Chap
Sol:
· Major histocompatibility complex (MHC) molecules were discovered as products of genes that evoke rejection of transplanted
organs, and their name derives from the recognition that they are responsible for tissue compatibility between individuals.
· The physiologic function of MHC molecules is to display peptide fragments of proteins for recognition by antigen-specific T cells.
· In humans the genes encoding the major histocompatibility molecules are clustered on a small segment of chromosome 6, the
major
histocompatibility complex, or the human leukocyte antigen (HLA) complex.
MHC CLASS I MHC CLASS II
All nucleated cells and platelets Present on macrophages, B cells, Dendritic cells (on APCs which
respond to T helper cells)
HLA A/B/C genes
Encoded in a region called HLA D (DP/DQ/DR)
Heterodimer α and β2 (not within MHC)
α àα1 , α2 ,α3 (peptide cleft between 1 and 2)
Heterodimer of α ( α1 and α2) and β ( β1 and β2 ) (peptide cleft
between both 1s)
display peptides derived from proteins (viral antigens) located in
cytoplasm and produced in cell
recognized by CD8 +ve T cells
Present antigens which are internalized into vesicles and are
typically derived from extracellular microbes and soluble proteins
Non polymorphic α3 is the binding site for CD8 Class II β2 domain binds CD4
Your Answer. c
Correct Answer. c
Copyright © 2014 Delhi Academy of Medical Sciences, All Rights Reserved. 51/146
(69). Cancers are usually associated with –
a. Mutations in intronic genes
b. Hypomethylation of oncogenes
c. Methylation of tumour suppressor genes
d. Loss of heterozygosity
Solution. Ans-69: (c) Methylation of tumour suppressor genes
Ref: Harrison’s ‘Principles of Internal Medicine’; 18/e, Chap 83
Sol:
EPIGENETIC INFLUENCES ON CANCER GENE TRANSCRIPTION –
· Chromatin structure regulates the hierarchical order of sequential gene transcription that governs differentiation and tissue
homeostasis.
· Disruption of chromatin remodeling leads to aberrant gene expression and can induce proliferation of undifferentiated cells.
· Epigenetics is defined as changes that alter the pattern of gene expression that persist across at least one cell division but are not
caused by changes in the DNA code.
· Epigenetic changes include alterations of chromatin structure mediated by methylation of cytosine residues in CpG dinucleotides,
modification of histones by acetylation or methylation, or changes in higher-order chromosome structure.
· The transcriptional regulatory regions of active genes often contain a high frequency of CpG dinucleotides (referred to as CpG
islands), which are normally unmethylated.
· Expression of these genes is controlled by transient association with repressor or activator proteins that regulate transcriptional
activation.
· Hypermethylation of promoter regions is a common mechanism by which tumor-suppressor loci are epigenetically silenced in
cancer
cells.
· Thus one allele may be inactivated by mutation or deletion (as occurs in loss of heterozygosity), while expression of the other allele
is
epigenetically silenced.
· The mechanisms that target tumor-suppressor genes for this form of gene silencing are unknown.
Your Answer. d
Correct Answer. c
Copyright © 2014 Delhi Academy of Medical Sciences, All Rights Reserved. 52/146
(70). All of the following pairs of antigens and tumours are correctly matched except –
Copyright © 2014 Delhi Academy of Medical Sciences, All Rights Reserved. 53/146
a. Vimentin in sarcoma
b. Desmin in carcinoma
Copyright © 2014 Delhi Academy of Medical Sciences, All Rights Reserved. 54/146
c. S100 protein in melanoma
Copyright © 2014 Delhi Academy of Medical Sciences, All Rights Reserved. 55/146
d. Leukocyte common antigen in lymphoma
Copyright © 2014 Delhi Academy of Medical Sciences, All Rights Reserved. 56/146
Solution. Ans-70: (b) Desmin in carcinoma
Ref: http://en.wikipedia.org/wiki/Desmin
Sol:
· Desmin is a type III intermediate filament found near the Z line in sarcomeres.
· It was first described in 1976, first purified in 1977, the gene was cloned in 1989 and the first knockout mouse was created in 1996.
· Desmin in humans is encoded by the DES gene.
· Desmin is only expressed in vertebrates, however homologous proteins are found in many organisms.
· It is a 52kD protein that is a subunit of intermediate filaments in skeletal muscle tissue, smooth muscle tissue, and cardiac muscle
tissue.
· The function of desmin has been deduced through studies in knockout mice, but the underlying mechanism of its action is not
known.
· Desmin forms a scaffold around the Z-disk of the sarcomere and connects the Z-disk to the subsarcolemmal cytoskeleton (the
cytoplasmic part of the muscle cell plasma membrane).
· It links the myofibrils laterally by connecting the Z-disks.
· Through its connection to the sarcomere, Desmin connects the contractile apparatus to the cell nucleus, mitochondria, and postsynaptic
areas of motor endplates.
· These connections maintain the structural and mechanical integrity of the cell during contraction while also helping in force
transmission and longitudinal load bearing.
· There is some evidence that desmin may also connect the sarcomere to the extracellular matrix (ECM) through desmosomes which
could be important in signalling between the ECM and the sarcomere which could regulate muscle contraction and movement.
· Finally, desmin may be important in mitochondria function. When desmin is not functioning properly there is improper mitochondrial
distribution, number, morphology and function.
· Since desmin links the mitochondria to the sarcomere it may transmit information about contractions and energy need and through
this regulate the aerobic respiration rate of the muscle cell.
CLINICAL RELEVANCE –
· Desmin-related myopathy (DRM or Desminopathy) is a subgroup of the myofibrillar myopathy diseases and is the result of a mutation
in the gene that codes for desmin which prevents it from forming protein filaments, instead forming aggregates of desmin and other
proteins throughout the cell.
· Recently, mutations were identified in patients suffered by an arrhythmogenic right ventricular cardiomyopathy (ARVC).
· Some of these DES mutations like p.N116S or p.E114del cause an aggregation of desmin within the cytoplasm.
· A mutation p.A120D was discovered in family where several members had a sudden cardiac death.
· It is also associated with Sarcoma botryoides (rhabdomyosarcoma variant) - a vaginal malignancy composed of spindle-shaped cells
that affects girls < 4 years of age.
Option ‘1’: Vimentin in sarcoma
Ref: http://en.wikipedia.org/wiki/Vimentin
· Vimentin is a type III intermediate filament (IF) protein that is expressed in mesenchymal cells.
· IF proteins are found in all metazoan cells as well as bacteria.
· IF, along with tubulin-based microtubules and actin-based microfilaments, comprise the cytoskeleton.
· All IF proteins are expressed in a highly developmentally-regulated fashion.
· Vimentin is the major cytoskeletal component of mesenchymal cells.
· Because of this, vimentin is often used as a marker of mesenchymally-derived cells or cells undergoing an epithelial-to-mesenchymal
transition (EMT) during both normal development and metastatic progression.
· Vimentin is encoded in humans by the VIM gene.
· Vimentin plays a significant role in supporting and anchoring the position of the organelles in the cytosol. Vimentin is attached to the
nucleus, endoplasmic reticulum, and mitochondria, either laterally or terminally.
CLINICAL RELEVANCE –
· Vimentin has been used as a sarcoma tumor marker to identify mesenchyme.
· Methylation of the vimentin gene has been established as a biomarker of colon cancer and this is being utilized in the development of
fecal tests for colon cancer.
· Statistically significant levels of vimentin gene methylation have also been observed in certain upper gastrointestinal pathologies
such as Barrett's esophagus, esophageal adenocarcinoma, and intestinal type gastric cancer.
· High levels of DNA methylation in the promotor region have also been associated with markedly decreased survival in hormone
positive breast cancers.
Option ‘3’: S 100 protein in melanoma
Ref: http://en.wikipedia.org/wiki/S-100_protein
· The S-100 proteins are a family of low-molecular-weight proteins found in vertebrates and characterized by two calcium-binding
sites that have helix-loop-helix ("EF-hand type") conformation.
· There are at least 21 different S-100 proteins.
· Their name is derived from the fact that these proteins are soluble in 100%, i.e. saturated ammonium sulfate at neutral pH.
· They are encoded by a family of genes whose symbols use the S100 prefix, for example, S100A1, S100A2, S100A3.
· S-100 is normally present in cells derived from the neural crest (Schwann cells, and melanocytes), chondrocytes, adipocytes,
myoepithelial cells, macrophages, Langerhans cells, dendritic cells, and keratinocytes.
· It may be present in some breast epithelial cells.
· S100 proteins have been implicated in a variety of intracellular and extracellular functions.
· S100 proteins are involved in regulation of protein phosphorylation, transcription factors, Ca2+ homeostasis, the dynamics of
cytoskeleton constituents, enzyme activities, cell growth and differentiation, and the inflammatory response.
· S100A7 (psoriasin) and S100A15 have been found to act as cytokines in inflammation, particularly in autoimmune skin conditions
such as psoriasis.
CLINICAL RELEVANCE –
· Several members of the S-100 protein family are useful as markers for certain tumors and epidermal differentiation.
· It can be found in melanomas, 100% of schwannomas, 100% of neurofibromas (weaker than schwannomas), 50% of malignant
peripheral nerve sheath tumors (may be weak and/or focal), paraganglioma stromal cells, histiocytoma and clear cell sarcomas.
· Further, S100 proteins are markers for inflammatory diseases and can mediate inflammation and act as antimicrobials.
· S100 proteins have been used in the lab as cell markers for anatomic pathology.
Option ‘4’: Leukocyte common antigen in lymphoma
Ref: http://en.wikipedia.org/wiki/PTPRC
· Protein tyrosine phosphatase receptor type C (PTPRC) is also known as CD45 antigen, which was originally called leukocyte common
antigen (LCA).
· It is an enzyme that, in humans, is encoded by the PTPRC gene.
· The protein encoded by this gene is a member of the protein tyrosine phosphatase (PTP) family.
· PTPs are known to be signaling molecules that regulate a variety of cellular processes including cell growth, differentiation, mitotic
cycle, and oncogenic transformation.
· PTPRC is a type I transmembrane protein that is in various forms present on all differentiated hematopoietic cells, except
erythrocytes and plasma cells, that assists in the activation of those cells (a form of co-stimulation).
CLINICAL RELEVANCE –
· PTPRC is expressed in lymphomas, B-cell chronic lymphocytic leukemia, hairy cell leukemia, and acute nonlymphocytic leukemia.
· A monoclonal antibody to CD45 is used in routine immunohistochemistry to differentiate between histological sections from
lymphomas and carcinomas.
· Stem cells of several varieties, including mesenchymal stem cells and hæmatopoietic stem cells are found in the bone marrow.
· CD45 marker is used to distinguish the two stem cell types, as CD45 is found on all leukocytes.
Copyright © 2014 Delhi Academy of Medical Sciences, All Rights Reserved. 57/146
Your Answer. d
Correct Answer. b
(71). A 16-year-old boy fractured his leg while running track at school. X-ray studies revealed an abnormality at the lesion site
indicating that
this was a pathological fracture. Abiopsy of the area was taken and a photomicrograph of the tissue is shown in Fig. The most likely
diagnosis is which of the following?
a. Chondroblastoma
b. Chondrosarcoma
c. Osteoid osteoma
d. Osteosarcoma
Solution. Ans-71: (d) Osteosarcoma
Ref:Read the text below
Sol:
· This section in Fig shows many large, hyperchromatic, pleomorphic cells that are producing osteoid, typical of an osteosarcoma.
· A chondroblastoma would display sheets of primitive chondroblasts within a cartilage matrix that is irregularly calcified.
· Chondrosarcoma most typically occur in the age range of 40–60 with the most common locations being the pelvic girdle, ribs,
shoulder girdle, and to a lesser extent the long bones, vertebrae, and sternum.
· Microscopically, the lower grade tumors will show chondroid differentiation whereas a grade 3 tumor may appear as a spindle cell
tumor with little chondroid differentiation.
· Osteoid osteoma and osteoblastoma have a very similar microscopic appearance consisting of a random pattern of woven bone
with
many osteoblasts in evidence within a stroma of granulation-like tissue.
Osteoid osteomas are small, very painful tumors that are found in teenagers and young adults and respond very well to aspirin. It
usually
occurs in the long bones of the leg
Your Answer. d
Correct Answer. d
(72). Edema is caused by
a. Increased hydrostatic pressure
b. Decreasd capillary pressure
c. Decreased venous pressure
d. Increased plasma proteins
Solution. Ans-72: (a) Increased hydrostatic pressure
Ref.: Read the text below
Sol :
Edema due to hydrostatic pressure Edema due to ¯plasma oncotic pressure
Impaired venous return Arteriolar dilation Hypoproteinemia
§ CCF
§ Constrictive pericarditis
§ Ascites
§ Venous obstruction
§ Heat
§ Neurohumoral dysregulation
§ Protein losing glomerulopathies
§ Liver cirrhosis
§ Malnutrition
§ Protein losing gastroenteropathy
Your Answer. a
Correct Answer. a
Copyright © 2014 Delhi Academy of Medical Sciences, All Rights Reserved. 58/146
Copyright © 2014 Delhi Academy of Medical Sciences, All Rights Reserved. 59/146
(73). A 65-year old male presented with chronic cough and hemoptysis. Investigations revealed a greyish white tumour located
at the central bronchus casing distal bronchiectasis and recurrent pneumonia. The biopsy of the tumour is most likely going
to show –

Copyright © 2014 Delhi Academy of Medical Sciences, All Rights Reserved. 60/146
a. Spindle cells with abundant stromal matrix
b. All three germ layers

Copyright © 2014 Delhi Academy of Medical Sciences, All Rights Reserved. 61/146
c. Abundant osteoid matrix formation

Copyright © 2014 Delhi Academy of Medical Sciences, All Rights Reserved. 62/146
d. Small round cells and hyperchromatic nuclei with nuclear moulding

Copyright © 2014 Delhi Academy of Medical Sciences, All Rights Reserved. 63/146
Solution. Ans-73: (d) Small round cells and hyperchromatic nuclei with nuclear moulding
Ref: Robbins and Cotran’s ‘Pathological Basis of Disease’; 9/e
Sol:
LUNG CANCER
· It is the most commonly diagnosed malignancy worldwide.
CLASSIFICATION
1. Squamous cell carcinoma (Incidence: 20%)
· It is highly associated with exposure to tobacco smoke.
· It harbors diverse genetic aberrations, many of which are chromosome deletions involving tumor suppressor loci.
· These losses, especially those involving 3p, 9p (site of the CDKN2A gene), and 17p (site of the TP53 gene) are early
events in tumor evolution, being detected at an appreciable frequency in the histologically normal respiratory mucosal cells
of smokers.
· It shows highest frequency of TP53 mutations of all histologic types of lung carcinoma.
· p53 protein overexpression (as seen by immunohistochemical staining), a marker of TP53 mutations, is also an early
event, being reported in 10% to 50% of squamous dysplasias and 60% to 90% of squamous cell carcinoma in situ.
· Loss of expression of the retinoblastoma (RB) tumor suppressor is identified by immunohistochemistry in 15% of
squamous cell carcinomas.
· The cyclin-dependent kinase inhibitor gene CDKN2A is inactivated and its protein product, p16, is lost in 65% of
tumors.
· It has recently been recognized that many squamous cell carcinomas have amplification of FGFR1, a gene encoding the
fibroblast growth factor receptor tyrosine kinase.
2. Small cell carcinomas (Incidence: 14%)
· It has the strongest association with smoking.
· Despite its divergent histologic features, it shares many molecular features with squamous cell carcinoma.
· This includes frequent loss-of-function aberrations involving TP53 (75% to 90% of tumors), RB (close to 100% of
tumors), and chromosome 3p deletions.
· Amplification of genes of the MYC family is also common
3. Adenocarcinomas (Incidence: 38%)
· The incidence of adenocarcinoma has increased significantly in the last 2 decades.
· Adenocarcinoma is now the most common form of lung cancer in women and, in many studies, men as well.
· It is marked by oncogenic gain-of-function mutations involving components of growth factor receptor signaling
pathways.
· These include gain-of-function mutations in multiple genes encoding receptor tyrosine kinases, including EGFR, ALK,
ROS, MET, and RET, which are all also mutated in other forms of cancer.
· Tumors without tyrosine kinase gene mutations often have mutations in the KRAS gene
4. Large cell carcinoma (Incidence: 3%)
· Large cell carcinoma probably represents squamous cell carcinomas and adenocarcinomas that are so undifferentiated
that they can be no longer recognized on microscopy.
· Ulrastructurally, however, minimal glandular or aquamous differentiation is common.
ETIOLOGICAL FACTORS
· Tobacco smoke
· Asbestos
· Arsenic
· Chromium
· Uranium
· Nickel
· Vinyl chloride
· Mustard gas.
· High-dose ionizing radiation is carcinogenic
· Air pollution (Radon gas)
LUNG CANCER IN NON-SMOKERS –
· The WHO estimates that 25% of lung cancer worldwide occurs in never smokers.
· This percentage is probably closer to 10% to 15% in Western countries.
· These cancers occur more commonly in women and most are adenocarcinomas.
· Cancers in nonsmokers are more likely to have EGFR mutations, and almost never have KRAS mutations.
· TP53 mutations are not uncommon, but occur less frequently than in smoking related cancers.
PRECURSOR (PREINVASIVE) LESIONS –
Four types of morphologic precursor epithelial lesions are recognized:
· Squamous dysplasia and carcinoma in situ,
· Atypical adenomatous hyperplasia,
· Adenocarcinoma in situ, and
· Diffuse idiopathic pulmonary neuroendocrine cell hyperplasia
HISTOLOGIC CLASSIFICATION –
Squamous cell carcinoma
· Papillary carcinoma
· Clear cell carcinoma
· Small cell carcinoma
· Basaloid carcinoma
Small cell carcinoma · Combined small cell carcinoma
Adenocarcinoma
· Minimally invasive adenocarcinoma – Non-mucinous, Mucinous
· Lepidic carcinoma
· Acinar carcinoma
· Papillary carcinoma
· Solid carcinoma
· Mucinous carcinoma
Large cell carcinoma · Large cell neuroendocrine carcinoma
Adenosquamous carcinoma
Carcinoma with pleomorphic, sarcomatoid or sarcomatous elements
Carcinoid tumour · Typical carcinoid tumour
· Atypical carcinoid tumour
Carcinomas of the salivary gland type
MORPHOLOGICAL AND HISTOLOGICAL FEATURES
1. Adenocarcinoma
· It is an invasive malignant epithelial tumor with glandular differentiation or mucin production by the tumor cells.
· Adenocarcinomas grow in various patterns, including acinar, lepidic,
papillary, micropapillary, and solid with mucin formation.
· Compared with squamous cell cancers, the lesions are usually more peripherally located and tend to be smaller.
· They vary histologically from well-differentiated tumors with obvious glandular elements to papillary lesions
resembling other papillary carcinomas to solid masses with only occasional mucin-producing glands and cells.
· The majority express thyroid transcription factor-
1; first identified in the thyroid, this factor is required for normal lung development.
· At the periphery of the tumor there is often a lepidic pattern of spread, in which the tumor cells “crawl” along
normal-appearing alveolar septae.
·
Tumors (≤3 cm) with a small invasive component (≤5 mm) associated with scarring and a peripheral lepidic growth
pattern are called microinvasive adenocarcinoma.
· These have a far better outcome than invasive carcinomas of the same size.
· Mucinous adenocarcinomas tend to spread aerogenously, forming satellite tumors.
·
These may present as a solitary nodule or as multiple nodules, or an entire lobe may be consolidated by tumor, rese
mbling lobar pneumonia and thus are less likely to be cured by surgery.
2. Squamous cell carcinoma
· It is often antedated by squamous
metaplasia or dysplasia in the bronchial epithelium, which then transforms to carcinoma in
situ, a phase that may last for several years.
· The tumor may then follow a variety of paths.
· It may grow exophytically into the bronchial lumen, producing an intraluminal mass.
· With further enlargement the bronchus becomes obstructed, leading to distal atelectasis and infection.
· The tumor may also
penetrate the wall of the bronchus and infiltrate along the peribronchial tissue into the adjacent carina or mediastinu
m.
· In other instances, the tumor grows along a broad front to produce a cauliflowerlike
intraparenchymal mass that pushes lung substance ahead of it.
· As in almost all types of lung cancer, the neoplastic tissue is gray-white and firm to hard.
· Especially when the tumors are bulky, focal areas of
hemorrhage or necrosis may appear to produce red or yellowwhite mottling and softening. Sometimes these necrotic
foci cavitate.
· Histologically, squamous cell carcinoma is characterized by the presence of keratinization and / or intercellular
bridges.
·
Keratinization may take the form of squamous pearls or individual cells with markedly eosinophilic dense cytoplasm.
· These features are prominent in well-differentiated tumors, are easily seen but not extensive in
moderately differentiated tumors, and are focally seen in poorly differentiated tumors.
3. Small cell carcinoma
· They may arise in major bronchi or in the periphery of the lung.
· There is no known preinvasive phase.
· They are the most aggressive of lung tumors, metastasizing widely and virtually always proving to be fatal.
· They are comprised of relatively small cells with scant cytoplasm, ill-defined cell borders, finely granular nuclear
chromatin (salt and pepper pattern), and absent or inconspicuous nucleoli.
· The cells are round, oval, or spindle-shaped, and nuclear molding is prominent.
· The mitotic count is high.
· The cells grow in clusters that exhibit neither glandular nor squamous organization.
· Necrosis is common and often extensive.
· Basophilic staining of vascular walls due to encrustation by DNA from necrotic tumor
cells (Azzopardi effect) is frequently present.
· All small cell carcinomas are high grade.
· Electron microscopy shows densecore
neurosecretory granules, about 100 nm in diameter, in two thirds of cases.
· The occurrence of neurosecretory granules, the expression of neuroendocrine markers such as chromogranin,
synaptophysin, and CD57, and the ability of some of these tumors to secrete hormones (e.g., parathormonerelated
protein, a cause of paraneoplastic hypercalcemia) suggest that this tumor originates
from neuroendocrine progenitor cells, which are present in the lining bronchial epithelium.
· Among the various types of lung cancer, small cell carcinoma is the one that is most commonly
associated with ectopic hormone production.
· Immunohistochemistry demonstrates high levels of the antiapoptotic protein BCL2 in 90% of tumors.
4. Large cell carcinoma
· It is an undifferentiated malignant epithelial tumor that lacks the cytologic features of other forms of lung cancer.
· The cells typically have large nuclei, prominent nucleoli, and a moderate amount of cytoplasm.
· Large cell carcinoma is a diagnosis of exclusion since is
expresses none of the markers associated with adenocarcinoma (TTF-1, napsin A) and squamous cell carcinoma (p63,
p40) .
· One histologic variant is large cell neuroendocrine carcinoma, which has molecular features similar to
those that characterize small cell carcinoma, but is comprised of tumor cells of larger size.

Copyright © 2014 Delhi Academy of Medical Sciences, All Rights Reserved. 64/146
Your Answer. a
Correct Answer. d

Copyright © 2014 Delhi Academy of Medical Sciences, All Rights Reserved. 65/146
(74). A 70-year old man has worked in asbestos industry for a long time. On routine examination, a mass was detected in the apex
of right upper lobe.
Which of the following is the characteristic electron microscopic appearance of the mass?
Copyright © 2014 Delhi Academy of Medical Sciences, All Rights Reserved. 66/146
a. Neurosecretory granules in the cytoplasm
b. Large tubular cells with long villi on their surface
Copyright © 2014 Delhi Academy of Medical Sciences, All Rights Reserved. 67/146
c. Melanosomes
Copyright © 2014 Delhi Academy of Medical Sciences, All Rights Reserved. 68/146
d. Cross striations and desmosomes
Copyright © 2014 Delhi Academy of Medical Sciences, All Rights Reserved. 69/146
Solution. Ans-74: (b) Large tubular cells with long villi on their surface
Ref:
Sol:
The clinical features suggest a diagnosis of mesothelioma. Mesothelioma is the most specific neoplasm associated with asbestos
exposure.
PLEURAL TUMORS
· Pleura may be involved by primary or secondary tumors.
· Secondary metastatic involvement is far more common than are primary tumors.
· The most frequent metastatic malignancies arise from primary neoplasms of the lung and breast.
· Primary tumors-may arise from visceral or parietal pleura.
Primary tumors of pleura include –
1. Solitary fibrous tumor
· Soft tissue tumors occur in the pleura and less commonly in the lung as well as other sites.
· Has no relation to asbestos exposure.
· The tumor cells are CD34+ and keratin-negative by immunostaining, features that are helpful in distinguishing these lesions from
malignant mesotheliomas (which show the opposite phenotype).
2. Malignant mesotheliomas
· There is a long latent period of 25 to 45 years for the development of asbestos-related mesothelioma, and there seems to be no
increased
risk of mesothelioma in asbestos workers who smoke.
· This is in contrast to the risk of asbestos-related lung carcinoma, already high, which is markedly magnified by smoking.
· Although several cytogenetic abnormalities have been detected, the most common is homozygous deletion of the tumor
suppressor gene
CDKN2A/INK4a, which occurs in about 80% of mesotheliomas.
· Demonstration of this deletion (usually by FISH) involving chromosome 9p can be very helpful in distinguishing mesothelioma from
reactive
mesothelial proliferations.
GROSS MORPHOLOGY –
· Mesotheliomas are diffuse lesions arising either from the visceral or parietal pleura.
· They spread widely in the pleural space.
· They are usually associated with extensive pleural effusion and direct invasion of thoracic structures.
· The affected lung becomes ensheathed by a thick layer of soft, gelatinous, grayish pink tumor tissue.
HISTOPATHOLOGY –
· Microscopically, malignant mesotheliomas may be epithelioid, sarcomatoid or mixed.
· This is based on the concept that mesothelial cells have the potential to develop as epithelium-like cells or mesenchymal stromal
cells.
· The epithelioid
type of mesothelioma consists of cuboidal, columnar, or flattened cells forming tubular or papillary structures resembling
adenocarcinoma.
Epithelioid mesothelioma may at times be difficult to differentiate grossly and histologically from pulmonary adenocarcinoma.
· Features that favor mesothelioma include:
· Positive staining for acid mucopolysaccharide, which is inhibited by previous digestion by hyaluronidase;
· Lack of staining for carcinoembryonic antigen and other epithelial glycoprotein antigens, markers that are generally expressed by
adenocarcinoma;
Strong staining for keratin proteins, with accentuation of perinuclear rather than peripheral staining;
· Positive staining for calretinin, Wilms tumor 1 (WT-1), cytokeratin 5/6, and D2–40;
ELECTRON MICROSCOPY –
· On electron microscopy, malignant mesotheliomas show the presence of long microvilli and abundant tonofilaments but absent
microvillous
rootlets and lamellar bodies.
· This is in contrast to pulmonary adenocarcinoma which is characterised by short and plump microvilli.
· This panel of antibodies is diagnostic in a majority of cases when interpreted in the context of morphology and clinical
presentation.
· The mesenchymal type of mesothelioma (sarcomatoid type) appears as a spindle cell sarcoma, resembling fibrosarcoma.
· Sarcomatoid mesotheliomas tend to have lower expression of
many of the markers described previously and some may be positive only for keratin.
· The mixed (biphasic) type of mesothelioma contains both epithelioid and sarcomatoid patterns.
CLINICAL FEATURES –
· The presenting complaints are chest pain, dyspnea, and recurrent pleural effusions.
· Concurrent pulmonary asbestosis (fibrosis) is present in only 20% of individuals with pleural mesothelioma.
· The lung is invaded directly, and there is often metastatic spread to the hilar lymph nodes and eventually, to the liver and other
distant
organs.
· Fifty percent of patients die within 12 months of diagnosis, and few survive longer than 2 years.
· Aggressive therapy (extrapleural pneumonectomy, chemotherapy, radiation therapy) seems to improve this poor prognosis in
some patients.
· Mesotheliomas also arise in the peritoneum, pericardium, tunica vaginalis, and genital tract (benign adenomatoid tumor).
· Peritoneal mesotheliomas are related to heavy asbestos exposure in 60% of male patients (the number is much lower in females).
· Although in about half cases the disease remains confined to the abdominal cavity, intestinal involvement frequently leads to death
from
intestinal obstruction or inanition.
KNOW MORE:
Asbestos-related diseases include –
· Localized fibrous plaques or, rarely, diffuse pleural fibrosis
· Pleural effusions, recurrent
· Parenchymal interstitial fibrosis (asbestosis)
· Lung carcinoma
· Mesotheliomas
· Laryngeal, ovarian and perhaps other extrapulmonary neoplasms, including colon carcinomas;
· Increased risk for systemic autoimmune diseases and cardiovascular disease has been proposed
Copyright © 2014 Delhi Academy of Medical Sciences, All Rights Reserved. 70/146
Your Answer. Not Attempted
Correct Answer. b
(75). Azzopardi effect is seen in which type of lung carcinoma
a. Small cell carcinoma
b. Large cell carcinoma
c. Squamous cell carcinoma
d. Adenocarcinoma
Solution. Ans-75: (a) Small cell carcinoma
Ref.: Read the text below
Sol :
In small cell Ca of lung,blood vessels in necrotic area may show smudged hematoxophilic material in their walls, which represents
DNA
released from tumour cells. This is referred to as Azzopardi effect.
Your Answer. a
Correct Answer. a
(76). Lepidic growth pattern is seen in which type of lung carcinoma
a. Small cell carcinoma
b. Large cell carcinoma
c. Squamous cell carcinoma
d. Adenocarcinoma
Solution. Ans-76: (d) Adenocarcinoma
Ref.: Read the text below
Sol :
Lung tumorAdenocarcinoma
● Mixed subtype is mostly dropped (previously included 95% of all cases so not that helpful); call "mixed mucinous and non
mucinous" if
at least 10% of each component is present
● Concept of “predominant” growth pattern is added (predominant: largest percentage, even if <50%)
● Minimally Invasive Adenocarcinoma (MIA) type is added; predominant lepidic growth pattern; excellent prognosis if invasion of 5
mm
or less
● Lepidic predominant and mucinous adenocarcinoma are added as variants; lepidic is defined as “growth restricted to neoplastic
cells,
along preexisting alveolar structures, lacking stroma, vascular or pleural invasion, with no papillary or micropapillary patterns, and
no
intra-alveolar tumor cells
● Bronchioloalveolar carcinoma (BAC) is dropped; BAC growth pattern regarded as pre-invasive lesion; entities formerly known as
BAC,
i.e. tumors with a prominent or exclusive lepidic growth pattern, are reclassified based on size and morphological features
● Adenocarcinoma in situ (AIS) is subdivided into nonmucinous and mucinous variants, but virtually all cases are non-mucinous,
consisting of type II pneumocytes or Clara cells; considered an intermediate step between normal and invasive adenocarcinoma;
has
lepidic growth pattern; no papillary or micropapillary patterns, no intra-alveolar tumor cells
Your Answer. Not Attempted
Correct Answer. d
Copyright © 2014 Delhi Academy of Medical Sciences, All Rights Reserved. 71/146
(77). Wilson’s disease is due to mutation in
a. ATP 7A
b. ATP 7B
c. ATQ 7B
d. ATR 7B
Solution. Ans-77: (b) ATP 7B
Ref.: Read the text below
Sol :
· The Wilson's disease gene (ATP7B) has been mapped to chromosome 13 (13q14.3) and is expressed primarily in the liver, kidney,

and
placenta.
· The gene codes for a P-type (cation transport enzyme) ATPase that transports copper into bile and incorporates it into

ceruloplasmin.
· Mutations can be detected in 90%.
Most (60%) are homozygous for ATP7B mutations (two abnormal copies), and 30% have only one abnormal copy. Ten per cent
have no
detectable mutation
Your Answer. c
Correct Answer. b
(78). Which of the following is not a form of vaccine derived polio virus (VDPV)?
a. mVDPVs
b. iVDPVs
c. cVDPVs
d. aVDPVs
Solution. Ans-78: (a) mVDPVs
Ref:K. Park’s ‘Textbook of Preventive and Social Medicine’; 22/e, pg 185
Sol :
VACCINE-DERIVED POLIOVIRUS (VDPV) –
· VDPVs resemble wild polio viruses (WPVs) biologically and differ from the majority of vaccine-related poliovirus (VRPV) isolates in
that they have genetic properties consistent with prolonged replication or transmission, which is substantially longer than the normal
period of vaccine virus replication of 4 – 6 weeks in the OPV recipient.
· VDPVs are divided into three categories –
1. Circulating VDPV (cVDPV) –
· Evidence of person-to-person transmission in the community exists.
2. Immunodeficiency-associated VDPV (iVDPV) –
· These are isolates from persons with primary immunodeficiencies, who have prolonged VDPV infections.
3. Ambiguous VDPV (aVDPV) –
· They are either clinical isolates from persons with no known immunodeficiency or sewage isolates whose source is unknown.
The key risk factors for cVDPV emergence and spread are –
· Development of immunity gaps arising from low rates of poliovirus vaccine coverage
· Prior elimination of the corresponding WPV serotypes
· Low rates of routine immunization coverage with trivalent OPV coupled with emphasis on the use in supplementary immunization
activities (SIAs) of monovalent OPV and bivalent OPV (types 1 and 3)
· Intensive AFP surveillance.
The prolonged large outbreak of cVDPV2 in Nigeria and Democratic Republic of Congo, the increased detection of iVDPV infection
in
developing countries and continued detection of aVDPVs that resemble cVDPV and iVDPV reaffirm the following points –
· The clinical signs and severity of paralysis associated with VDPV and WPV infections are indistinguishable.
· cVDPVs pose the same public health threat as WPVs and require the same control measures.
· Surveillance for WPVs and VDPVs should continue to be strengthened.
· Environmental surveillance to detect VDPV and WPV infection can serve as an important, senstitive supplement to AFP
surveillance
in many settings.
· Persons with prolonged iVDPV infections may transmit the polio virus to others, raising the risk of VDPV circulation in the settings
of
low population immunity to the corresponding poliovirus serotype.
· Prolonged iVDPV excretion is uncommon among persons with primary immunodeficiencies exposed to OPV.
The prevalence of long-term iVDPV excretors may be higher than suggested
Your Answer. Not Attempted
Correct Answer. a
Copyright © 2014 Delhi Academy of Medical Sciences, All Rights Reserved. 72/146
(79). Regarding Chandler’s index, all are true except –
a. Calculates the average number of hookworm eggs per gram of faeces for the entire community
b. Indicates magnitude of public health problem
c. Used for detecting case severity and prognosis
d. Indicates effectiveness of mass treatment
Solution. Ans-79: (c) Used for detecting case severity and prognosis
Ref:K. Park’s ‘Textbook of Preventive and Social Medicine’; 22/e, pg 221
Sol :
CHANDLER’S INDEX –
· Morbidity and mortality of hookworm infection depends much on the worm load.
· Chandler worked out an index on the basis of average number of hookworm eggs per gram of faeces for the entire community.
· This index is still used in epidemiological studies of hookworm disease.
· By this index, worm loads in different population groups can be compared.
· Also, the degree of reduction of egg output after mass treatment can be determined.
Chandler’s index –
Average number of eggs per gram of stools for the community
< 200 Hookworm infection is not of much significance
200 – 250 May be regarded as a potential danger
250 – 300 Minor public health problem
> 300 Major public health problem
Your Answer. c
Correct Answer. c
(80). Under RNTCP, which of the following is used for diagnosis of a TB case?
a. Sputum staining
b. Sputum culture
c. Chest X ray
d. Quantiferon assay
Solution. Ans-80: (a) Sputum staining
Ref:K. Park’s ‘Textbook of Preventive and Social Medicine’; 22/e, pg 394-396
Sol :
· In 1993, the Government of India decided to give a new thrust to TB control activities by revitalizing the National Tuberculosis
Programme (NTP), with assistance from international agencies.
· The Revised National Tuberculosis Control Programme (RNTCP) thus formulated, adopted the internationally recommended
Directly
Observed Treatment Short-Course (DOTS) strategy, as the most systematic and cost-effective approach.
· Political and administrative commitment, to ensure the provision of organized and comprehensive TB control services was
obtained.
· Adoption of smear microscopy for reliable and early diagnosis was introduced in a decentralized manner in the general health
services.
· DOTS was adopted as a strategy for provision of treatment to increase the treatment completion rates.
· Supply of drugs was also strengthened to provide assured supply of drugs to meet the requirements of the system.
· Under the RNTCP, active case finding is not pursued.
· Case finding is passive.
· Patients presenting themselves with symptoms suspicious of tuberculosis are screened through 2 sputum smear examinations.
· Sputum microscopic examination is done in designated RNTCP microscopy centres.
· They are located either in the CHC, PHC, Taluka hospitals or in the TB dispensary.
· Each centre has a skilled technician to ensure quality control.
· A senior TB laboratory supervisor is appointed for every 5 microscopy centres.
· The supervisor rechecks all the positive slides and 10 percent of the negative slides of these five microscopy centres.
· Thus, the error in diagnosing a patient is minimized.
· It is essential to examine 2 sputum specimens of each patient before a conclusive diagnosis can be made.
· One sputum sample is not sufficient for diagnosis as the chance of detecting smear positive case is only 80 percent.
· Sputum microscopy not only confirms the diagnosis, but also indicates the degree of infectivity and response to treatment.
Your Answer. a
Correct Answer. a
Copyright © 2014 Delhi Academy of Medical Sciences, All Rights Reserved. 73/146
(81). Population attributable risk is –
a. Incidence of disease / death in the exposed to those who were not exposed in relation to the total population
b. Incidence of disease / death in the total population to those who were exposed in relation to the total population
c. Incidence of disease / death in the total population to those who were exposed in relation to those who were not exposed
d. Incidence of disease / death in the total population to those who were not exposed in relation to the total population
Solution. Ans-81: (d) Incidence of disease / death in the total population to those who were not exposed in relation to the total
population
Ref: K. Park’s ‘Textbook of Preventive and Social Medicine’; 22/e, pg 75
Sol :
POPULATION ATTRIBUTABLE RISK –
· The concept of population attributable risk is the incidence of the disease (or death) in the toal population minus the incidence of
the
disease (or death) among those who were not exposed to the suspected causal factor.
· This concept is useful in that it provides an estimate of the amount by which the disease could be reduced in the population if the
suspected causal factor was eliminated or modified.
· For example, if the population attributable risk for lung cancer deaths in relation to smoking is 86 percent, one might expect that 86
percent of deaths from lung cancer could be avoided if the risk factor of cigarette smoking were eliminated.
Lung cancer deaths among smokers and non-smokers: UK physicians.
Deaths per 1,00,000 person years
Heavy smokers (Exposed to suspected factor) = a 224
Non-smokers (Not exposed to suspected factor) = b 10
Deaths in total population = c 74
Individual relative risk
a/b
= 224/10
= 22.40
Population attributable risk
(c- b) / c
= (74 – 10) / 74
= 64 / 74
= 86 percent
Your Answer. b
Correct Answer. d
Copyright © 2014 Delhi Academy of Medical Sciences, All Rights Reserved. 74/146
(82). If the association between smoking and CHD increases from low level of physical exercise to high level of physical exercise,
then physical
exercise in relation to the association between smoking and CHD is known as –
Copyright © 2014 Delhi Academy of Medical Sciences, All Rights Reserved. 75/146
a. Collinear factor
b. Confounder
Copyright © 2014 Delhi Academy of Medical Sciences, All Rights Reserved. 76/146
c. Effect modifier
Copyright © 2014 Delhi Academy of Medical Sciences, All Rights Reserved. 77/146
d. Bias factor
Copyright © 2014 Delhi Academy of Medical Sciences, All Rights Reserved. 78/146
Solution. Ans-82: (c) Effect modifier
Ref:https://onlinecourses.science.psu.edu/stat507/node/34
Sol :
EFFECT MODIFIER –
· A variable that differentially (positively and negatively) modifies the observed effect of a risk factor on disease status.
· Different groups have different risk estimates when effect modification is present.
· Effect modification occurs when the effect of a factor is different for different groups.
· We see evidence of this when the crude estimate of the association (odds ratio, rate ratio, risk ratio) is very close to a weighted
average of group-specific estimates of the association.
· Effect modification is similar to statistical interaction, but in epidemiology, effect modification is related to the biology of disease, not
just a data observation.
· Examples –
o The immunization status of an individual modifies the effect of exposure to a pathogen and specific types of infectious diseases.
o Breast Cancer occurs in both men and women. Breast cancer occurs in men at approximately a rate of 1.5/100,000 men. Breast
cancer
occurs in women at approximately a rate of 122.1/100,000 women. This is about an 800 fold difference.
· Effect modification is important –
o To define high-risk subgroups for preventive actions,
o To increase precision of effect estimation by taking into account groups that may be affected differently,
o To increase the ability to compare across studies that have different proportions of effect-modifying groups, and
o To aid in developing a causal hypotheses for the disease.
Option ‘1’: Collinear factor
Ref: http://en.wikipedia.org/wiki/Multicollinearity
· Collinearity (also multicollinearity) is a linear association between two explanatory variables.
· Two variables are perfectly collinear if there is an exact linear relationship between the two.
· Multicollinearity is a statistical phenomenon in which two or more predictor variables in a multiple regression model are highly
correlated.
· This means that one can be linearly predicted from the others with a non-trivial degree of accuracy.
Option ‘2’: Confounder
Ref: https://onlinecourses.science.psu.edu/stat507/node/34
· Confounding is a situation in which a measure of association or relationship between exposure and outcome is distorted by the
presence of another variable.
· Positive confounding (when the observed association is biased away from the null) and negative confounding (when the observed
association is biased toward the null) both occur.
· Confounder is an extraneous variable that wholly or partially accounts for the observed effect of a risk factor on disease status.
· The presence of a confounder can lead to inaccurate results.
· A confounder meets all three conditions listed below:
o It is a risk factor for the disease, independent of the putative risk factor.
o It is associated with putative risk factor.
o It is not in the causal pathway between exposure and disease.
· The first two of these conditions can be tested with data.
· The third is more biological and conceptual.
· Some methods to control for a confounding variable (known a priori) are –
o Randomize individuals to different groups (use an experimental approach)
o Restrict / filter for certain groups
o Match in case-control studies
o Analysis (stratify, adjust)
Option ‘4’: Bias factor
Ref: https://onlinecourses.science.psu.edu/stat507/node/34
· Bias is a systematic error in the design, recruitment, data collection or analysis that results in a mistaken estimation of the true
effect of the exposure and the outcome.
· Bias limits validity (the ability to measure the truth within the study design) and generalizability (the ability to confidently apply the
results to a larger population) of study results.
· Bias is rarely eliminated during analysis.
· There are two major types of bias:
o Selection bias – systematic error in the selection or retention of participants and
o Information bias (misclassification bias) – systematic error due to inaccurate measurement or classification of disease, exposure
or
other variables.
· Misclassification can be differential or non-differential.
o Differential misclassification – the probability of misclassification varies for the different study groups, i.e., misclassification is
conditional upon exposure or disease status.
o Nondifferential misclassification – the probability of misclassification does not vary for the different study groups; is not conditional
upon exposure or disease status, but appears random
· Either type of misclassification can produce misleading results.
To summarize,
If two variables have an exact linear relationship with each other Collinear factors
If the method used to select subjects or collect data results in an incorrect association Bias
If an observed association is not correct because a different (lurking) variable is associated with
both the potential risk factor and the outcome, but it is not a causal factor itself Confounding
If an effect is real but the magnitude of the effect is different for different groups of individuals Effect modification
Copyright © 2014 Delhi Academy of Medical Sciences, All Rights Reserved. 79/146
Your Answer. b
Correct Answer. c
(83). The objective criteria for detecting community spread of pandemic influenza A H1N1 are all except –
a. One or more cases should be laboratory confirmed with pandemic influenza A H1N1
b. Cases should be reported over 2 weeks
c. Cases should be reported in at least 5 or more cities
d. Presence of at least 25 linked epidemiologically suspect cases
Solution. Ans-83: (c) Cases should be reported in at least 5 or more cities
Ref:MoHFW Guidelines on objective criteria to determine community spread of Pandemic Influenza A H1N1 (http://mohfwh1n1.
nic.in./../../../../documents/pdf/1.Guidelines%20on%20community%20spread.pdf
Sol :
PANDEMIC INFLUENZA A H1N1 –
· If there is 25 or more epidemiologically linked suspect cases of Pandemic Influenza A H1N1 of which, at least one or more are
laboratory confirmed for Pandemic Influenza A H1N1, in two or more cities, over a period of two weeks, then the State would be
considered to be having community spread”.
Application
States that report community spread, the curative approach would be followed –
· To screen Influenza like illness in designated health facilities
· Categorization into A, B and C categories
· Home isolation for category A and B and hospitalization for Category C
· Treatment with Oseltamivir would continue to be for Category B and Category C.
States not reporting community spread, would continue to do –
· Testing of suspect cases
· Contact tracing and
· Chemoprophylaxis to family, school and social contacts.
This would continue till such time they report community
Your Answer. b
Correct Answer. c
Copyright © 2014 Delhi Academy of Medical Sciences, All Rights Reserved. 80/146
(84). School closure is a common strategy to keep pandemics of influenza under control in the absence of vaccines in developing
countries.
This rationale is based on all but –
Copyright © 2014 Delhi Academy of Medical Sciences, All Rights Reserved. 81/146
a. School closure does not play a key role in the reduction of contact rates between children
b. Children as the most susceptible age group
Copyright © 2014 Delhi Academy of Medical Sciences, All Rights Reserved. 82/146
c. Children are the primary transmitters of influenza
Copyright © 2014 Delhi Academy of Medical Sciences, All Rights Reserved. 83/146
d. Contact rates are particularly high in school context
Copyright © 2014 Delhi Academy of Medical Sciences, All Rights Reserved. 84/146
Solution. Ans-84: (a) School closure does not play a key role in the reduction of contact rates between children
Ref:MoHFW guidelines for schools during pandemic of influenza (http://mohfwh1n1.
nic.in./../../../../documents/pdf/11.Guidelines_for_Schools_Colleges_and_Institutions.pdf)
Sol :
School closure may play a role in reducing the contact rates between children. The theory behind the mitigating effect of school
closings
is that
· School age children have high contact rates
· They end to be more susceptible to influenza infection than other age groups and
· They have increased viral shedding.
Broad guidelines for schools / educational institutions –
1. Schools are advised to avoid any large gathering of students during the course of the day in the school. This would reduce the
possibility of the spread of the infection, if any, to a much larger number of students who would be in close contact with each other in
such a gathering.
2. It should be made mandatory by the school authorities that all class teachers should begin their class with active screening of
each
student in the class so as to detect any student who is having symptoms of flu (mild fever with cough / sore throat with or without
bodyache, headache, diarrhea and vomiting). If such student is detected, he / she should be referred immediately to the medical
facility
of the school. He / She should be further advised to stay at home for 7 days and observe strict discipline of home isolation. The
onus of
observance of discipline of isolation henceforth is shifted to the parents and school authorities should accordingly write a letter to the
parent. The parents should also be advised to keep other wards at home, in case they are found to be having flu like symptoms.
There
should be constant self-monitoring and if symptoms deteriorates, it should be reported to health authorities immediately.
3. Students, teachers and other employees working in schools / educational institutions are advised to stay at home if they develop
flu
like symptoms. They should consult the medical doctor and take treatment as advised including the home isolation and drugs for
treatment of the symptoms.
4. Students, teachers and other employees working in schools / educational institutions are advised to continue to stay at home for
at
least 7 days if they are advised by the doctor to take Oseltamivir treatment and they should observe home isolation. There should
be
constant self-monitoring and if symptoms deteriorates, it should be reported to health authorities immediately.
5. School authorities should not insist on production of medical certificate from such preventive absentees.
6. Students, teachers and other employees working in schools / educational institutions are advised to wash their hands frequently
with
soap and water.
7. Students, teachers and other employees working in schools / educational institutions are advised to observe strict cough / sneeze
etiquette i.e. use tissue while sneezing and coughing. The tissue paper so used should be kept in a separate plastic bag, so that it
can be
disposed of safely.
8. If any student, teachers or other employees is at high risk (suffering from chronic diseases of lung / heart / kidney /neuroloigcal
system or blood disorder) for H1N1 and he / she becomes sick with influenza-like sickness, the school authories should immediately
take
him/her to the medical officer of the school and carry out further treatment strictly under advice of the doctor. Mostly such patients
will
be kept on Oseltamivir and home isolation.
9. All the schools / educational institutions should observe regular cleaning of the area with cleaner they ordinarily use so that all the
droplets and shredding from any unnoticed mildly infected students / employees are taken care of.
10. Closure of schools has not been recommended by Centre for Disease Control, Atlanta, USA. Its value for prevention of spread
of
disease within school is outweighed by the possibility of community spread, which is more likely because the children will play and
mix
with public in various places and at social gathering and there will be nobody to advise them or prevent such incidents. On the other
hand, in schools there will be a teacher who will be able to detect their symptoms everyday at the earliest. However, school
authorities
have to use their own judgement for any type of temporary closure in the event of wide spread H1N1 influenza in school. There
should be
constant self-monitoring and if symptoms deteriorate, it should be reported to health authorities immediately.
11. In case of students staying in hostels, the school authority should monitor the health status of students as well as the other
ancillary
staff in the hostel on regular basis.
12. In case there are suspected cases in the hostel, the authorised local medical authority should be called for examination of all
students
and school authority should not close the hostel and send the students back to their home.
13. School should discourage excursion of the students to the affected countries.
14. However, if students go on tour to the affected countries, they should be thoroughly examined on return by medical doctor
before
permitting attendance in the class. If there is one suspect case of H1N1 then the whole group should be kept under isolation at
home and
their health status should be regularly watched.
15. All the schools should display “DO’S AND DON’TS” for H1N1 infection at all important places.
16. All the schools should circulate pamphlets containing “DO’S AND DONT’S” for H1N1 infection and answers to frequently asked
questions (FAQ) to the students.
Your Answer. a
Correct Answer. a
Copyright © 2014 Delhi Academy of Medical Sciences, All Rights Reserved. 85/146
(85). Which of the following strategies is employed by the National Programme for Control of Blindness to evaluate post-operative
visual
outcomes following cataract surgery extraction?
a. Routine monitoring of all operated cases
b. Passive surveillance
c. Sentinel surveillance
d. Active surveillance
Solution. Ans-85: (b) Passive surveillance
Ref:K. Park’s ‘Textbook of Preventive and Social Medicine’; 22/e, pg 406
Sol :
· Revised strategies under NPCB includes shift from the Eye Camp Approach to a Fixed Facility Surgical Approach.
· The Eye Camp approach is synonymous to field visit which comes under the definition of active Surveillance.
· The Fixed Facility approach necessitates that the patient comes to a fixed facility for follow-up after the surgery which comes under
the definition of passive surveillance.
Note that routine monitoring of all operated cases (option ‘1’) is done in all cases but is not specific to NPCB.
KNOW MORE –
· In India, the first eye bank was established in 1945 at Madras [Chennai].
· Dr. Dhanda of Indore carried the first successful corneal transplantation in 1960.
Your Answer. b
Correct Answer. b
Copyright © 2014 Delhi Academy of Medical Sciences, All Rights Reserved. 86/146
(86). The outcome indicator for National Programme for the Control of Blindness is –
a. Decrease in the prevalence of blindness
b. Increase in the number of children with corrected refractive errors
c. Number of cataract surgeries with restored vision
d. Number of eye units upgraded
Solution. Ans-86: (c) Number of cataract surgeries with restored vision
Ref:Murthy G, Gupta S, John N and Vasisht P. Current status of cataract blindness and Vision 2020: The right to sight initiative in
India.
Indian J Ophthalmol. 2008;56(6):489–94
Sol :
· At the turn of the century, WHO and the International Agency for Prevention of Blindness launched the Vision 2020: the right to
sight initiative.
· The most recent estimates from WHO reveal that 47.8% of global blindness is due to cataract and in South Asia region which
includes India, 51% of blindness is due to cataract.
· Since cataract is a major cause of avoidable blindness in the developing countries, the key to the success of the Global Vision
2020:
the right to sight initiative is a special effort to tackle cataract blindness.
· Cataract surgery has been viewed as one of the most cost-effective health interventions with a cost of disability-adjusted life years
saved of US $ 20-40.25
· Cataract surgical rate is a quantifiable measure of the delivery of cataract services in a country.
· It is thus a good indicator of how well a country is organizing its efforts in tackling cataract-related blindness.
· There has been a substantial increase in CSR in India especially after the inception of the World Bank-supported Cataract
Blindness
Control Project.
· Professional interest and technological upgradation of skills and the availability of affordable equipment and intraocular lenses
have
all fuelled the increase in cataract surgery in India.
· A CSR of 3000 was targeted under Vision 2020: the right to sight, for India, by the year 2000.
· Current trends show that this target has been achieved, but still there are regional disparities across the country.
· The CSR for the year 2002-03 ranges from a high of 8440 per million population to a low of 130 per million population.
· Most of the bigger states in the country have already achieved a CSR of > 4000 per million population.
· The states of Gujarat (8440), Puducherry (7440), Tamil Nadu (5920) Andhra Pradesh (5260), Delhi (5090), Punjab (4950),
Maharashtra (4840), Karnataka (4560) and Haryana (4180) have already gone past the recommended norm for Vision 2020: the
right to
sight.
· In view of this encouraging performance, it is very likely that the entire country can achieve a CSR of 6000 + per million population
by 2020.
· This increased performance will reduce the prevalence of blindness and severe visual impairment in the country as half the
blindness in India is attributable to cataract.
· Cataract surgical coverage (CSC) is an efficient indicator for planning as it provides information on what proportion of those
needing
surgery have been covered and therefore is a good indicator of the work remaining.
· However, CSC needs population-based surveys to provide information which is not easily available as against CSR which is most
readily available.
· It is difficult to say if there is any over-reporting in cataract surgery when information is compiled at the national level.
· However, it needs to be emphasized that outcome measures like CSR, by themselves are inadequate to describe the benefit to the
operated individuals and their quality of life.
· More widespread use of indicators like Sight Restoration Rate (SRR) and developing sensitive indicators for assessment of visual
function after surgery should be effectively used in the future.
· At the same time, other causes of avoidable blindness need to be given adequate attention.
Your Answer. c
Correct Answer. c
Copyright © 2014 Delhi Academy of Medical Sciences, All Rights Reserved. 87/146
(87). The chairman of the District Blindness Control Society is –
a. District Health Officer
b. District Collector
c. District Program Manager
d. District Eye surgeon
Solution. Ans-87: (b) District Collector
Ref:K. Park’s ‘Textbook of Preventive and Social Medicine’; 22/e, pg 406
Sol :
DISTRICT BLINDNESS CONTROL SOCIETY –
· The conept of District Blindness Control Society (DBCS) was first developed in India in five pilot districts.
· Based on the success of these socities, as many as 604 DBCS have been formed under the Chairmanship of District Collector /
Deputy Commssioner.
· These socities have multidisciplinary structure with representatives from government, non-government and private sector.
Development of infrastructure for eye care –
Item Current achievement
Strengthening of PHCs 5633
Paramedical Ophthalmic Assistants posted 4881
Establishment of DBCS 604
Strenghtening of District Hospitals 445
District Mobile Units 341
Eye banks (Govt.) 166
Upgrading of Departments of Ophthalmology in Medical Colleges 82
Central Mobule Units 80
Ophthalmic Assistant Training Centres 39
State Ophthalmic Cells 21
Establishment of Regional Institutes 11
Your Answer. a
Correct Answer. b
Copyright © 2014 Delhi Academy of Medical Sciences, All Rights Reserved. 88/146
(88). Regarding Cataract Surgical Rate (CSR), which of the following is incorrect?
Copyright © 2014 Delhi Academy of Medical Sciences, All Rights Reserved. 89/146
a. It is the number of cataract surgeries performed per million population per year
b. It is a true rate
c. A CSR of 3000 is recommended under Vision 2020
d. It indicates the effectiveness of cataract services delivery
Solution. Ans-88: (b) It is a true rate
Ref:Murthy G, Gupta S, John N and Vasisht P. Current status of cataract blindness and Vision 2020: The right to sight initiative in
India.
Indian J Ophthalmol. 2008;56(6):489–94
Sol :
CATARACT SURGICAL RATE (CSR) –
CSR is defined as:
(Total number of cataract surgeries performed) / (Total population) × 1,000,000.
· Conventionally, a rate as defined in epidemiology has a numerator which is related to and derived from the denominator and has a
specific time dimension (such as one year).
· As against ′rates′, CSR is a proportion, where the numerator may not be derived from the denominator.
· Therefore it was felt that using the 50+ population as the denominator rather than the whole population would be more appropriate
as cataract blindness and cataract surgery are commonly seen above the age of 50 years.
· Therefore a new indicator was defined and labeled as ″CSR 50+″.
· This indicator was defined as:
· (Total number of cataract surgeries performed) / (Total 50+ population) × 1,000,000
· In epidemiologic parlance, a ′true′ rate is a proportion and is defined as the presence or absence of a characteristic in a group of
people among people who are at risk of developing the particular disease during a specific time period.
· Therefore the denominator should comprise only individuals in the population who are at risk of developing the disease.
· In such a case, the denominator of the general population is not appropriate for defining CSR as the younger population would not
be at risk of age-related cataract.
· It is therefore recommended that since most blinding cataract occurs after the age of 50 years, the denominator for CSR should
consist only of the 50+ population.
Ref: http://www.iapb.org/vision-2020/what-is-avoidable-blindness/cataract; Foster A. Vision 2020: The Cataract Challenge.
Community Eye Health. 2000; 13(34):17–19.
CATARACT SURGICAL RATE –
· Cataract remains the leading cause of blindness and an important cause of visual impairment across the globe.
· Cataract surgical rate (CSR) is the number of cataract operations per million population per year.
· CSR serves as a quantifiable measure of cataract surgical service delivery, but this indicator is meaningful only when it includes all
cataract surgeries performed in a country with a defined population size and age structure.
· Economically well-developed countries usually perform between 4000 and 6000 cataract operations per million population per
year.
· At this level of service it is unusual to find people who are blind from unoperated cataract – although several population-based
studies show that even in industrialised countries not all those with visual impairment due to cataract, enquire or accept surgery.
· India has dramatically increased its CSR in the last 10 years from less than 1500 to a figure of around 3000 today.
· In the middle income countries of Latin America and parts of Asia the CSR is between 500 and 2000 per million per year.
· In most of Africa, China and the poorer countries of Asia the rate is often less than 500.
· The major reasons for low cataract surgical rates include –
o Low demand because of fear of surgery
o Low demand from poor people because of high cost of surgery
o Low demand because of poor visual results
o Lack of eye surgeons, particularly in Africa.
· In order to reduce the cataract backlog it is necessary to have a cataract surgical rate which is at least as great as the incidence of
‘operable’ cataract, where ‘operable’ depends upon the indication for surgery.
· In India and other countries of South East Asia, in order to deal with cataract causing an acuity less than 6/60, it is necessary to do
at least 3000 operations per million population per year and perhaps more.
· In Africa, and other parts of the world where there is a lower percentage of elderly people in the population, a realistic target for the
next 5–10 years is around 2000 operations/million population /year.
· It is possible to achieve these rates if good quality cataract surgery is performed at a reasonable cost, close to where people live.
· Models for this type of cataract service have now been developed in several developing countries, most notably in India.
KNOW MORE:
Indicators of cataract service delivery:
· Prevalence and causes of visual impairment, including blindness (preferably disaggregated by age and gender)
· Number of eye care personnel by cadre: ophthalmologists
· Number of eye care personnel by cadre: allied ophthalmic personnel
· Cataract surgical rate
· Cataract surgical coverage.
Your Answer. b
Correct Answer. b
Copyright © 2014 Delhi Academy of Medical Sciences, All Rights Reserved. 90/146
(89). Which of the following is true regarding Rashtriya Swastha Bima Yojana (RSBY)?
a. It covers expenditure incurred in both outpatient and hospital
b. The entitlement is for Rs. 30,000 per family member
c. Only below-poverty line (BPL) families to be covered under RSBY
d. The actual health expenditure is borne by the family and subsequently reimbursed by the Government
Solution. Ans-89: (c) Only below-poverty line (BPL) families to be covered under RSBY
Ref:http://en.wikipedia.org/wiki/Rashtriya_Swasthya_Bima_Yojana
Sol :
· Rashtriya Swasthya Bima Yojana (RSBY, literally "National Health Insurance Programme") is a government-run health insurance
scheme for the Indian poor.
· It is a project under the Ministry of Labour and Employment.
· It provides for cashless insurance for hospitalisation in public as well private hospitals.
· The scheme started enrolling on April 1, 2008 and has been implemented in 25 states of India.
· A total of 23 million families have been enrolled as of February 2011.
· Every "below poverty line" (BPL) family holding a yellow ration card pays Rs. 30 registration fee to get a biometric-enabled smart
card containing their fingerprints and photographs.
· This enables them to receive inpatient medical care of up to Rs. 30,000 (as of March 2011) per family per year in any of the
empanelled hospitals.
· Pre-existing illnesses are covered from day one, for head of household, spouse and up to three dependent children or parents.
· The scheme is run on shared financial contribution by both central and state governments: 75% of the 600 (approx US$13)
premium
(per person per year) is borne by the central government, and the rest by the state governments.
· There is profit motive for all parties involved.
· Private insurance companies provide the risk coverage, while private third-party administrators empanel hospitals and manage
claims.
· Fixed rates have been prescribed for a long list of interventions.
Thus,
· RSBY covers expenditure incurred only in hospitals, not outpatient expenditure.
· The entitlement is for Rs. 30,000 per family, not per family member.
The actual health expenditure is borne by the Government.
Your Answer. d
Correct Answer. c
(90). A medical officer wants to estimate the number of children in a village. He can get a rough estimate quickly from –
a. Number of women in the reproductive age group
b. TT doses consumed in the last 1 year
c. Adult Folifer consumption in the last 1 year
d. Total population of the village
Solution. Ans-90: (c) Adult Folifer consumption in the last 1 year
Ref:
Sol :
Option ‘3’ is the answer of exclusion. Number of Folifer tablets consumed can give a rough idea about the number of children in a
village.
Option ‘1’: Number of women in the reproductive age group – cannot be used to calculate the number of children as each woman
can
give birth to more than one child.
Option ‘2’: TT doses consumed in the last 1 year – also cannot determine the total number of children as TT doses can be
administered to
pregnant women or even to adults in case of injury.
Option ‘4’: Total population of the village – Trying to estimate the number of children from total population is absolutely ridiculous.
Your Answer. b
Correct Answer. c
Copyright © 2014 Delhi Academy of Medical Sciences, All Rights Reserved. 91/146
(91). The nutritional supplementation recommended under ICDS for pregnant women is –
a. 300 kcal and 10 g of proteins
b. 200 kcal and 12 g of proteins
c. 600 kcal and 18 g of proteins
d. 250 kcal and 15 g of proteins
Solution. Ans-91: (c) 600 kcal and 18 g of proteins
Ref:K. Park’s ‘Textbook of Preventive and Social Medicine’; 22/e, pg 547
Sol :
Under ICDS, a pregnant woman is given nutritional supplementation of 600 calories and 18 – 20 grams of protein per day.
SUPPLMENTARY NUTRITION UNDER ICDS –
· Supplementary nutrition is given to children below 6 years, and nursing and expectant mothers from low income group.
· The type of food depends up on local availability, type of beneficiary, location of the project, etc.
· The aim is to supplement nutritional intake for 300 days in a year as follows –
Beneficiary Energy (calories) Proteins (g) Financial norm (per beneficiary
per day)
Children 6 – 72 months of age 500 12 – 15 4
Severely malnourished children 6 –
72 months of age 800 20 – 25 6
Pregnant and nursing women 600 18 – 20 5
Your Answer. c
Correct Answer. c
(92). A 35-year old male complains of vision distortion. The distortion increases progressively in both the meridians on wearing his
spectacles.
All of the following are correct regarding the patient’s problem except –
a. It is also called pincushion distortion
b. It is due to wearing of cylindrical glasses
c. It is due to asymmetrical convex lenses in both the eyes
d. It is aniseikonia
Solution. Ans-92: (a) It is also called pincushion distortion
Ref: Read the text below
Sol :
The description of the distortion is characteristic of aniseikonia, which is different from pincushion distortion.
· In pincushion distortion, image magnification increases with the distance from the optical axis.
· The visible effect is that lines that do not go through the centre of the image are bowed inwards, towards the centre of the image,
like a pincushion
ANISEIKONIA –
· Aniseikonia is an ocular condition where there is a significant difference in the perceived size of images.
· It can occur as an overall difference between the two eyes, or as a difference in a particular meridian.
· Aniseikonia can occur naturally or be induced by the correction of a refractive error, usually anisometropia (having significantly
different refractive errors between each eye) or antimetropia (being myopic (nearsighted) in one eye and hyperopic (farsighted) in
the
other.)
· Meridional aniseikonia occurs when these refractive differences only occur in one meridian.
· Refractive surgery can cause aniseikonia in much the same way that it is caused by glasses and contacts.
· One cause of significant anisometropia and subsequent aniseikonia has been aphakia.
· Aphakic patients do not have a crystalline lens.
· The crystalline lens is often removed because of opacities called cataracts.
· The absence of this lens leaves the patient highly hyperopic (farsighted) in that eye.
· For some patients the removal is only performed on one eye, resulting in the anisometropia / aniseikonia.
· Today, this is rarely a problem because when the lens is removed in cataract surgery, an intraocular lens, or IOL is left in its place.
Your Answer. b
Correct Answer. a
Copyright © 2014 Delhi Academy of Medical Sciences, All Rights Reserved. 92/146
(93). Under hypoxic conditions, which of the following substances is strongly implicated as a cause of corneal edema?
a. Lactate
b. Glycogen
c. Carbon dioxide
d. Pyruvate
Solution. Ans-93: (a) Lactate
Ref: Yanoff & Ducker’s Ophthalmology; 4/e
Sol :
CONTACT LENS AND CORNEAL PHYSIOLOGY –
· Contact lenses are synthetic polymer foreign bodies worn on the surface of the eye.
· Although they are generally very well tolerated, they cause numerous changes in corneal physiology and the tear film.
· Most attention has been given to corneal hypoxia: decreased oxygen tension that occurs with contact lens use, and especially
when
contact lenses are worn overnight.
· If the lids are closed for more than 5 minutes, the oxygen tension drops from 155 mmHg to 57 mmHg.
· Corneal carbon dioxide also increases due to retention and a shift in glucose metabolism that increases the production of CO2,
causing a shift to acidic pH in the corneal stroma.
Stromal edema following hypoxia reflects an osmotic solute load produced by lactate accumulation
Your Answer. a
Correct Answer. a
Copyright © 2014 Delhi Academy of Medical Sciences, All Rights Reserved. 93/146
(94). Which of the following can be used for self tonometry?
a. Dynamic contour tonometer
b. Diaton tonometer
c. Rebound tonometer
d. Perkins tonometer
Solution. Ans-94: (c) Rebound tonometer
Ref: Yanoff & Ducker’s Ophthalmology; 4/e; www.ncbi.nlm.nih.gov
Sol :
· Clinical measurement of IOP has undergone several technical advances from the initial digital tension measurements, through
indentation tonometry, to applanation tonometry and non-contact tonometry.
· The current gold standard for the measurement of intraocular pressure is the Goldmann applanation tonometer.
INDENTATION TONOMETRY –
· Indentation tonometry introduced by Schiøtz in 1905 measures IOP following a principle of indentation of the cornea.
· The anesthetized cornea is indented by a metallic footplate that is attached to a plunger of known weight (5.5 g) encased in a
shaft.
· Indentation of the cornea results in the displacement of a set volume of fluid in the eye, which is represented by the movement of a
lever along a measuring scale that is attached to the plunger and is proportional to the pressure inside the eye .
· This instrument comes with plungers of different weights, which can be used in situations when the eye is soft or hard.
· A large amount of displacement of the lever along the measuring scale represents a low IOP and small amount of displacement
represents a high IOP.
APPLANATION TONOMETRY –
· The Goldman applanation tonometer is based on Imbert-Fick principle, which states that the force necessary to applanate a
perfectly
infinitely thin and dry sphere divided by the area flattened (P = F/A) equals the pressure inside the sphere.
PORTABLE VARIABLE-FORCE TONOMETERS –
· Portable applanation tonometers with built-in biprisms are called Perkins tonometers.
· This tonometer allows the clinician to measure IOP in the supine position.
· Pneumotonometry performed with the Mackay-Marg-type tonometer measures IOP via a central sensing device controlled by air
pressure, while the force required to bend the cornea is transferred to a surrounding sleeve.
· The pneumotonometer can be used in the presence of scarred and irregular corneas.
· The Tono-Pen is a variation of the pneumotonometer that uses a strain gauge to create an electrical impulse as the footplate
flattens
the cornea.
· A microprocessor chip senses appropriate force curves, calculates the average of 4–10 readings, and produces a final digital
readout
with variability percentages.
· The Tono-Pen underestimates IOP in the presence of high IOP and overestimates IOP in the presence of low IOP measured with
the
Goldmann applanation tonometer.
NON-CONTACT TONOMETRY –
· Noncontact tonometers flatten the cornea using a puff of air; the time required to flatten the cornea (measured in milliseconds) is
then correlated with IOP.
· Noncontact tonometry measurements have been shown to be affected by ocular pulsation and can lead to errors in IOP estimation
ranging from 1 to 3 mm Hg.
DIGITAL CONTOUR TONOMETRY –
· The dynamic contour tonometer, also known as the PASCAL tonometer (Swiss microtechnology), has been recently introduced.
· The principle behind this technology relies on the design of the measuring plastic head tip, which matches the curvature of the
cornea and minimizes tissue distortion while obtaining direct IOP readings that are not affected by central corneal thickness.
REBOUND TONOMETRY –
· It is a novel, portable device that contacts the cornea, but does not require topical anaesthesia or use of fluorescein.
· It is suitable for patients with disability and screening at home, as well as saving space and time in the consulting room.
DIATON TONOMETER –
· It is a hand held device for measuring IOP through the eye lid and over the sclera with no contact to the cornea.
Your Answer. Not Attempted
Correct Answer. c
Copyright © 2014 Delhi Academy of Medical Sciences, All Rights Reserved. 94/146
(95). Which diagnostic procedure is not done in a dilated pupil?
a. Gonioscopy
b. Laser inferometry
c. Fundus examination
d. Electroretinography
Solution. Ans-95: (a) Gonioscopy
Ref: Basics and Clinical Science Course, American Academy of Ophthalmology, 2012-13; Sec 10, pg 46
Sol :
· By gonioscopy we assess angle structures, especially relationship of root of iris and trabecular mesh work in normal physiological
state.
· To avoid pupillary contraction, it is done in dark room with minimum slit lamp light.
NOTE: Pupil contraction will falsely open up the angle. Likewise pupillary dilatation will cause crowding of the iris at angle and angle
appears as closed
Your Answer. a
Correct Answer. a
(96). The normal ratio of light peak and dark trough on an EOG is –
a. 1
b. 1.5
c. More than 185%
d. Less than 185%
Solution. Ans-96: (c) More than 185%
Ref: Parsons’ ‘Diseases of the Eye’; 21/e
Sol :
ELECTRO-OCULOGRAPHY
· In this technique, changes in the resting ocular potential are picked up by electrodes placed at the inner and outer canthi when the
eyes are moved form side to side.
· Changes in the potential thus obtained with changes of illumination are indicative of the activity of the pigmentary epithelium and
the outer segments of the visual receptors.
· These changes are often diminished or absent in retinal dystrophies and degenerations before visual symptoms are evident.
· The patient is asked to alternately look towards two targets placed in front of him to the right and to the left, and the potentials
recorded by the electrodes are printed.
· The potentials are continuously recorded for a fixed time interval in a light-adapted and dark adapted state.
· The ratio of the light peak over the dark trough is known as Arden Index.
· A value above 185 is normal, below 150 abnormal and 150-185 borderline.
· The technique is of value in diagnosing objectively the early stages of retinal diseases, especially those that affect the retinal
pigment epithelium, or in cases where the fundus cannot be clearly seen.
The electrodes and apparatus used for EOG can also be used to measure the change in potentials produced by ocular movements
and
record the pattern of eye movements in nystagmus (electronystagmography or ENG).
Your Answer. d
Correct Answer. c
Copyright © 2014 Delhi Academy of Medical Sciences, All Rights Reserved. 95/146
(97). A diagnosis of fungal corneal ulcer is not made if there is presence of –
a. Marked photophobia and pain compared to signs
b. Sloughing corneal ulcer
c. Fixed hypopyon
d. Presence of hyphae on KOH mount
Solution. Ans-97: (a) Marked photophobia and pain compared to signs
Ref: Parsons’ ‘Diseases of the Eye’; 21/e
Sol :
FUNGAL CORNEAL ULCERS –
· Mycotic or fungal keratitis is frequently seen in tropical countries, rural areas and in immunocompromised individuals.
· It is commonly due to Aspergillus, Fusarium, or Candida albicans.
· Fungal ulcers are typically seen after injury with vegetable matter such as thorn or wooden stick and are characterised by a
relatively indolent course.
· Symptoms are much milder than the clinical signs would suggest.
· The slough in these ulcers is dry in appearance with feathery borders, surrounded by a yellow line of demarcation which gradually
deepens into a gutter, and there may also be a hypopyon.
· An immune ring (Wesseley) may be visible due to deposition of immune complexes and inflammatory cells around the ulcer.
· Satellite lesions may also be seen.
· There is marked ciliary and conjunctival congestion, but symptoms of pain, watering and photophobia are disproportionately less
as
compared to those in cases of bacterial corneal ulcers.
The hypopyon, if present is thick and immobile, and is due to direct invasion into the anterior chamber of fungal hyphae enmeshed
in
thick exudates
Your Answer. b
Correct Answer. a
Copyright © 2014 Delhi Academy of Medical Sciences, All Rights Reserved. 96/146
(98). A patient presents to the OPD and a diagnosis of fungal keratomycosis is made. Which of the following drugs would be
effective in him?
a. Doxycycline
b. Silver sulfadiazine
c. Vancomycin
d. Linezolid
Solution. Ans-98: (b) Silver sulfadiazine
Ref: H. V. Nema and Nitin Nema’s ‘Recent Advances in Ophthalmology’; Volume 11, pg 16-17
Sol :
TREATMENT OF FUNGAL KERATITIS –
· The treatment for fungal keratitis is usually prolonged.
· Improvement may not be visible for several days.
· Lack of progression of the stromal infiltrate is the first sign that the antifungal drug is effective.
· This is followed by a rounding of the feathery margins, blunting of the perimeters and reduction in cellular infiltrate and edema in
the surrounding stroma.
· Prolonged conjunctival injection, protracted epithelial ulceration, punctate corneal epithelial erosions and diffuse stromal haze
imply
drug toxicity.
· The appearance of a transient hypopyon in the face of an improving corneal picture has been attributed to hypersensitivity or a
toxic
reaction.
· The initiation of treatment can be performed as soon as the results of the KOH mount and Gram’s stain are obtained.
· Studies have questioned the reliability of the positivity of culture as gold standard and have recommended that all smear positive
cases of mycotic keratitis be treated accordingly despite negative culture reports.
· For suspected filamentous fungal keratitis, natamycin 5% drops should be administered hourly for the initial 24 – 48 hours and at
2-
hour intervals.
· The dosage can then be gradually reduced depending up on the response.
· If natamycin is unavailable, then 1% econazole or 1% miconazole can be used.
· For yeast keratitis, topical amphotericin B 0.1% to 0.25% drops are initially applied at 15-minute intervals for the first 24 – 48 hours
and then the frequency of application is gradually reduced for the next 4 – 6 days.
· If initial therapy was topical natamycin or amhotericin B alone and the stromal suppuration is progressing, oral ketoconazole or
fluconazole should be added for either filamentous fungal or yeast keratitis.
· Subconjunctival miconazole (5 mg) or fluconazole (1 mg) can be given.
· Prolonged treatment with systemic antifungals is reserved for deep keratitis associated with scleritis and endophthalmitis.
· The most frequently used oral antifungal has been ketoconazole.
· Recent evidence suggests that fluconazole may penetrate better into the cornea after systemic administration than some other
azoles and associated with fewer side effects.
· The other drugs that have been tried for the treatment of fungal keratitis in past include silver sulfadiazine and sulfacetamide.
· The issue of synergy or antagonism when the antifungal drugs are combined has given varying results.
· A combination of flucytosine and miconazole or natamycin was found to be synergistic in vitro against Aspergillus keratitis.
· Certain investigators routinely use flucytosine topical (1% solution) and systemic (oral 150 mg/kg) in combination with amphotericin
B in the management of Candida keratitis.
· Antagonism has been reported when amphotericin B and the imidazoles have been used systemically.
· Supplementary therapy with keratomycosis includes a cycloplegic-mydriatic agent, such as atropine 1% twice a day; and topical
beta
blocker or oral carbonic anhydrase inhibitor to control secondary ocular hypertension.
Corticosteroids do not have any role in the treatment regimen for fungal keratitis and is not recommended in any stage of the
disease
Your Answer. b
Correct Answer. b
Copyright © 2014 Delhi Academy of Medical Sciences, All Rights Reserved. 97/146
(99). Endoscopic surgery through Intranasal approach is used for surgery of all organs except
a. Lacrimal gland
b. Cerebellum
c. Pituitary gland
d. Optic nerve
Solution. Ans-99: (b) Cerebellum
Ref:Dhingra 4th ed pg 373
Sol:
Indications
1. Chronic bacterial sinusitis unresponsive to adequate medical treatment.
2. Recurrent acute bacterial sinusitis.
3 . Polypoid rhinosinusitis (diffuse nasal polyposis) .
4. Fungal sinusitis with fung<11 ball or nasal polypi.
5. Antrochnanal polyp.
6. Mucocele of frontoethmoid or sphenoid sinus.
7. Control of epistaxis is by endoscopic cautery.
8. Removal of foreign body from the nose or sinus.
9. Endoscopic se ptoplasty.
Advanced Nasal Endoscopic Techniques
1. Removal of benign tumours, e.g. inverted papillomas or angiofibromas.
2. Orbital abscess or cellulitis management.
3. Dacryocystorhinostomy.
4 Repair of CSF leak.
5. Pituitary surgery.
6. Optic nerve decompress ion.
7. Orbital decompression for Graves disease.
8. Control of posterior epistaxis (endoscopic clipping of sphenopalatine artery).
9. Choanal atresia.
Your Answer. b
Correct Answer. b
(100). Cochlear implant which is true
a. Not Contraindicated in cochlear malformation
b. Contraindicated in children< 5yr of age
c. Indicated in mild to moderate hearing loss
d. Port is inserted through oval window
Solution. Ans-100: (a) Not Contraindicated in cochlear malformation
Ref:Read the text below
Sol:
Criteria for adults
(a) Age: 18 years or more
(b) Bilateral severe to profound hearing loss
(c) Limited or no benefit from hearing aids
(d) No medical contraindication to undergo surgery.
· Post- lingually deafened adults are good candidates.Results are not so good in pre-lingually deaf adults unless they had been
receiving aural-oral training for communication .
Criteria for children.
· Auditory deprivation , i.e. lack of auditory stimulus in the early developmental period causes degenerative changes in central
auditory pathways. An early auditory stimulation by cochlear implants can prevent it. Now implants can be used at an age as early
as 12
months. implants can be used in both pre-lingualy or postlingually deafened infants and children. The criteria are:
(a) Bilateral profound or severe to profound hearing Loss
(b) Minimal or no benefit from hearing aid
(c) No medical contraindication
(d) Willingness and support of family to enrol the child for post-implant training programme
· Intracochlear placement of the electrode(s) into the scala tympani is superior to extracochlear placement over the promontory or
round window .
Your Answer. d
Correct Answer. a
Copyright © 2014 Delhi Academy of Medical Sciences, All Rights Reserved. 98/146
(101). CSF rhinorrhea treatment of choice
a. Putting swab in nostril
b. Craniotomy
c. Advising frequent blowing of nose
d. Wait and watch for 7 days and start antibiotics
Solution. Ans-101: (d) Wait and watch for 7 days and start antibiotics
Ref:Dhingra 4th ed pg 155
Sol:
· Early cases of post-traumatic CSF rhinorrhoea are managed conservatively by placing the patient in the semi-sitting position,
avoiding blowing of nose, sneezing and straining. Prophylactic antibiotics are also administered to prevent meningitis.
· Persistent cases of CSF rhinorrhoea are treated surgically by nasal endoscopic or intracranial approach. Nasal endoscopic
approach
is useful for leaks from the frontalsinus, cribriform plate, ethmoid or sphenoid sinuses
Your Answer. d
Correct Answer. d
Copyright © 2014 Delhi Academy of Medical Sciences, All Rights Reserved. 99/146
(102). Regarding retropharyngeal abscess all are true except
a. Lies only on one side
b. Presents behind prevertebral fascia
c. Surgical drainage is required
d. Can be palpated by inserting finger in mouth
Solution. Ans-102: (b) Presents behind prevertebral fascia
Ref:PL Dhingra 4th ed pg 247
Sol:
Retropharyngeal space.
· It lies behind the pharynx between the buccopharyngeal fascia covering pharyngeal constrictor musc les and the pre vertebral
fascia.
· It extends from the base of skull to the bifurcation of trachea.
· The space is divid ed into two late ral compartments (spaces of Gillette) by a fibrous raphe .
· Each lateral space contains retropharyngeal nodes which usually disappear at 3-4 years of age.
· Parapharyngeal space communicates with the retropharyngea l space. Infection of retropharyngeal space can pass down behind
the
oesophagus into the mediastinum .
Prevertebral space.
· It lies between the vertebral bodies posteriorly and the prevertebral fascia anteriorly. It extends from the base to skull of coccyx.
· Infection of this space usually comes from the caries of spine.
· Abscess of this space produces a midline bulge in contrast to abscess of retropharyngeal space which causes unilateral bulge.
ACUTE RETROPHARYNGEAL ABSCESS
Aetiology
· It is commonly seen in children below 3 years. It is the result of suppuration of retropharyngeal lymph nodes secondary to infection
in the ade noids , nasopharynx, posterior nasal sinuses or nasal cavity.
· In adults, it may result from penetrating injury of posterior pharyngeal wall or cervical oesophagus.
· Rarely, pus from acute mastoiditis tracks along the undersurface of petrous bone to present as retropharyngeal abscess.
Clinical Features
1. Dysphagia and difficulty in breathing are prominent symptoms as the abscess obstructs the air and food passages.
2. Stridor and croupy cough may be present.
3. Tonicollis. The neck becomes stiff and the head is kept extended.
4. Bulge in posterior pharyngeal wall. Usually seen on one side of the midline .
Radiograph of soft tissue lateral view of the neck shows widening of prevertebral shadow and poss ibly even the presence of gas
Treatment
· Incision and drainage of abscess . This is usually done without anaesthesia as there is risk of rupture of abscess during intubation.
Child is kept supine with head low. Mouth is opened with a gag. A vertical incision IS given in the m.ost fluctuant area of the
abscess.
Suction should always be available to prevent aspiration of pus.
· Systemic antibiotics . Suitable antibiotics are given.
· Tracheostomy. A large abscess may cause mechanical obstruction to the airway or lead to la ryngeal oedema.
CHRONIC RETROPHARYNGEAL ABSCESS
Cl inical Features
Patient may complain of discomfort in throat. Dysphagia,though present, is not marked. Posterior pharyngeal wallshows a fluctuant
swelling centrally or on one side ofmidline. neck may show tuberculous lymph nodes. Incases with caries of cervical spine, X-rays
are
diagnostic
Your Answer. b
Correct Answer. b
Copyright © 2014 Delhi Academy of Medical Sciences, All Rights Reserved. 100/146
(103). 40 year old man met motor vehicle catastrophe came to hospital in an hour severe maxillo facial trauma PR 120 / min BP -
100 / 70 mm
hg sPo2 - 80% with oxygen . which was the immediate management required
a. Intravenous acess
b. Orotracheal intubation
c. Naso tracheal intubation
d. Tracheostomy
Solution. Ans-103: (d) Tracheostomy
Ref: Sabiston Textbook of Surgery, 18th ed
Sol:
· After all critical issues in the primary survey have been addressed, a full head-to-toe secondary survey is undertaken, with the goal
of carefully examining the entire patient and identifying all injuries. The primary survey is conducted according to the mnemonic
ABCDE:
airway, breathing, circulation, disability, exposure.
Airway
· The crucial first step in managing an injured patient is securing an adequate airway.
· Mechanical removal of debris and the chin-lift or jaw-thrust maneuver, both of which pull the tongue and oral musculature forward
from the pharynx, are often useful in clearing the airway of less severely injured patients.
· However, if there is any question about the adequacy of the airway, if there is evidence of severe head injury, or if the patient is in
profound shock, more definitive airway control is necessary and appropriate.
· In the majority of patients this is accomplished by endotracheal intubation.
· Endotracheal intubation must be done rapidly, under the assumption of cervical spine instability, and in a fashion that does not
induce increased intracranial pressure (ICP) in patients with head injury.
· Intubation is best accomplished by a technique borrowed from surgical anesthesia, known as rapid-sequence induction.
· In rapid-sequence induction the patient is given a fast-acting anesthetic agent, followed by a neuromuscular blocking agent.
· This combination of deep sedation and muscular relaxation allows careful intubation without cervical hyperextension and with
minimal physiologic impact.
· The technique can be used with a number of different pharmacologic agents, depending on the knowledge and preferences of the
individual practitioner. It is incumbent on the individual responsible for the procedure to be fully aware of the dosage, risks, and
indications associated with the agents chosen.
· Excessive ventilation must be avoided after intubation, particularly in a hypovolemic patient because excessive ventilation will
increase mean intrathoracic pressure and compromise cardiac filling.
· Although nasotracheal intubation had been widely suggested as a central modality for emergency airway control in the past, it
should now be used only rarely in the initial management of an injured patient.
· Nasotracheal intubation has a number of drawbacks, and the goal of safe endotracheal intubation with cervical spine precautions
can better be accomplished by orotracheal intubation after rapid-sequence induction.
· In a few patients endotracheal intubation is either impractical or impossible, and a surgical airway is required.
· Indications for a surgical airway include massive maxillofacial trauma, anatomic distortion as a result of neck injury, and inability to
visualize the vocal cords because of the presence of blood, secretions, or airway edema.
· Cricothyroidotomy is the preferred emergency procedure in the majority of circumstances.
· Actual tracheotomy may be indicated in select patients, such as those with laryngeal injuries.
Either surgical procedure may be preceded by needle cricothyroidotomy with jet insufflation to improve oxygenation and allow the
surgical procedure to be performed in more orderly fashion.
Your Answer. c
Correct Answer. d
(104). What is the reason that an infant can breathe while suckling breast milk
a. Small wide tongue
b. High place larynx
c. Small pharynx
d. Short soft palate
Solution. Ans-104: (b) High place larynx
Ref:Read the text below
Sol:
· The newborn’s larynx is located so much higher in its throat than that of an adult that the opening (covered by the epiglottis) is
above the stream of milk and the baby can breathe and eat at the same time without choking.
· At about three years of age, the larynx moves down into the throat along with the back of the tongue which is fastened to it
Your Answer. b
Correct Answer. b
Copyright © 2014 Delhi Academy of Medical Sciences, All Rights Reserved. 101/146
(105). Perineural invasions seen in
a. Adenocarcinoma
b. Adenoid cystic carcinoma
c. BCC
d. Squamous.Cell Carcinoma
Solution. Ans-105: (b) Adenoid cystic carcinoma
Ref:Textbook of surgery by Joe J. Tjandra 3rd ed pg 33
Sol:
Adenoid cystic carcinoma:
· Adenoid cystic carcinoma, formerly known as cylindromas, also present with a slowly growing asymptomatic parotid mass.
· These tumours are characterised by early perineural spread and have a propensity for late recurrence, often to bone or lung,even
up
to 20 years following an apparent cure.
Your Answer. b
Correct Answer. b
(106). Cock’s peculiar tumor is
a. Cylindroma
b. BCC
c. Infected sebaceous cyst
d. SCC
Solution. Ans-106: (c) Infected sebaceous cyst
Ref:Read the text below
Sol:
· Cock's peculiar tumour is a sebaceous cyst linked growth.
· One form of sebaceous cyst is the trichilemmal cyst; these are thought to arise from the hair follicle epithelium and are most
commonly found on the scalp.
· Usually they are multiple but occasionally there can be a proliferating trichilemmal cyst which grows to a large size and ulcerates.
· These can resemble a squamous cell carcinoma (both clinically and histologically) and these ulcerating solitary cysts are called
Cock's peculiar tumour.
· Chronic inflammation causes the cyst to take the form of a granuloma.
· This granuloma mimicks a squamous cell tumor. Hence it is a misnomer.
· It is not a tumor but looks like a tumor. The most common sites are the ones where one can find hairs. these are, scalp and
scrotum.
Your Answer. c
Correct Answer. c
(107). A patient has a damaged pelvic outlet as the result of an automobile accident. Which of the following structures is intact?
a. Sacrotuberous ligament
b. Inferior pubic ramus
c. Pubic crest
d. Ischial tuberosity
Solution. Ans-107: (c) Pubic crest
Ref:Read the text below
Sol:
· Although a pubic test crest forms a part of the pelvic inlet (pelvic brim), it does not contribute to the formation of the pubic outlet.
The pelvic outlet(lower pelvic aperture) is bounded posteriorly by the sacrum and coccyx; laterally by the ischial tuberosities and
sacrotuberous ligaments; and anteriorly by the pubic symphysis, the accurate ligament, and the rami of the pubis and ischium
Your Answer. c
Correct Answer. c
Copyright © 2014 Delhi Academy of Medical Sciences, All Rights Reserved. 102/146
(108). Pulmonary artery banding in children is indicated in which of the following conditions ?
a. Large ventricular septal defect
b. Coarctation of the aorta
c. “Pink” tetralogy of fallot
d. Distal pulmonary atresia
Solution. Ans:108: (a) Large ventricular septal defect
Ref:Read the text below
Sol:
§ High volume left to right shunt can give refractory congestive failure in patients too ill or too small to undergo total surgical
correction.
§ Temporary palliation may be achieved by diminishing pulmonary artery pressure through a constriction of the pulmonary trunk.
§ This would hardly be necessary in a patient who already had an atretic segment of pulmonary artery.
§ The patient with pink tetralogy is manifesting compensation whereby congestive failure is not prominent from left to right shunt.
§ In the classic operation for cyanotic tetralogy, pulmonary blood flow and pressure was actually increased to early systolic systemic
levels by creating a fistula through the subclavian artery diversion (Blalock-Taussig) or aortopulmonary window anastomosis
(Waterston).
This is the reverse of the pressure dynamics attempted with a pulmonary banding procedure
Your Answer. c
Correct Answer. a
(109). 70 yr old patient presents with c/o difficulty in micturition since 10 yrs. He Presents with B/L hydrouretronephrosis, full bladder
&
prostate 70g, In addition to the above he had 400ml residual volume of urine on USG. BUN was 43.8 creatinine was 2.1. Immediate
intervention to be done
a. MRI pelvis
b. Contrast enhanced CT for Carcinoma
c. Insertion of foley's catheter
d. B/L ureterostomies
Solution. Ans-109: (c) Insertion of foley's catheter
Ref: Sabiston Textbook of Surgery, 18th ed
Sol:
· If the urinary retention has lasted several days (often accompanied by overflow incontinence), patients may be in renal failure.
· Treatment should be the placement of a urethral catheter as quickly as possible. However, BPH or urethral strictures often make
the
placement of a catheter difficult.
· For men with BPH, a coude (French for curved) catheter is helpful in negotiating past the angulation in the prostatic urethra
· The curved portion (which is angled in line with the balloon port) is maintained at the 12 o'clock position as it is passed through the
urethra
Your Answer. c
Correct Answer. c
Copyright © 2014 Delhi Academy of Medical Sciences, All Rights Reserved. 103/146
(110). 45 yr old man after heavy work and travel complains of passing fresh blood PR for 7 days. No significant findings on PR
examination and
no other past history. What would you recommend next for the man
a. Proctoscopy
b. Refer him to a surgeon for sigmoidoscopy
c. Refer him to a gastroenterologist for colonoscopy
d. Barium enema
Solution. Ans-110: (a) Proctoscopy
Ref:Read the text below
Sol:
· After resuscitation has been initiated, the first step in the workup is to rule out anorectal bleeding with a digital rectal exam and
anoscopy or sigmoidoscopy.
· With significant bleeding, it is also important to eliminate an upper GI source. An NG aspirate that contains bile and no blood
effectively rules out upper tract bleeding in most patients. However, when emergent surgery for life-threatening hemorrhage is being
contemplated, preoperative or intraoperative EGD is usually appropriate.
Colonoscopy is most appropriate in the setting of minimal to moderate bleeding; major hemorrhage interferes significantly with
visualization, and the diagnostic yield is low. In addition, in the unstable patient, sedation and manipulation may be associated with
additional complications and can interfere with resuscitation
Your Answer. a
Correct Answer. a
(111). Each of the following constitutes a risk factor for breast cancer except :
a. Heredity
b. Fibrocystic disease
c. Nulliparity
d. Prolonged breast feeding
Solution. Ans:111: (d) Prolonged breast feeding
Ref:Read the text below
Sol:
§ Earlier prior breast cancer in the patient or first order female relatives increases the risk of breast cancer as does prolonged
estrogen
stimulation as seen with nulliparity.
§ Dysplastic changes in fibrocystic disease may be precursors of neoplasia.
§ Prolonged breast feeding interrupts estrogen stimulation and is actually protective against the development of breast cancer.
Your Answer. d
Correct Answer. d
Copyright © 2014 Delhi Academy of Medical Sciences, All Rights Reserved. 104/146
(112). Most appropriate reason for TURP
a. Prostate wt. 70gm
b. B/L hydronephrosis
c. Needs 3day course of antibiotic for an episode of urinary tract infection
d. Can not use alpha blocker due to hypertension
Solution. Ans-112: (b) B/L hydronephrosis
Ref: Sabiston Textbook of Surgery, 18th ed, Campbell-Walsh Urology, 9th ed
Sol:
Campbell writes
· The most common reasons for recommending intervention in a patient with symptoms of bladder outlet obstruction and irritability
are that the symptoms are moderate to severe, bothersome, and interfere with the patient's quality of life
· Although symptoms constitute the primary reason for recommending intervention, in patients with an obstructing prostate there are
some absolute indications. These are acute urinary retention, recurrent infection, recurrent hematuria, and azotemia
Sabiston writes
· Transurethral resection of prostate (TURP) is a proven surgical technique that significantly improves lower urinary tract symptoms
associated with BPH.
· This intervention is most commonly recommended in the patient with symptoms of bladder outlet obstruction and irritability that are
moderate to severe, bothersome, and interfere with the patient's quality of life.
· Although symptoms constitute the primary reason for recommending intervention, in patients with an obstructing prostate, there
are
some absolute indications.
These are acute urinary retention, recurrent infection, recurrent hematuria, and azotemia
Your Answer. b
Correct Answer. b
Copyright © 2014 Delhi Academy of Medical Sciences, All Rights Reserved. 105/146
(113). Brilliantly translucent is all except
a. Vaginal hydrocele
b. Cystic hygroma
c. Meningocele
d. Lipoma
Solution. Ans-113: (d) Lipoma
Ref:Read the text below
Sol:
Cysts of the neck
Midline cyst
1. Submental dermoid cyst.
2. Thyroglossal cyst.
3. Subhyoid bursitis.
4. Cyst in the thyroid isthmus.
5. Cold abscess in submental or prelaryngeal l.n.
6. Laryngeocele.
7. Suprasternal dermoid.
Lateral cyst
1. Cystic hygroma.
2. Plunging ranula.
3. Branchial cyst.
4. Cyst in thyroid lobe.
5. Cold abscess.
6. Pharyngeal pouch.
7. Pneumatocele.
Branchial cyst:
It arises from vestigial remnant of the 2nd branchial cleft. The cyst contains mucoid fluid with many cholesterol crystals. The wall is
lined
with epithelial tissue and contains lymphoid tissue.
· Clinical picture
Usually at the age of 20 years as a slowly growing painless swelling at the upper lateral part of the neck between upper and middle
third
which is partially undercover the anterior border of the sternomastoid. It transilluminates.
· Complications: infection which is treated by antibiotics - branchial fistula - branchiogenic carcinoma.
· Treatment by complete excision reaching the deep structures near the external auditory meatus.
N.B.: the branchial fistula may be congenital due to persistence of 2nd branchial cleft. It occurs at the lower 1/3 of the anterior border
of
the sternomastoid. The inner opening is just behind the tunsil or may be blind (sinus). It may also be acquired due to infection or
incomplete removal of a branchial cyst. It discharge mucoid or mucopurulent material. It is treated by complete excision via
transverse
neck incision.
Laryngeocele:
· Occurs in wind instruments players due to herniation of the laryngeal mucous membrane through the thyrohyoid membrane. It
forms
a tense cystic resonant translucent swelling which is compressible and enlarge when the patient blow and close the nose.
Cystic hygroma:
· Due to congenital lymphatic anomaly. Where the lymphatic spaces have afferent without efferent vessels.
· It appears at birth commonly at the lower part of the neck in the posterior triangle but it may arise in the groin, mediastinum or
axilla.
· It is brilliantly transleuscent cystic swelling which is partially compressible.
Complications: infection, respiratory obstruction, obstructed labour.
Treatment: better at 2 years by excision but if causing respiratory obstruction aspiration is done if failed tracheostomy.
Your Answer. d
Correct Answer. d
(114). A 14 year-old boy enters the emergency room with bright red blood at the urethral meatus. He is not forthcoming with any
history, and
his midstream urinalysis is unremarkable as are other preliminary laboratory studies. A likely diagnosis might be
a. Prostatitis
b. Instrumentation
c. Mumps
d. Syphilis
Solution. Ans:114: (b) Instrumentation
Ref:Read the text below
Sol:
§ There is no evidence for urinary tract pathology beyond the urethra, and the clinical context is that often seen with manipulation
and
fear of consequences.
§ Mumps would have acute orchitis and the inflammatory conditions of epididymitis or prostatitis are not likely to give a negative
physical exam and blood work.
§ This is not the presentation of chancre, either, so the patient should be invited to talk about his fears to the limit that he is willing to
do
so over a follow-up observation.
Your Answer. a
Correct Answer. b
Copyright © 2014 Delhi Academy of Medical Sciences, All Rights Reserved. 106/146
(115). Each of the following treatment is acceptable in managing benign prostatic hypertrophy except:
a. Proscar (Fenastamide)
b. Transurethral resection
c. Suprapubic prostatectomy
d. Perineal prostatectomy with interstitial radiation therapy
Solution. Ans:115: (d) Perineal prostatectomy with interstitial radiation therapy
Ref:Read the text below
Sol:
§ Benign prostatic hypertrophy’s management is principally designed to relieve urinary tract obstruction symptoms and also to rule
out
the presence of malignant prostate disease.
§ If there is no obstruction, observation can be safely followed after biopsy confirmation in some age groups that no malignant
prostate
component is present.
§ Open prostatectomy was the standard before wide-scale application of transurethral resection techniques, but now a new therapy
has
been made available in the form of an enzyme inhibitor that shrinks benign prostatic hypertrophy and may postpone or eliminate the
requirement for operations designed to relieve obstruction.
§ This tendency toward more benign and less invasive procedures for benign prostatic hypertrophy would preclude a radical
perineal
prostatectomy, and no interstitial radiation implant source would be required for the benign disease.
Your Answer. d
Correct Answer. d
(116). Newer techniques alternative to pyelolithotomy include all the following except :
a. Basketing the stone for retrieval by ureteral catheter
b. ESWL lithotripsy
c. Chemically dissolving calcium oxalate stones by infusion into the kidney
d. Pulverizing the stone by laser or shock energy introduced through ureteral catheterization
Solution. Ans:116: (c) Chemically dissolving calcium oxalate stones by infusion into the kidney
Ref:Read the text below
Sol:
§ Although lesser invasive procedures have been developed for kidney stone removal, most have involved cystoscopic manipulation
of
ureteral catheters to retrieve stones or fragments or pulverizing the stone through extracorporeal shock wave lithotripsy or direct
contact with the stone through the ureteral approach.
§ As yet, no available chemical dissolves the calcific stone without causing damage to the surrounding tissue that would preclude its
use.
Your Answer. c
Correct Answer. c
Copyright © 2014 Delhi Academy of Medical Sciences, All Rights Reserved. 107/146
(117). The most frequently performed procedure for coronary insufficiency currently is
a. Internal mammary artery bypass
b. Coronary endarterctomy
c. Coronary thrombectomy
d. Aortocoronary bypass graft
e. Percutaneous coronary angioplasty
Solution. Ans:117: (d) Aortocoronary bypass graft
Ref:Read the text below
Sol:
§ Thrombectomy for coronary artery thrombosis is as temporary as coronary endarterectomy, both of which do not improve blood
flow
long-term and, hence, myocardial performance.
§ The Vineberg internal mammary artery bypass was useful when originally proposed and remains so for selected indications, but
the
majority of patients today undergo bypass grafting from aorta to coronary arteries through reversed saphenous veins or some other
conduit than the internal mammary artery.
§ Percutaneous transcatheter coronary angioplasty might be applicable to patients with isolated lesions, but most coronary
altherosclerosis is generalized with not just a single focal significant stenotic lesion, so even patients with coronary artery bypass
graft
from aortas, which is the most frequently performed operation.
Your Answer. e
Correct Answer. d
(118). Which of the following is not a absorbable sutures
a. Catgut
b. Silk
c. Polyglyconate (Maxon)
d. Polyglactin (Vicryl)
Solution. Ans-118: (b) Silk
Ref:Read the text below
Sol:
· Silk is non absorbable suture.
· Catgut is broken by proteolysis while Vicryl by hydrolysis.
· Polyglconate is also absorbable but does not retain enough tensile strength.
Your Answer. b
Correct Answer. b
Copyright © 2014 Delhi Academy of Medical Sciences, All Rights Reserved. 108/146
(119). Renal stone resistant to ESWL
a. Cystine stone
b. Calcium oxalate
c. Uric acid
d. Calcium phosphate
Solution. Ans-119: (a) Cystine stone
Ref:Smith’s general urology 17th ed page 253
Sol:
· Cystine lithiasis is secondary to an inborn error of metabolism resulting in abnormal intestinal (small bowel) mucosal absorption
and
renal tubular absorption of dibasic amino acids, including cystine, ornithine, lysine, and arginine.
· The genetic defects associated with cystinuria have now been mapped to chromosome 2p.16 and more recently to 19q13.1.
· Cystine lithiasis is the only clinical manifestation of this defect.
· Classic cystinuria is inherited in an autosomal recessive fashion.
· It represents 1–2% of all urinary stones, with a peak incidence in the second or third decade.
· The solubility of cystine is pH-dependent, with a pK of approximately 8.1. There is no known inhibitor for cystine calculi, and
cystine
stone formation is completely dependent on excessive cystine excretion.
· Cystine stones are frequently associated with calcium calculi and their related metabolic abnormalities. They may present as
single,
multiple, or staghorn stones.
· The diagnosis is suspected in patients with a family history of urinary stones and the radiographic appearance of a faintly opaque,
ground-glass, smooth-edged stone . Urinalysis frequently reveals hexagonal crystals.
· Stone analysis confirms the diagnosis.
· Quantitative urinary cystine evaluation helps confirm the diagnosis and differentiate heterozygous from homozygous states.
· Medical therapy includes high fluid intake (>3 L/day) and urinary alkalinization.
· Patients should monitor their pH with nitrazine indicator paper and keep their pH values >7.5.
· A low-methionine (precursor to cystine) diet has limited impact, as most of the cystine is endogenous and most of the ingested
methionine is incorporated into protein.
· Glutamine, ascorbic acid, and captopril are effective in some patients.
· Penicillamine can reduce urinary cystine levels. It complexes with the amino acid, and this complex is dramatically more soluble.
· Mercaptopropionylglycine, forms a soluble complex with cystine and can reduce stone formation.
· Surgical treatment is similar to that for other stones except that most stones are recalcitrant to extracorporeal (outside the body)
shock wave lithotripsy (ESWL).
One should have a low threshold to proceed with percutaneous stone extraction in symptomatic patients
Your Answer. a
Correct Answer. a
(120). The most clinically significant early physiologic abnormality in post-traumatic pulmonary insufficiency is
a. Increased physiologic shunt
b. Increased dead space
c. Ventilator barotraumas
d. Cardiogenic pulmonary edema
Solution. Ans:120: (a) Increased physiologic shunt
Ref:Read the text below
Sol:
§ Shock lung has increased fluid content, much of it interstitial. This increased fluid volume and decreased gas space has lead to a
greater fraction of the lung perfused but not ventilated which is by definition the physiologic shunt.
§ Pulmonary fibrosis occurs at a very late stage and can inhibit gas exchange and impair compliance.
§ This same alveolar capillary blockade can occur from lung injury by high pressures or oxygen tensions administered to the lung.
§ However, the physiologic basis for the inability of the lung to oxygenate blood adequately is the greater fraction of lung perfused
without effective ventilation, that is, physiologic shunt.
Your Answer. b
Correct Answer. a
Copyright © 2014 Delhi Academy of Medical Sciences, All Rights Reserved. 109/146
(121). Advantages of crystalloid solution over colloid in fluid resuscitation include all of the following except :
a. Cost
b. Availability
c. Retrievability
d. Edema
Solution. Ans:121: (d) Edema
Ref:Read the text below
Sol:
§ Crystalloid solution is readily available at 1/20th of the cost of albumin, and has fewer associated pyrogenic reactions.
§ It is also possible to retrieve infused crystalloid through diuresis in patients with functioning kidneys.
§ It does cross permeable membranes to accumulate in “third space” components of extracellular fluid; whereas, albumin has a
longer
circulating half life in patients tending toward congestive edema.
Your Answer. d
Correct Answer. d
(122). Which of the following characteristics of a patient with thyroid cancer has the most effect on prognosis?
a. Degree of differentiation
b. Tumor stage
c. Lymph node involvement
d. Patient’s age
Solution. Ans:122: (d) Patient’s age
Ref:Read the text below
Sol:
§ The biologic factors predominate in prognosis of thyroid cancers and many of them are “given” rather than controllable.
§ The single most important is the patient’s age, since if all other features were held constant, the younger patient has a better
prognosis.
§ Lymph node involvement, as previously noted, is not a principle factor in changing the staging of tumors of the same size, but size
is a
factor, and staging is principally determined by T-stage with much less contribution from N-staging.
§ Differentiation of the tumor for a given cell type, such as papillary, is a significant prognostic factor for groups of patients, but is not
as
significant as age.
§ Since most of the patients have well differentiated tumors, it is also humbling to note that one prognostic factor absent as a
principle
determinant of how well patients do is treatment, about which each clinician carries such passionate belief that it is difficult to test
treatment options to determine a superior method which we honestly do not know.
Your Answer. b
Correct Answer. d
(123). The ability to manipulate small objects with the pincer grasp is usually noted at what age?
a. 0 to 2 months
b. 3 to 5 months
c. 6 to 7 months
d. 8 to 9 months
Solution. Ans:123: (d) 8 to 9 months
Ref: Nelson Textbook of Pediatrics, 15th ed- Chapter 11
Sol:
§ The pincer grasp, which is noted at age 8 to 9 months, along with increasing mobility, enables an infant to explore the
environment.
Your Answer. d
Correct Answer. d
Copyright © 2014 Delhi Academy of Medical Sciences, All Rights Reserved. 110/146
(124). Visual fixation by newborn infants is associated with which Prechtl behavioral state?
a. Drowsy
b. Quiet, alert
c. Awake, active
d. Crying
Solution. Ans:124: (b) Quiet, alert
Ref: Nelson Textbook of Pediatrics, 15th ed- Chapter 10
Sol:
§ Newborns have been shown to fixate visually during the quiet, alert state. This state represents 10% of a 24-hour day.
Your Answer. b
Correct Answer. b
(125). Salmonella osteomyelitis is a common problem in children with
a. Sickle cell anemia
b. A history of consuming unpasteurized milk
c. Prior antibiotic exposure
d. Sickle cell trait
Solution. Ans:125: (a) Sickle cell anemia
Ref: Nelson Textbook of Pediatrics, 15th ed- Chapter 182.1
Sol:
§ Necrotic areas of bone due to osteonecrosis from local sickling may predispose patients with sickle cell anemia to salmonella
osteomyelitis.
Your Answer. a
Correct Answer. a
(126). Positive fecal leukocyte finding in a child with diarrhea allows the clinician to make a presumptive diagnosis of
a. S. typhi
b. Entamoeba histolytica
c. Colitis
d. None
Solution. Ans:126: (c) colitis
Ref: Nelson Textbook of Pediatrics,171
Sol:
§ Fecal leukocyte examination is a nonspecific test that may suggest lower colonic-rectal inflammation.
Your Answer. c
Correct Answer. c
Copyright © 2014 Delhi Academy of Medical Sciences, All Rights Reserved. 111/146
(127). Oral rehydration therapy is the treatment of choice for all children with gastroenteritis except those with
a. vomiting, high fever
b. ileus, coma, or shock
c. cholera
d. shigellosis
e. poor skin turgor, sunken fontanel, dry mouth, and decreased urine ouput
Solution. Ans:127: (b) ileus, coma, or shock
Ref: Nelson Textbook of Pediatrics, 15th ed- Chapter 171
Sol:
Ileus, coma with a risk of aspiration, shock, and peritionitis (e.g., pneumoperitoneum) are contraindications for oral therapy
Your Answer. e
Correct Answer. b
(128). The metabolic alternation found in infants with pyloric stenosis is
a. hypochloremic acidosis
b. hypochloremic alkalosis
c. hyperchloremic acidosis
d. hyperchloremic alkalosis
Solution. Ans:128: (b) hypochloremic alkalosis
Ref: Nelson Textbook of Pediatrics- Chapter 275
Sol:
§ When an obstruction is proximal to the ampulla of Vater, the vomitus cannot be bile stained, and because most of the vomitus has
stomach (hydrochloric) acid, hypochloremic alkalosis ensues.
Your Answer. b
Correct Answer. b
(129). Persistent jaundice with hypergammaglobulinemia, positive results of a lupus erythematosus cell preparation, anti DNA
antibodies, and
an elevated transaminase value are compatible with which disease ?
a. Hepatitis A
b. Hepatitis B
c. Chronic persistent hepatitis
d. Chronic active hepatitis
Solution. Ans:129: (d) Chronic active hepatitis
Ref: Nelson Textbook of Pediatrics, 15th ed- Chapter 307
Sol:
§ Chronic active hepatitis is hepatitis with chronic portal inflammation that extends into the parenchyma, with piecemeal necrosis
and
fibrosis that usually progresses to a coarsely nodular postnecrotic cirrhosis
Chronic active hepatitis is the more serious form of chronic hepatitis
Your Answer. d
Correct Answer. d
Copyright © 2014 Delhi Academy of Medical Sciences, All Rights Reserved. 112/146
(130). Which of the following statements about Gilles de la Tourette syndrome is true ?
a. It is characterized by tics and coprolalia
b. It is characterized by tics and encopresis
c. It is treated with haloperidol and methylphenidate
d. It is a common disorder of childhood
Solution. Ans:130: (a) It is characterized by tics and coprolalia.
Ref: Nelson Textbook of Pediatrics, - Chapter 22
Sol:
§ Gilles de la Tourette syndrome, which has a lifetime prevalence rate of 0.5 per 1000 individuals, is a rare condition in children.
§ It is characterized by multiple tics, compulsive barking, and shouting obscene words (coprolalia).
§ It is more common in first-degree relatives of patients with Tourette syndrome than in the general population and affects boys
three to
four times more often than girls.
§ The cause is uncertain, but research has shown that drugs that increase dopaminergic action precipitate or worsen both tics and
Gilles
de la Tourette syndrome.
§ Many environmental precipitants have been noted to serve as emotional stress sources.
§ The syndrome can be fairly well managed with haloperidol, a dopaminergic antagonist, and Orap, a powerful dopamine
antagonist.
Anecdotal reports in the literature suggest that the serotonin reuptake inhibitors are also efficacious in its treatment.
Your Answer. Not Attempted
Correct Answer. a
(131). Separation anxiety disorder is most likely to be associated with
a. Aggressive behaviors
b. Poor academic performance
c. School refusal
d. Increased risk of schizophrenic outcome
Solution. Ans:131: (c) School refusal
Ref: Nelson Textbook of Pediatrics, 15th ed- Chapter 23
Sol:
§ Separation anxiety disorder is characterized by unrealistic and persistent worries of possible harm befalling primary caregivers,
reluctance to go to school or to sleep without being near the parents.
§ Persistent avoidance of being alone, nightmares involving themes of separation, and numerous somatic symptoms and complaints
of
subjective distress. Affected children come from middle and lower socioeconomic classes.
§ Slightly more girls than boys are affected; 80% of the affected children are Caucasian. The majority of older children with
separation
anxiety disorder are also said to suffer with an affective disorder.
§ Treatment includes a supportive understanding of the child’s fear, liaison work with the school to help reintroduce the child to the
classroom, and family work, especially helping parents learn how not to reinforce the symptoms.
Benzodiazepines and tricyclics have also been shown to be quite efficatious in reducing symptoms sufficiently to allow a child to
return to
school.
Your Answer. a
Correct Answer. c
(132). Truancy is considered developmentally normal if it is associated with
a. Lying
b. fear of school failure
c. fear of peer ridicule
d. none of the above
Solution. Ans: 132: (d) None of the above
Ref: Nelson Textbook of Pediatrics, 15th ed- Chapter 26
Sol:
§ Truancy and runaway behavior are never developmentally normal.
Your Answer. b
Correct Answer. d
Copyright © 2014 Delhi Academy of Medical Sciences, All Rights Reserved. 113/146
(133). Wenckebach’s AV block is characterised by all of the following except:
a. Progressive prolongation of PR interval followed by a dropped QRS
b. Progressive shortening of RR interval
c. Incremental increments in PR interval
d. Longest RR interval is less than twice the shortest RR interval
Solution. Ans-133: (c) Incremental increments in PR interval
Ref: Read the text below
Sol:
· The increments in PR interval in Wenckebach’s AV block are decremental. This is the reason for the progressive shortening of RR
intervals.
Your Answer. d
Correct Answer. c
(134). Venous Corrigan is seen in:
a. Aortic regurgitation
b. Mitral regurgitation
c. Tricuspid regurgitation
d. Pulmonary regurgitaton
Solution. Ans-134: (c) Tricuspid regurgitation
Ref: Read the text below
Sol:
· Venous Corrigan is the name given to the prominent V wave (also called the CV wave) seen in the jugular venous pulse in
tricuspid
regurgitation.
· Corrigan’s sign is the prominent carotid pulsation seen in aortic regurgitation.
Your Answer. Not Attempted
Correct Answer. c
(135). The most likely diagnosis in the case of a patient with multiple pulmonary cavities, haematuria and red cell casts is :
a. Anti – GBM disease
b. Churg-Strauss allergic granulomatosis
c. Systemic lupus erythematosus
d. Wegner’s granulomatosis
Solution. Ans-135: (d) Wegner’s granulomatosis
Ref.: Nelson - 731.
Sol :
Classical triad of Wegner’s granulomatosis :
§ Upper respiratory tract involvement (sinus and nasopharynx)
§ Lower respiratory tract involvement (bilateral mulitiple, nodular cavitary pulmonary infiltrates)
§ Kidney disease (77%) – proteinuria, hematuria, red blood cell casts.
§ Characteristic lab findings –ESR, mild anemia, leukocytosis, mild hypergammaglobulinemia (particularly IgA and mildly elevated
rheumatic factor)
Your Answer. a
Correct Answer. d
Copyright © 2014 Delhi Academy of Medical Sciences, All Rights Reserved. 114/146
(136). All are seen in Parkinson’s disease except
a. Resting tremors
b. Bradykinesia
c. Rigidity
d. Preserved postural reflexes
Solution. Ans-136: (d) Preserved postural reflexes
Ref: Read the text below
Sol :
Pakinson’s Disease :
§ 4 cardinal features
Parkinson’s tremor
§ Rest tremors
§ Absent during action
§ Slow frequency (4-7 Hz)
§ Pill rolling tremors in some
§ Postural tremors (8-10 Hz) ; persists on
movement)
Rigidity : Lead pipe or cog wheel rigidity
Bradykinesia
§ Mask like face, drooling of saliva, reduced blinking, reduced arm swing
§ Small and spidery hand writing (micrographia)
Postural instability
§ Postural reflexes lost
§ Festinating gait (rapid small steps) difficulty in stopping suddenly and impaired
balance on turning.
Your Answer. d
Correct Answer. d
(137). True about Wallenberg’s syndrome
a. Ipsilateral pain and numbness of face
b. Contralateral ataxia
c. Contralateral Horner’s syndrome
d. Ipsilateral loss of temperature sense
Solution. Ans-137: (a) Ipsilateral pain and numbness of face
Ref: Read the text below
Sol :
Wallenberg syndrome/lateral medullary syndrome
§ Due to infarction of lateral medulla :
§ Caused by occlusion of any of the five vessels : vertebral artery (MC), posterior inferior cerebellar artery (PICA), superior, middle
or
inferior lateral medullary arteries.
On the side of lesion Opposite side of lesion
§ Pain and numbness of one-half of face (5th N nucleus)
§ Nystagmus, oscillopsia, diplopia, vertigo, nausea, vomiting (8th N
nucleus)
§ Dysphagia, hoarseness, paralysis of palate and vocal cord (9th and
10th N)
§ Ataxia, fall on the side of lesion
§ Loss of taste (NTS)
§ Horner’s syndrome (sympathetic chain)
§ Numbness of ipsilateral arm and trunk (cuneate and gracile nuclei)
§ Weakness of lower face (Genuflected UMN fibers to ipsilateral
facial nucleus)
§ Impaired pain and temperature sense over half due to
spinothalamic tract involvement
Your Answer. b
Correct Answer. a
Copyright © 2014 Delhi Academy of Medical Sciences, All Rights Reserved. 115/146
(138). Risk factors of Alzheimer’s disease all of the following except:
a. Down’s syndrome
b. Smoking
c. Females
d. Low educational status
Solution. Ans-138: (b) Smoking
Ref: Read the text below
Sol :
Alzheimer’s disease :
§ FACTORS ASSOCIATED WITH INCREASED RISK
§ Old age (20-40% of the population over the age of 85)
§ Positive family history
§ Female sex
§ Low level of education
§ Diabetes mellitus
§ Apo E4 § Down’s syndrome
§ Increased production of Aβ peptide
§ Mutations in the gene encoding presinilin-1 and presinilin-2
Your Answer. c
Correct Answer. b
(139). True about Myasthenia gravis all of the following except
a. Sensory loss
b. Smooth muscles not involved
c. Normal tendon reflexes
d. Ocular muscle involvement
Solution. Ans-139: (a) Sensory loss
Ref: Read the text below
Sol :
Myasthenia Gravis :
§ Autoimmune disorder mediated by specific anti – ACH receptor antibodies at post synaptic membrane
§ Women more affected
§ Peaks of incidence occur in women in their twenties and thirties and in men in their fifties and sixties clinical features.
§ The cardinal features are weakness and fatigability of muscles
§ Fluctuating weakness of voluntary muscles which increases on repeated use ; improves after rest
§ The lids and extraocular muscles, are involved early; diplopia and ptosis are common initial complaints
§ Facial weakness produces a snarling expression when the patient attempts to smile
§ Speech may have a dysarthric mushy quality due to tongue weakness
§ Bulbar weakness is especially prominent in Musk (muscle specific kinase) antibody-positive MG
§ If weakness remains restricted to the extraocular muscles for 3 years (ocular MG), it is likely that it will not become generalized
§ The limb weakness is often proximal and may be asymmetric
§ Myasthenic crisis : exacerbation of weakness sufficient to endanger life; it usually consists of respiratory failure caused by
diaphragmatic and intercostals muscle weakness.
§ Not affected/involved :
§ Smooth muscles and cardiac muscles
§ Tendon reflexes
Sensory system
Your Answer. a
Correct Answer. a
Copyright © 2014 Delhi Academy of Medical Sciences, All Rights Reserved. 116/146
(140). In ischemic stroke thrombolysis should be started within-hrs for maximum benefit
a. 1 hr
b. 3 hrs
c. 6 hrs
d. 12 hrs.
Solution. Ans-140: (b) 3 hrs
Ref: Read the text below
Sol :
§ Recombinant Tissue plasminogen Activator (rtPA) in acute ischemic stroke :
§ The risk of fatal intracranial h’ge but ther is improvement in long – term outcome amongst survivors
§ The maximum benefit occurs when thrombolysis is given within 3 hours of onset.
Your Answer. a
Correct Answer. b
(141). Intentional tremor is characteristic of lesion at
a. Pons
b. Cerebellum
c. Spinal cord
d. Basal ganglia
Solution. Ans-141: (b) Cerebellum
Ref: Read the text below
Sol :
Features of cerebellar dysfunction
§ Dysmetria
§ Ataxia
§ Pastpointing
§ Dysdiadokinesia
§ Scanning Dysarthria
§ Intentional tremors
§ Cerebellar nystagmus
§ Hypotonia
Your Answer. b
Correct Answer. b
(142). Hyperventilation can provoke the following seizure
a. Grand mal epilepsy
b. Petitimal epilepsy
c. Myoclonic seizures
d. Partial seizure
Solution. Ans-142: (b) Petitimal epilepsy
Ref: Read the text below
Sol :
· Typical Petitimal epilepsy is easily induced by hyperventilation in more than 90% of the patients.
· This is a reliable test for the diagnosis of absence seizures: a patient suspected of typical absences should be asked to
overbreathe
for 3 min, counting his or her breaths. Intermittent photic stimulation may precipitate or facilitate absence seizures; eyelid myoclonia
is a
common clinical feature.
· A specific mechanism difference exists in absence seizures in that T-type Ca++ channels are believed to be involved.
· Ethosuximide is specific for these channels and thus it is not effective for treating other types of seizure.
· Valproic acid andgabapentin (among others) have multiple mechanisms of action including blockade of T-type Ca++ channels and
are useful in treating multiple seizure types.
Your Answer. c
Correct Answer. b
Copyright © 2014 Delhi Academy of Medical Sciences, All Rights Reserved. 117/146
(143). Not a feature of Millard Gubler syndrome
a. Lesion at Ponto-medullary junction
b. Ipsilateral 6th nerve palsy
c. Ipsilateral LMN facial palsy
d. Ipsilateral hemiplegia
Solution. Ans-143: (d) Ipsilateral hemiplegia
Ref: Read the text below
Sol :
Syndrome Site of lesion Clinical features
Weber Anterior cerebral peduncle (midbrain)
Ipsilateral 3rd N palsy
Contralateral UMN7th N palsy
Contralateral hemiplegia
Claude Cerebral peduncle involving red nucleus Ipsilateral 3rd N palsy
Contralateral cerebellar signs
Perinaud Dorsal midbrain (tectum)
Vertical gaze palsy
Convergence disorders
Convergence retraction nystagmus
Papillary & lid disorders
Millard – Gubler Pontomedullary Junction
Ipsilateral 6th N palsy
Ipsilateral LMN 7th N Palsy
Contralateral hemiplegia
Wallenberg Lateral medulla
Ipsilateral 5th, 9th , 10th 11th palsy
Ipsilateral Horner’s syndrome
Ipsilateral cerebellar signs
Contralateral spinothalamic sensory loss
Vestibular disturbance
Your Answer. Not Attempted
Correct Answer. d
(144). Most common cardiac manifestation of SLE
a. Libman Sack’s endocarditis
b. Pericarditis
c. Myocarditis
d. Valvular insufficiency
Solution. Ans-144: (b) Pericarditis
Ref: Read the text below
Sol :
§ Systemic Lupus Erythematosus
§ More common in women
Antibody Clinical utility
Anti nuclear (ANA) Best screening test (98% prevalence)
Anti ds DNA SLE specific, correlate with disease activity, nephritis, vasculitis
Anti – Smith Specific, no clinical correlation
Anti histone Ab Drug induced lupus
Anti – Ros (SS-A) Predisposes to Subacute cutaneous lupus, neonatal lupus with congenital heart block; Decreased
risk of nephritis (also anti – La)
Anti Ribosomal Ab Psychosis due to CNS lupus
Anti – neuronal (Anti-glutamate
receptor) Correlates with CNS lupus activity
Antiphospholipid Fetal loss, clotting, thrombocytopenia
Your Answer. a
Correct Answer. b
Copyright © 2014 Delhi Academy of Medical Sciences, All Rights Reserved. 118/146
(145). N-acetyl penicillamine is used in the treatment of poisoning by :
a. Mercury
b. Lead
c. Cadmium
d. Arsenic
Solution. Ans-145: (a) Mercury
Ref.: Read the text below
Sol :
Penicillamine :
§ Drug of choice for Cu poisoning.
§ Alternative drug to dimercaptrol for Hg poisoning.
May be used as an adjuvant to CaNa2 EDTA in chronic lead poisoning
Your Answer. b
Correct Answer. a
(146). HLA B27 is usually detected in the following conditions except :
a. Systemic lupus erythematosus
b. Ankylosing spondylitis
c. Psoriatic arthritis
d. Reactive arthritis
Solution. Ans-146: (a) Systemic lupus erythematosus
Ref.: Harrison’s - 2026
Sol :
HLA B27
· Ankylosing spondylitis
· Reiter’s symdrome
· Reactive arthritis (Yersinia, Salmonella, Gonococcus)
· Psoriatic arthritis
· Acute anterior uveitis
· Juvenile rheumatoid arthritis.
Your Answer. a
Correct Answer. a
(147). The use of which one of the following may cause cardiomyopathy?
a. Adriamycin
b. Carboplatin
c. Dactinomycin
d. Etoposide
Solution. Ans-147: (a) Adriamycin
Ref.: Read the text below
Sol :
Drugs causing cardiomyogpathy
· Sympathomimetics. Doxorubicin (adriamycin)
· Cyclophosphamide
· 5-fluorouracil
· Cocaine abuse
· Daunorubicin
· Emetine
· Li
· Phenothiazines
· Sulphonamide
Your Answer. b
Correct Answer. a
Copyright © 2014 Delhi Academy of Medical Sciences, All Rights Reserved. 119/146
(148). Prevalence of diabetes mellitus in adults of India is about :
a. 0.9%
b. 2.4%
c. 3.8%
d. 6.5%
Solution. Ans-148: (c) 3.8%
Ref.: Park JE. - 313.
Sol :
Prevalence of diabetes mellitus in adult :
§ In world – 4% projected by 2025 – 5.4%
§ In south – east Asia region – 2.1 – 4.1%
§ India – 3.8% (2.4% in rural and 4.0 – 11.6% urban)
Your Answer. b
Correct Answer. c
(149). Which one of the following is the most appropriate therapy for an individual who has ingested an overdose of aspirin?
a. Acetazolamide
b. Sodium bicarbonate
c. Flumazenil
d. Allopurinol
Solution. Ans-149: (b) Sodium bicarbonate
Ref.: Harrison’s Principal of Internal medicine- 259
Sol :
Treatment of acute salicylate poisoning
§ Symptomatic and supportive.
§ Most important is external cooling and IV fluid with Na+, K+, HCO-3 and glucose according to need determined by repeated
monitoring.
§ Gastric lavage to remove unabsorbed drug, forced alkaline dieresis or haemodialysis to remove absorbed drug is indicated in
severe
cases.
§ Blood transfusion and vitamin K should be given if bleeding occurs.
Your Answer. b
Correct Answer. b
(150). Each of the following may cause acute pelvic pain except:
a. Diverticulitis
b. Pelvic thrombophlebitis
c. Ureteral lithiasis
d. Ovarian remnant syndrome
Solution. Ans-150: (d) Ovarian remnant syndrome
Ref.: Read the text below
Sol :
§ A wide variety of conditions can produce acute pelvic pain; that is, pain characterized by sudden onset, sharp rise, and short
course.
§ Conversely, ovarian remnant syndrome is a chronic pain generally.
Your Answer. d
Correct Answer. d
Copyright © 2014 Delhi Academy of Medical Sciences, All Rights Reserved. 120/146
(151). All of the following advantages of Minipill except:
a. No alteration of milk production
b. Tolerance in women who are unable to take estrogen
c. Independent of sexual intercourse
d. Irregular vaginal bleeding
Solution. Ans-151: (d) Irregular vaginal bleeding
Ref: Read the text below
Sol :
Minipill
Less than 1% of oral contraceptive prescriptions in the United States are for this progestin-only oral contraceptive; however, typical
use
failure rates approximate the typical use failure rates of combined oral contraceptive pills.
· Advantages
o No alteration of milk production and nearly 100% effectiveness in breastfeeding women
o Tolerance in women who are unable to take estrogen
o Independent of sexual intercourse
· Disadvantages
o Irregular vaginal bleeding
o No protection against STDs
o Need for daily administration
§ Progestin-only pills are taken continuously for 28 days without a pill-free interval. Because these pills have a dose of progestin that
is
very close to the threshold of contraceptive efficacy, they must be taken at approximately the same time each day.
§ Suppressed ovulation occurs in only a proportion, and contraceptive efficacy depends on the other progestin-related mechanisms
previously listed (see IV A).
Efficacy. Pregnancy rate is 5 in 100 woman-years of use. Consistent administration is necessary. A difference of a few hours may
contribute to reduced contraceptive protection
Your Answer. b
Correct Answer. d
(152). Arrest of descent of head means no descent for ____ hrs in multipara.
a. 1
b. 1.5
c. 2
d. 1.2
Solution. Ans-152: (a) 1
Ref: Read the text below
Sol:
DEFINITION Nullipara Multipara
Prolonged latent phase >20 h >14 h
Average second stage 50 min 20 min
Prolonged second stage without (with) epidural >2 h (>3 h) >1 h (>2 h)
Protracted dilation(ACTIVE PHASE) < 1.2 cm/h < 1.5 cm/h
Protracted descent < 1 cm/h < 2 cm/h
Arrest of dilation* >2 h >2 h
Arrest of descent* >2 h >1 h
Prolonged third stage >30 min >30 min
Your Answer. b
Correct Answer. a
Copyright © 2014 Delhi Academy of Medical Sciences, All Rights Reserved. 121/146
(153). A woman presents with acute pain, and after a careful history and physical examination, you decide the most likely diagnosis
is
diverticulitis. A leukocytosis is noted, further increasing your suspicion. Which test should be performed to confirm the diagnosis?
a. CT scan
b. Barium enema
c. Gastrografin enema
d. Pelvic MRI
Solution. Ans-153: (a) CT scan
Ref.: Read the text below
Sol :
§ Acute diverticulitis is a condition in which there is inflammation of a diverticulum or outpouching of the wall of the colon.
§ Diverticulitis typically affects postmenopausal women, but it can occur rarely in women in their thirties and forties.
§ A CT scan is a useful adjunct to the history and physical examination. Enemas, however, are contraindicated.
Your Answer. b
Correct Answer. a
(154). Each of the following hormonal changes occurs early in normal pubertal development in girls except :
a. Increased circulating estradiol
b. Sleep entrained increases in LH and FSH
c. Increased circulating dehydroepiandrosterone sulfate
d. Decreased circulating estrone
Solution. Ans-154: (d) Decreased circulating estrone
Ref.: Read the text below
Sol :
Major hormonal changes occurs early in normal pubertal development in girls :
ØIncreased circulating estradiol
ØSleep entrained increases in LH and FSH
ØIncreased circulating dehydroepiandrosterone sulfate
ØIncreased circulating estrone in early pregnancy
ØIncreased circulating dehydroepiandrosterone
Your Answer. d
Correct Answer. d
(155). Amount of iron required for fetus in pregnancy
a. 1000 mg
b. 0.1 gm
c. 0.3 gm
d. 3 gm
Solution. Ans-155: (c) 0.3 gm
Ref: Read the text below
Sol:
· Total iron reqd. in pregnancy is 1000mg: 300 in fetus and placenta, 400 in red cell mass, 200-300mg is obligatory loss through
normal route.
Your Answer. c
Correct Answer. c
Copyright © 2014 Delhi Academy of Medical Sciences, All Rights Reserved. 122/146
(156). Secondary oocyte is
a. 46 XY
b. 46 XX
c. 23 X
d. 23 Y
Solution. Ans-156: (c) 23 X
Ref: Read the text below
Sol:
· Secondary oocyte is formed after first meiosis and hence it is haploid 23x.
Your Answer. c
Correct Answer. c
(157). Endometrial hyperplasia is caused by
a. Granulosa cell tumour
b. Dysgerminoma
c. Serous Ca
d. Mucinous Ca
Solution. Ans-157: (a) Granulosa cell tumour
Ref: Read the text below
Sol:
Granulosa cell tumour
It is an estrogen secreting tumour of sex-cord, stromal cell origin. Hence it causes endometrial hyperplasia
Your Answer. a
Correct Answer. a
Copyright © 2014 Delhi Academy of Medical Sciences, All Rights Reserved. 123/146
(158). Cervical cancer with upper 2/3 vagina involved
a. I
b. IIa
c. II b
d. III a
Solution. Ans-158: (b) IIa
Ref: Read the text below
Sol:
Your Answer. c
Correct Answer. b
(159). Monochorionic monoamniotic twins occurs due to division during
a. Within 24 Hours
b. 24-72 hours
c. 72 Hours- 8 days
d. After 8 days
Solution. Ans-159: (d) After 8 days
Ref: Read the text below
Sol:
· Monoamniotic twins are identical twins that share the same amniotic sac within their mother’s uterus.

· Monoamniotic twins are always identical, and always monochorionic as well (sharing the sameplacenta), and are sometimes

termed Monoamniotic-Monochorionic("MoMo") twins.


· They also share the placenta, but have two separateumbilical cords. Monoamniotic twins develop when an embryo does not split

until
after formation of the amniotic sac, at about 9 days afterfertilization.
· Monoamniotic triplets or other monoamniotic multiplesare possible, but extremely rare
Your Answer. b
Correct Answer. d
Copyright © 2014 Delhi Academy of Medical Sciences, All Rights Reserved. 124/146
(160). All are indications for hysterectomy except :
a. Couvelaire uterus
b. Rupture uterus
c. PPH not controlled by ligation of internal iliac artery
d. Placenta accreta
Solution. Ans-160: (a) Couvelaire uterus
Ref.: Read the text below
Sol :
§ Couvelaire uterus is a result of widespread extra-vasation of blood into the uterine musculature and beneath the uterine serosa, so

called uteroplacental apoplexy.
These myometrial hemorrhages are not an indication for hysterectomy.
Your Answer. d
Correct Answer. a
(161). Breech presentation is mostly mistaken for :
a. Face presentation
b. Brow
c. Shoulder
d. Vertex
Solution. Ans-161: (a) Face presentation
Ref.: Read the text below
Sol :
§ “The main diagnostic confusion of breech is with face presentation, which produces the characteristic bony landmarks of the malar
eminences, mouth, and mentum that produce a bony triangle whereas the ischial tuberosities and the sacrum are in the line.
§ The mouth in contrast to the anus, has a firm unyielding margin of the bone.”
Your Answer. a
Correct Answer. a
(162). Birth trauma is a risk factor for :
a. Prolapse uterus
b. Endometriosis
c. PID
d. Abortions
Solution. Ans-162: (a) Prolapse uterus
Ref.: Read the text below
Sol :
§ “In 95% of cases of genital prolapse, the patient is multiparous and the implication is that childbearing is an important causal
factor.”
Your Answer. b
Correct Answer. a
Copyright © 2014 Delhi Academy of Medical Sciences, All Rights Reserved. 125/146
(163). Ovarian epithelial carcinoma constitutes what percent of ovarian malignancies?
a. 40%
b. 90%
c. 60%
d. 70%
Solution. Ans-163: (b) 90%
Ref.: Read the text below
Sol :
§ “Approximately 90% of ovarian cancers are derived from the tissues that come from coelomic epithelium or mesotheium” which
comprise the epithelial ovarian cancers.
§ Non-epithelial malignancies of the ovary account for about 10% of all ovarian cancers, which include malignancies of : germ cell
tumors – account for -3% of all ovarian cancers.
§ Sexcord – stromal cell origin – account for 5-8% of ovarian malignancies. Metastatic carcinomas to the ovary – account for 5-6%
of
ovarian malignancies.
Your Answer. b
Correct Answer. b
(164). Colour of lochia alba is due to :
a. Decidual cells
b. WBC
c. RBC
d. Pus cells
Solution. Ans-164: (a) Decidual cells
Ref.: Read the text below
Sol :
In the field of the obstetrics, lochia is the vaginal discharge after giving birth (puerperium) containing blood, mucus, anduterian
tissue.
Lochia discharge typically continues for 4 to 6 weeks after childbirth.
It is sterile for the first 2–3 days, but not so by the third or fourth day, as the uterus begins to be colonized by vaginal commensals
such
as non-hemolyticstreptococci and E. coli.
It progresses through three stages:
1. Lochia rubra (or cruenta) is the first discharge, red in color because of the large amount of blood it contains. It typically lasts no
longer than 3 to 5 days after birth.
2. Lochia serosa is the term for lochia that has thinned and turned brownish or pink in color. It contains
serousexudate, erythrocytes, leukocytes, and cervical mucus. This stage continues until around the tenth day after delivery. Lochia
serosa which persists to some weeks after birth can indicate late postpartum hemorrhaging, and should be reported to a physician.
3. Lochia alba (or purulenta) is the name for lochia once it has turned whitish or yellowish-white. It typically lasts from the second
through the third to sixth weeks after delivery. It contains fewer red blood cells and is mainly made up ofleukocytes, epithelial
cells, cholesterol, fat, and mucus. Continuation beyond a few weeks can indicate a genital lesion, which should be reported to a
physician.
Your Answer. b
Correct Answer. a
Copyright © 2014 Delhi Academy of Medical Sciences, All Rights Reserved. 126/146
(165). Candidiasis can be caused by all except :
a. OC pills
b. Pregnancy
c. Diabetes
d. Hypertension
Solution. Ans-165: (d) Hypertension
Ref.: Read the text below
Sol :
§ Candida infection is more likely in patients of diabetes (more glycogen in calls and renal glycosuria).
§ Pregnancy (increased vaginal acidity, increased glycogen and renal glycosuria), having broad spectrum antibiotics (destroying
acid
forming lactobacilli) or having oral pills.
§ The important host factor is depressed cell mediated immunity. The recurrence is from the bowel.
§ There may be temporary relief during or soon following menstruation because of diminished acidity of vaginal flora.
Your Answer. d
Correct Answer. d
(166). Chadwick sign means
a. Bluish discoloration of vagina
b. Lateral forniceal pulsations
c. Softening of Cervix
d. Assymetrical Uterus
Solution. Ans-166: (a) Bluish discoloration of vagina
Ref: Read the text below
Sol:
CHADWICK’S SIGN IS BLUISH DISCOLOURATION OF VAGINA.
Jacquemier’s or Chadwick’s sign(8th wk) :-
Dusky hue of vestibule & ant vaginal wall
Cause:- local vascular congestion
d/d pelvic tumour , uterine fibroid
Your Answer. c
Correct Answer. a
(167). Pemphigus is a disease in which patients make antibodies to one of their own skin desmogleins involved in the formation of
the junctional
complexes between cells. Which of the following junctional complexes would be most affected in this disease?
a. Macula adherens
b. Hemidesmosomes
c. Zonula occludens
d. Gap junctions
Solution. Ans 167: - (a) Macula adherens.
Ref- Read the text below.
Sol.
· In pemphigus, autoantibodies to desmoglein (a member of the cadherin protein family) result in disruption of the macula
adherentes
(plural) or desmosomes.
· The desmogleins are the transmembrane linker proteins of the desmosome. Specific desmogleins are the target of the
autoantibodies
in different forms of the disease.
Cadherins are CA2+-dependent transmembrane-linker molecules essential for cell-to-cell contact, so their disruption in pemphigus
leads
to severe blistering of the skin because of disrupted cell-cell interactions early in the differentiation of the keratinocyte (epidermal
cell)
and excessive fluid loss
Your Answer. b
Correct Answer. a
Copyright © 2014 Delhi Academy of Medical Sciences, All Rights Reserved. 127/146
(168). A 25- year- old man with excessive dandruff and a pruritic erythematous rash affecting his eyebrows, ears, scalp and lateral
margins of
the nose.Diagnosis is
a. Seborrhoeic dermatitis
b. Sarcoidosis.
c. Lupus pernio
d. Lupus vulgaris
Solution. Ans 168: - (a) Seborrhoeic dermatitis
Ref- Read the text below.
Sol
· Seborrhoeic dermatitis may be associated with overgrowth of Pityrosporum ovale, commensal yeast, and can be severe in patients
with HIV disease.
· The eruption often affects sebaceous areas of the skin. Scalp and facial involvement is a common presentation in young men.
Your Answer. a
Correct Answer. a
(169). A5- year old girl develops colicky abdominal pain that is intermittent. The next day she has painful swelling of both her ankles
and feet.
Examination also reveals a palpable purpuric rash on the legs and larger confluent ecchymosis on the back of the thighs and the
buttocks. Diagnosis is
a. Lichen planus
b. CML
c. Idiopathic thrombocytopaenic purpura.
d. Henoch schonlein purpura
Solution. Ans 169: - (d) Henoch schonlein purpura
Ref- Read the text below.
Sol
· This is one of the commonest presentations of Henoch schonlein purpura that causes a generalized vasculitis.
· The rash is palpable and typically appears in the lower extremity and the buttocks but could involve other areas of the body.
Your Answer. c
Correct Answer. d
(170). Treatment of granuloma inguinale is:
a. Tetracycline
b. Sulphonamide
c. Penicillin
d. None.
Solution. Ans-170: (a) Tetracycline
Ref: Read the text below
Sol:
GRANULOMA INGUINALE
· Three weeks of treatment with erythromycin, streptomycin, or tetracycline, or 12 weeks of treatment with ampicillin are standard
forms of therapy.
· Normally, the infection will begin to subside within a week of treatment, but the full treatment period must be followed in order to
minimize the possibility of relapse.
Your Answer. a
Correct Answer. a
Copyright © 2014 Delhi Academy of Medical Sciences, All Rights Reserved. 128/146
(171). Burrow is characteristically seen in
a. Syphilis
b. Scabies
c. LGV
d. Strongyloidiasis
Solution. Ans-171: (b) Scabies
Ref:Read the text below
Sol:
· Scabies is a contagious skin infection that occurs among humans and animals. It is caused by a parasite—the mite Sarcoptes
scabiei—which burrows under the host's skin, causing him or her to suffer intense itching.
· Crusted scabies, formerly known as Norwegian scabies, is a more severe form of the infection; the mites, numbering thousands in
such cases, cause scaly rashes and thick crusts of skin to form on the host.
· The burrows that mites make are usually hard to see. They are thread-line ridges 5-15 mm long.
· The scabies mite usually burrows into the skin such as the wrists, the webs of the fingers, the abdomen, buttocks, and genital
areas.
Small blisters might show up at the end of burrows.
Your Answer. b
Correct Answer. b
(172). The pathognomonic “Exclamation Mark” hairs are seen in
a. Alopecia areata
b. Tinea capitis
c. Icthyosis vulgaris
d. Dermatomyositis
Solution. Ans-172: (a) Alopecia areata
Ref:Read the text below
Sol:
· The prime diagnostic feature of acute alopecia areata is the presence of exclamation mark hairs.
· These characteristic hairs fracture at their distal end and taper proximally towards the scalp, giving them the appearance of an
exclamation mark.
· Hair morphology was studied in 8 patients with untreated acute alopecia areata and 3 normal adults without hair loss.
· Light microscopy, transmission and scanning electron microscopy revealed distinct structural differences in the distal end of hairs
compared with the remainder of their length and with normal hair shafts.
·
Your Answer. b
Correct Answer. a
(173). Virchow cells are found in:
a. Syphilis
b. Lichen planus
c. Leprosy
d. Cutaneous Leishmaniasis
Solution. Ans-173: (c) Leprosy
Ref: Read the text below
Sol:
· When a biopsy of the affected Lepromatous Leprosy (LL) skin is examined pathologically, it shows large collections of
macrophages
with foamy cytoplasm, called “Foam Cells” (they also are called “Lepra Cells” or “Virchow Cells).
· Along with these, there are “Plasma Cells” and a few lymphocytes. Such collections of cells, initially are seen around the hair
follicles, sweat and sebaceous glands, blood-vessels and nerves.
· As the disease proliferates, they cover the whole sub-epithelial tissue, forming a band-like infiltrate.
· Acid-fast Zeihl Neelsen stain reveals M.leprae inside the macrophages.
Your Answer. b
Correct Answer. c
Copyright © 2014 Delhi Academy of Medical Sciences, All Rights Reserved. 129/146
(174). A 25-year old male is undergoing an incision drainage of an abscess under general anesthesia with spontaneous respiration.
The best
anaesthetic circuit for him would be –
a. Mapleson A
b. Mapleson B
c. Mapleson C
d. Mapleson D
Solution. Ans-174: (a) Mapleson A
Ref: Miller’s Anaesthesia; 7/e, Chap 25
Sol:
· In 1954 Mapleson described and analyzed five different semiclosed anesthetic systems, and they are now classically referred to as
the Mapleson systems and are designated with letters A through E.
· Subsequently, in 1975 Willis and coauthors described the F system that was added to the original five.
· The Mapleson systems consist of several common components, including –
o A facemask,
o A spring-loaded pop-off valve,
o Reservoir tubing,
o Fresh gas inflow tubing and
o A reservoir bag.
· Within the Mapleson systems, three distinct functional groups can be seen: A, BC, and DEF groups.
· The Mapleson A, also known as the Magill circuit, has a spring-loaded pop-off valve located near the facemask, and fresh gas flow
enters the opposite end of the circuit near the reservoir bag.
· In the B and C systems, the spring-loaded pop-off valve is located near the facemask, but the fresh gas inlet tubing is located near
the patient.
· The reservoir tubing and breathing bag serve as a blind limb where fresh gas, dead space gas, and alveolar gas can collect.
· Finally, in the Mapleson D, E, and F or “T-piece” group, fresh gas enters near the patient, and excess gas is popped off at the
opposite end of the circuit.
· Even though the components and their arrangement are simple, functional analysis of the Mapleson systems can be complex.
The following summarizes the relative efficiency of different Mapleson systems with respect to prevention of rebreathing –
During spontaneous ventilation A > DFE > CB
During controlled ventilation DFE > BC > A
Your Answer. b
Correct Answer. a
Copyright © 2014 Delhi Academy of Medical Sciences, All Rights Reserved. 130/146
(175). All of the following are strategies to improve oxygenation, when used with appropriate ventilators, except –
a. Prone ventilation
b. Low tidal volume high PEEP method
c. High frequency oxygenation ventilation
d. Extracorporeal membrane oxygenation
Solution. Ans-175: (b) Low tidal volume high PEEP method
Ref: Villar J, Kacmarek R, Pérez-Méndez L and Aguirre-Jame A. A High Positive End-Expiratory Pressure, Low Tidal Volume
Ventilatory
Strategy Improves Outcome in Persistent Acute Respiratory Distress Syndrome: A Randomized, Controlled Trial. Crit Care Med.
2006;34(5):1311-18.
Sol:
· It has been shown in a two-center study that high positive end-expiratory pressure (PEEP) and low tidal volume (LTV) improved
outcome in ARDS.
· However, that study involved patients with underlying diseases unique to the study area, was conducted at only two centers, and
enrolled a small number of patients.
· Acute respiratory distress syndrome (ARDS), a severe form of acute lung injury (ALI), is one of the most challenging problems in
the
intensive care unit (ICU).
· The prognosis of patients with ARDS is poor, with death often attributed to the underlying disease and the presence of sepsis.
· Nevertheless, it was found that the use of a small, physiologic Vt and a PEEP level set on day 1 at Pflex + 2 cm H2O in comparison
with
a ventilatory protocol with a higher Vt and low PEEP level resulted in a 21.3% absolute improvement in ICU mortality.
· The improved mortality observed in the Pflex/LTV (low tidal volume) group could be a result of two mechanisms –
· A higher PEEP level and a lower Vt and plateau pressure, the high PEEP preventing recruitment-derecruitment injury and
· The lower Vt and plateau pressure avoiding overdistension injury.
· Although the Vt difference was small (about 2.8 mL/kg PBW), plateau pressures in the control group averaged >32 cm H2O,
whereas
the plateau pressures in the Pflex/LTV group averaged ≤30 cm H2O throughout the ventilation period.
· This low plateau pressure resulted in less end-inspiratory stress and strain on lung tissue and, as a result, less imposed lung
injury.
Your Answer. c
Correct Answer. b
Copyright © 2014 Delhi Academy of Medical Sciences, All Rights Reserved. 131/146
(176). Which of the following is most commonly used for day care surgery?
Copyright © 2014 Delhi Academy of Medical Sciences, All Rights Reserved. 132/146
a. Morphine
b. Ketamine
Copyright © 2014 Delhi Academy of Medical Sciences, All Rights Reserved. 133/146
c. Propofol
d. Etomidate
Solution. Ans-176: (c) Propofol
Ref: Chandrapriya H, Gurav S. Anesthesia for Day Care Surgery. (http://ejournal.mitmimer.com/issue3/Anaesthesia.pdf)
Sol:
DAY CARE SURGERY –
· Day care surgery is known by several names as outpatient surgery / ambulatory surgery / fast track surgery / same day surgery.
· In day care surgery, patient is operated and sent back home on the same day with an escort.
· The availability of rapid, short acting anaesthetic, analgesic and muscle relaxant drugs has clearly facilitated the recovery process
and allowed more extensive procedures to be performed on an ambulatory basis, irrespective of pre-existing medical conditions.
BENEFITS –
· Reduced dependence on availability of hospital beds.
· Greater flexibility in scheduling operations
· Short surgical waiting lists.
· Lower overall procedural costs.
· Greater turnover of patients.
· Less incidence of hospital acquired infections and respiratory complications.
COMMON PROCEDURES SUITABLE FOR DAY CARE SURGERY –
General surgery
· Herniorrhaphy
· Hemorrhoidectomy
· Lap procedures
Gynecology
· Dilation and Curettage (D&C)
· Laparoscopy
· Hysteroscopy
· Vaginal hysterectomy
· Tubal ligation
Orthopedics
· Implant removal
· Arthroscopy
· Closed reduction procedures
Otorhinolaryngology
· Myringotomy
· Mastoidectomy
· Tympanoplasty
· Adenoidectomy
· Tonsillectomy
· Rhinoplasty
· Polypectomy
Urology
· Circumcision
· Cystoscopy
· Lithotripsy
· Prostate surgery
· Orchidectomy
RECOMMENDED DRUGS FOR PRE-MEDICATION –
Anxiolytics · Alprazolam
· Midazolam
Analgesics · NSAIDs
Antiemetics · Ondansetron
· Dexamethasone
Aspiration prophylaxis
· Ranitidine
· Pantoprazole
· Sodium citrate
CHOICE OF DRUGS FOR GENERAL ANESTHESIA –
· Since rapid recovery is the aim, short acting drugs are preferred for general anaesthesia.
· Preferred choice of drugs is as follows:
Inhalational induction agents · Sevoflurane
Intravenous induction agents · Propofol
· S-ketamine
Analgesics
· Fentanyl
· Alfentanyl
· Sufentanyl
· Remifentanil
Maintenance agents · Nitrous oxide
Muscle relaxants
· Succinylcholine
· Atracurium
· Cisatracurium
· Mivacurium
CONTRAINDICATIONS –
· Serious life-threatening diseases not optimally controlled
· Morbid obesity complicated by CVS & RS symptoms
· Chronic use of centrally active drugs
· Extremely premature infants (<60 weeks post conceptional age requiring general anaesthesia)
· Lack of responsible adult at home to take care of the patient.
· Surgical procedure associated with major fluid shifts, significant blood loss, significant postoperative pain, nausea and vomiting.
·
Your Answer. c
Correct Answer. c
Copyright © 2014 Delhi Academy of Medical Sciences, All Rights Reserved. 134/146
(177). A child underwent eye surgery under general anesthesia with propofol and succinylcholine. After 8 hours, the child started
walking and
developed muscle pain. The most likely cause of his muscle pain is –
a. Propofol
b. Succinylcholine
c. Early mobilization
d. Effects of the eye surgery
Solution. Ans-177: (b) Succinylcholine
Ref: Goodman & Gilman’s ‘Pharmacological Basis of Therapeutics’; 12/e, pg 268
Sol:
The development of muscle pain in this case is due to hyperkalemia induced by succinylcholine.
· The depolarizing agents like succinylcholine can release K+ rapidly from intracellular sites.
· This may be a factor in production of prolonged apnea in patients who receive these drugs while in electrolyte imbalance.
· Succinylcholine-induced hyperkalemia is a life-threatening complication of the drug, especially in patients with congestive heart
failure who are receiving digoxin or diuretics.
· For the same reason, caution should be used or depolarizing blocking agents should be avoided in patients with –
o Extensive soft tissue trauma
o Burns
o Non-traumatic rhabdomyolysis
o Ocular lacerations
o Spinal cord injuries with paraplegia or quadriplegia and
o Muscular dystrophies.
· A higher dose of competitive blocking agent is often indicated in these patients.
· Succinylcholine is no longer indicated for children ≤ 8 years of age unless emergency intubation or securing an airway is
necessary.
· Hyperkalemia, rhabdomyolysis and cardiac arrest have been reported.
A subclinical dystrophy is frequently associated with these responses
Your Answer. b
Correct Answer. b
Copyright © 2014 Delhi Academy of Medical Sciences, All Rights Reserved. 135/146
(178). The terms ‘Bethune’ and ‘Brethren’ are associated with which of the following?
a. Plethysmography
b. End tidal CO2 measurement
c. Colour Doppler
d. Transesophageal echocardiography
Solution. Ans-178: (d) Transesophageal echocardiography
Ref: Ref: http://en.wikipedia.org/wiki/Transesophageal_echocardiogram
Sol:
The terms ‘Bethune’ and ‘Brethren’ are associated with transesophageal echocardiography.
TRANSESOPHAGEAL ECHOCARDIOGRAPHY –
· A transesophageal echocardiogram, or TEE, is an alternative way to perform an echocardiogram.
· A specialized probe containing an ultrasound transducer at its tip is passed into the patient's esophagus.
· This allows image and Doppler evaluation which can be recorded.
· Before inserting the probe, mild to moderate sedation is induced in the patient to ease the discomfort and to decrease the gag
reflex,
thus making the ultrasound probe easier to pass into the esophagus.
· Sedation can be induced with medications such as midazolam, fentanyl or propofol.
· Usually a local anesthetic spray is used for the back of the throat, such a xylocaine and / or a jelly / lubricant anesthetic for the
esophagus.
· Children are anesthetized.
· Unlike the TTE, the TEE is considered an invasive procedure and is thus performed by physicians in the U.S., not sonographers.
Advantages –
· The advantage of TEE over transthoracic echocardiography (TTE) is usually clearer images, especially of structures that are
difficult
to view transthoracically (through the chest wall).
· The explanation for this is that the heart rests directly upon the esophagus leaving only millimeters that the ultrasound beam has to
travel.
· This reduces the attenuation (weakening) of the ultrasound signal, generating a stronger return signal, ultimately enhancing image
and Doppler quality.
· Comparatively, transthoracic ultrasound must first traverse skin, fat, ribs and lungs before reflecting off the heart and back to the
probe before an image can be created.
· All these structures, along with the increased distance the beam must travel, weaken the ultrasound signal thus degrading the
image
and Doppler quality.
· In adults, several structures can be evaluated and imaged better with the TEE, including the aorta, pulmonary artery, valves of the
heart, both atria, atrial septum, left atrial appendage, and coronary arteries.
· TEE has a very high sensitivity for locating a blood clot inside the left atrium.
Disadvantages –
· TEE requires a fasting patient.
· The patient must follow the ASA NPO guidelines (i.e. usually not eat anything for eight hours and not drink anything for two hours
prior to the procedure.)
· Requires a team of medical personnel
· Takes longer to perform than TTE
· May be uncomfortable for the patient
· May require sedation or general anesthesia
· There are some risks associated with the procedure (esophageal perforation in ~ 1 in 10,000, and adverse reactions to the
medication.)
Your Answer. Not Attempted
Correct Answer. d
Copyright © 2014 Delhi Academy of Medical Sciences, All Rights Reserved. 136/146
(179). A 65-year-old man complains of increasing sadness and inability to find pleasure in anything . He has recently been forced to
retire from
his job, and he has several chronic medical problems. Which of the following symptoms most increases his risk for suicide?
a. Tearfulness
b. Feelings of hopelessness
c. Sleep disturbance
d. Lassitude
Solution. Ans 179: (b) Feelings of hopelessness
Reference – Read the text below.
Sol
· The presence of hopelessness a as component of depression greatly increases the chance of suicide.
· Another high-risk findings would be a suicide plan.
· Tearfulness, sleep disturbance,lassitude,and anorexia are associated with the syndrome of depression, but are not as strongly
associated with suicide behavior.
Your Answer. b
Correct Answer. b
(180). Which of the following receives the greatest weighting on the Sad Persons Scale
a. Organized attempt
b. Sex Male
c. Excessive drug use
d. Psychiatric care
Solution. Ans 180: (a) Organized attempt. .
References: Read the text below.
Sol:
SAD PERSONS SCALE
This scale is used to identify risk factors for completed suicide
Sex: Male 1
Depression: hopelessness, despair, esp if associated psychological shift symptoms 2
Psychiatric care: previous DSH, psychiatric care or personality disorder 1
Excessive drug use 1
Rational thinking loss: severe depression with psychotic features, organic brain
syndrome, delusions 2
Single separated, divorced, widowed 1
Organized attempt: planned, premeditated, lethal and available method 2
No life supports: social isolation, homeless, unemployed 1
States future intent: continued suicidal ideation 2
Your Answer. b
Correct Answer. a
Copyright © 2014 Delhi Academy of Medical Sciences, All Rights Reserved. 137/146
(181). In which of the following diagnoses is splitting mechanism most commonly encountered?
a. Major depressive episodes
b. Obsessive-compulsive disorder
c. Schizoid personality disorder
d. Borderline personality disorder
Solution. Ans 181: (d) Borderline personality disorder
Ref– Read the text below
Sol:
· Splitting is a defense mostly encountered in more severely disturbed patients, such as patients with personality disorders and, in
particular, borderline personality disorder.
· According to the object relation theory, patients with severe personality disorders, due to faulty parenting during infancy, have not
been able to form stable and realistic intrapsychic representations of themselves and of the important people in their lives.
· Instead of a seeing themselves and others as consistent entities containing both good and bad traits, they perceive the negative
and
positive aspects as separate, irreconcilable parts.
· Splitting, by allowing the manifestation of only one side of the ambivalence at a time, preserves the “good objects,”which,
otherwise,
in the patient’s experience, would consistently risk being contaminated and neutralized by the “bad objects.”
Your Answer. c
Correct Answer. d
(182). A phobic patient who has an overwhelming fear of snakes is shown the picture of a cobra while she practices deep muscle
relaxation.
This technique is called
a. Desensitization
b. Contingency management
c. Flooding
d. Operant conditioning
Solution. Ans 182: (a) Desensitization
Ref– Read the text below
Sol:
· All behavioral treatments for phobias have in common exposure to the feared stimulus.
· Desensitization is based on the concept that when the feared stimulus is presented paired with a behavior that induces a state
incompatible with anxiety (e.g., deep muscle relaxation), the phobic stimulus loses its power to create anxiety (counterconditioning).
· For desensitization to work, the anxiety elicited by the exposure has to be low. Treatment starts with exposure to stimuli that
produce minimal anxiety and proceeds to stimuli with a higher anxiety potential.
· Operant conditioning refers to the concept that behavior can be modified by changing the antecedents or the consequences of the
behavior (contingency management).
· Flooding is another exposurebased treatment for phobia, based on extinction rather than counterconditioning.
Your Answer. a
Correct Answer. a
Copyright © 2014 Delhi Academy of Medical Sciences, All Rights Reserved. 138/146
(183). A 35-year-old woman spends most of her day rocking, muttering softly to herself,or looking at her reflection in a small mirror.
She needs
help with dressing and showering and she often giggles and laughs for no apparent reason.Patient is suffering from:
a. Schizophreniform disorder
b. Catatonia
c. Shared psychotic disorder
d. Schizophrenia, disorganized
Solution. Ans 183: (d) Schizophrenia, disorganized
Ref– Read the text below
Sol:
· Depending on the predominance of particular symptoms, four subtypes of schizophrenia are recognized: paranoid, disorganized,
catatonic, and residual.
· The essential characteristics of the disorganized type are disorganized speech and behavior, flat or inappropriate affect, great
functional impairment,and inability to perform basic activities such as showering or preparing meals.
· Grimacing, silly and odd behavior, and mannerisms are common. Hallucinations and delusions, if present, are fragmented and not
organized in a coherent theme.
· This subtype is associated with poor premorbid functions, early insidious onset, and a progressive course without remissions.
Your Answer. b
Correct Answer. d
(184). Which of the following statements regarding delusions is true?
a. Delusions are almost exclusively found in schizophrenia
b. Delusions of grandiosity are rarely encountered except in mania
c. Delusions involve a disturbance of thought content
d. Delusions involve a disturbance of perception
Solution. Ans 184: (c) Delusions involve a disturbance of thought content
Ref– Read the text below
Sol:
· A delusion is defined as a firmly held false belief, not shared by other people in the same social and cultural group, and firmly held
against evidence that disproves the belief.
· Delusions are classified as disturbances of thought content and they are found in a wide variety of psychotic conditions other than
schizophrenia,including organic disorders, states of intoxication, and mood disorders.
· Although certain types of delusions are more common in certain disorders (delusions of being controlled by external agents in
schizophrenia and grandiose delusions in mania), delusions are not diagnostically specific.
· Hallucinations are disorders of perception.
Your Answer. d
Correct Answer. c
Copyright © 2014 Delhi Academy of Medical Sciences, All Rights Reserved. 139/146
(185). Which of the following is false regarding suicide risk assessment?
a. Males are at a higher risk than females.
b. Single individuals are at higher risk than married ones.
c. Individuals with chronic medical problems are at higher risk than those without medical problems
d. Individuals with a history of substance abuse are at a lower risk than those without.
Solution. Ans 185: (d) Individuals with a history of substance abuse are at a higher risk than those without.
References: Read the text below.
Sol:
S sex M>F
A age <19 or >45 years
D depression, major depression, hopelessness
P psychiatric history, personality disorder
E excessive alcohol and substance abuse
R rational thinking loss, psychosis, organic brain syndrome, delusions
S single, separated, socially isolated
O organized attempt
N no supports, unemployed homeless
S states future intent
Your Answer. d
Correct Answer. d
(186). The Thematic Apperception Test (TAT) would be most useful for which of the following purposes?
a. In prediction of suitability for psychotherapy
b. In assessment of suicidal risk
c. In assessment of intellectual level
d. In assessment of motivational variables
Solution. Ans 186: (d) In assessment of motivational variables
Reference – Read the text below.
Sol
· The Thematic Apperception Test (TAT), which consists of a series of 30 ambiguous pictures about which subjects are asked to
construct a story, is most useful in assessing motivational variables.
· It provides a case-study exploration of a person's personality. Although it also has been employed to assess variables such as
intellectual level, other tests are more suitable for such purposes (e.g., IQ tests).
· The TAT is not particularly helpful in differential diagnosis.
Your Answer. a
Correct Answer. d
(187). A child presents with a history of injury to left forefinger tip with a glass piece. On examination, the finger is swollen and
tender. Which
of the following would be the best investigation to determine the glass foreign body in the patient’s finger?
a. Plain radiograph
b. Ultrasonography
c. CT scan
d. MRI
Solution. Ans-187: (a) Plain radiograph
Ref: Read the text below
Sol:
· Glass foreign bodies are radio-opaque, and therefore, should be evaluated by X-ray.
· If we are suspecting a radiolucent foreign body, then USG would have been advised.
Your Answer. b
Correct Answer. a
Copyright © 2014 Delhi Academy of Medical Sciences, All Rights Reserved. 140/146
(188). Radiation therapy to hypoxic tissues may be potentiated by the treatment with :-
a. Mycostatin
b. Metronidazole
c. Methotrexate
d. Melphalan
Solution. Ans-188: (b) Metronidazole
Ref: Read the text below
Sol :
The simple Principle of Radiotherapy :-
Radiation therapy is a form of physical therapy that damages any tissue in its path.
Mechanism of injury of radiation in cells-
– Radiation usually damages DNA
– Generates free radicals from cell water. Free radicals in turn damage cell membrane proteins
& organelles.
– Radiation damage is dependent on oxygen. So Hypoxic cells are resistant to radiotherapy.
Your Answer. d
Correct Answer. b
Copyright © 2014 Delhi Academy of Medical Sciences, All Rights Reserved. 141/146
(189). A middle-aged female presents with severe bone pains at multiple sites. Plain radiographs reveal multiple lytic lesions in
pelvis, ribs and
femur, fracture of the clavicles and subperiosteal reabsorption of metacarpals over the radial aspect. The most likely diagnosis is –
a. Hyperthyroidism
b. Hyperparathyroidism
c. Renal osteodystrophy
d. Osteomalacia
Solution. Ans-189: (b) Hyperparathyroidism
Ref: Sutton’s ‘Textbook of Radiology and Imaging’; 7/e, pg 1364
Sol:
RADIOLOGICAL FEATURES OF HYPERPARATHYROIDISM –
· Subperiosteal erosion of bone, particularly along the radial aspect of the middle phalanx of the middle and index finger, is virtually
pathognomonic of hyperparathyroidism.
· Other sites include the medial aspect of the proximal tibia, femur and humerus, and the ribs.
· Loss of the lamina dura around the teeth occurs, although this is not specific for hyperparathyroidism.
· Subchondral bone resorption is another common occurrence, being found at the distal and sometimes proximal end of the clavicle,
symphysis pubis and sacroiliac joints.
· This may also occur at the vertebral end-plates, which may permit disc herniation.
· Intracortical bone resorption is another feature of hyperparathyroidism, resulting from osteoclastic activity within the Haversian
canals.
· This gives rise to small (2-5 mm) oval or cigar-shaped lucencics within the cortex.
· Skull X-rays show characteristic salt & pepper or pepper pot appearance.
Your Answer. d
Correct Answer. b
(190). Intracranial neoplasm to metastasize to bone other than skull is :-
a. Medulloblastoma
b. Astrocytoma
c. Ependymoma
d. Pituitary adenoma
Solution. Ans-190: (a) Medulloblastoma
Ref: Diagnostic Radiology, Grainger 4th - 1875
Sol :
· The commonest intracranial neoplasm to metastasize to bones other than skull is medulloblastoma of the cerebellum.
Your Answer. a
Correct Answer. a
Copyright © 2014 Delhi Academy of Medical Sciences, All Rights Reserved. 142/146
(191). Imaging modality of choice in the setting of acute scrotal trauma :-
a. USG
b. Contrast enhance CT
c. MRI
d. Radionuclide scanning
Solution. Ans-191: (a) USG
Ref: Diagnostic radiology Grainger 4th - 1668.
Sol :
· Ultrasound is imaging modality of choice in case of acute scrotal trauma, and is performed with 5 or 7.5 MHz transducer.
· A scrotal haematoma appears as an echogenic collection b/w the tunica dartos and tunica vaginalis or in the scrotal septum.
Your Answer. a
Correct Answer. a
(192). The investigation of choice for acute appendicitis in children is –
a. Barium study
b. Ultrasonography
c. CT scan
d. MRI
Solution. Ans-192: (b) Ultrasonography
Ref: http://cursoenarm.net/UPTODATE/contents/mobipreview.htm?24/63/25599/abstract/9
Sol:
ACUTE APPENDICITIS IN CHILDREN –
· Appendicitis is the most common condition in children requiring emergency abdominal surgery.
· The key to a successful outcome has always been early diagnosis followed by appendectomy before gangrene or perforation
develops.
· Nevertheless, confirming the diagnosis of appendicitis and avoiding the risks of unnecessary surgery continue to be a challenge,
particularly in young children.
· Imaging plays an important role in the evaluation of patients who do not present with the classic signs and symptoms of
appendicitis
but in whom the diagnosis is being considered.
· Ultrasonography (US) and computed tomography (CT), separately or in combination, are the modalities used most frequently.
· In addition, there are data that suggest that using an imaging protocol improves diagnostic accuracy.
INDICATIONS –
· For children who have atypical or equivocal findings for appendicitis on physical examination and laboratory testing, imaging may
be
helpful to establish or exclude the diagnosis.
· A systematic institutional approach to the choice of imaging may improve outcomes.
· In contrast, imaging is not warranted in most children who are unlikely to have appendicitis based upon the clinical examination
and
laboratory studies.
· Similarly, children with a typical clinical presentation for acute appendicitis are likely to have appendicitis.
· In this circumstance, it is recommended that clinicians consult a surgeon with experience caring for children rather than obtaining
imaging studies.
1. ULTRASONOGRAPHY -
· Ultrasonography (US) is available in most institutions, is relatively inexpensive, and is safe.
· It has the added advantage of identifying ovarian pathology, such as torsion or an ovarian cyst.
· However, accuracy depends upon the skill and experience of the sonographer.
· US improves diagnostic accuracy in selected children with suspected appendicitis.
· Sensitivities have been more variable, ranging from 74 to 100 percent.
· Specificities from 88 to 99 percent have been reported.
· However, US may also result in a significant number of false positive and false negative results.
· The diagnosis of appendicitis cannot be reliably excluded unless a normal appendix is seen.
· Reported visualization rates vary from 22 percent to 98 percent.
· Factors that affect this variability primarily include the experience and technique of the sonographer.
· Techniques have been described that may improve visualization of the appendix and more accurately diagnose appendicitis.
o Posterior compression – The addition of posterior manual compression to graded compression can help to identify the appendix.
o Positional scanning — Scanning in the flank and pelvis, in addition to the right lower quadrant, may be useful.
Saline enema — A saline enema may distend the cecum and fill the appendix, making it easier to identify by ultrasound
Your Answer. b
Correct Answer. b
Copyright © 2014 Delhi Academy of Medical Sciences, All Rights Reserved. 143/146
(193). Inability to flex the distal phalanx of the fourth and fifth digits of the hand would indicate damage to which of the following
nerves?
a. Radial
b. Deep radial
c. Median
d. Ulnar
Solution. Ans-193: (d) Ulnar
Ref.: Read the text below
Sol :
§ In the forearm, the ulnar nerve innervates the flexor carpi ulnaris and the ulnar half of the flexor digitorum profundus. This latter
muscle flexes the distal phalanx of the fourth arid fifth digits.
Your Answer. c
Correct Answer. d
(194). A patient presents with a slight weakness in the lateral arch of the foot on one side and cannot dorsiflex the foot. These signs
probably
indicate damage to the :
a. Superficial peroneal nerve
b. Lateral plantar nerve
c. Deep peroneal nerve
d. Sural nerve
Solution. Ans-194: (c) Deep peroneal nerve
Ref.: Read the text below
Sol :
§ The deep peroneal nerve supplies the anterior muscles of the leg, including the tibiatis anterior, extensor hallucis longus, extensor
digitorum longus, and the peroneus tertius, which dorsiflex the foot.
Your Answer. c
Correct Answer. c
(195). A 15-year-old girl comes to you with a swelling on the shaft of the tibia. He complains of pain and fever. Histopathology
reveals small
rounded cells. Probable diagnosis is :
a. Ewing’s sarcoma
b. Osteoclastoma
c. Chondro sarcoma
d. Osteo sarcoma
Solution. Ans-195: (a) Ewing’s sarcoma
Ref.: Read the text below
Sol :
§ Ewing’s sarcoma is a highly malignant tumor occurring in patients between the age of 5 and 20 years with predilection for
occurrence
in females.
§ Patient presents with pain which gradually increases and is followed by the appearance of a swelling. The swelling is firm to soft in
consistency with indefinite margins. There is usually fever, anemia and leukocytosis.
§ The individual tumor cells comprising the lobules are small and uniform resembling lymphocytes and have ill – defined cytoplasmic
outlines, scanty cytoplasm and round nuclei with frequent mitoses.
Your Answer. b
Correct Answer. a
Copyright © 2014 Delhi Academy of Medical Sciences, All Rights Reserved. 144/146
(196). Causes of recurrent patellar dislocation :
a. Weaking of quadriceps expansion
b. Patella alta
c. Abnormal insertion of ligamentum patellae
d. All the above
Solution. Ans-196: (d) All the above.
Ref.: Read the text below
Sol :
Conditions causing recurrent dislocation of patella can be grouped as follows
§ Weakening and laxity of the quadriceps expansion and the capsule on the medial side of the knee.
§ Tightening and contracture of the structures on the lateral side of the knee.
§ Abnormal insertion of the ligamentum patella into a more lateral site on the tibia.
§ Defective development of the lateral femoral condyle.
§ Gross genu valgum
Patella alta and breva
Your Answer. d
Correct Answer. d
(197). Bunion refers to :
a. Exostosis of the base of the 2nd metatarsal bone
b. Bursa at the base of the metatarsal bone
c. Inflamed adventitial bursa at the 1st metatarsal bone
d. Exostosis at the head of the 1st metatarsal bone
Solution. Ans-197: (d) Exostosis of the base of the 1st metatarsal bone
Ref.: Read the text below
Sol :
§ A bunion is generally considered as an enlargement of the joint (a lump of bone) at the base and side of the big toe-(specifically,
the
first metatarsophalan-geal joint).
§ Bunions form when the toe moves out of place. As the big toe bends towards the others this lump becomes larger and the bunion
the
others this lump becomes larger and the bunion can become painful – arthritis and stiffness can eventually develop.
Your Answer. d
Correct Answer. d
(198). The rider’s bone is due to calcification of:
a. Adductor longus
b. Adductor magnus
c. Popliteus
d. Vastus lateralis
Solution. Ans-198: (a) Adductor longus
Ref.: Read the text below
Sol :
§ Rider’s bone is a localized ossification of the inner aspect of the lower end of the tendon of the adductor longus muscle of the
thigh
(adductor tubercle); sometimes seen in horseback riders; called also as cavalry bone, exercise bone.
Your Answer. b
Correct Answer. a
Copyright © 2014 Delhi Academy of Medical Sciences, All Rights Reserved. 145/146
(199). About fracture healing, all are true, except :
a. Lower limb long bones heal faster than other bones
b. Fracture in children heal faster than in adults
c. Oblique and spiral fractures heal faster than transverse fractures
d. Transverse fractures unite slowly.
Solution. Ans-199: (a) Lower limb long bones heal faster than other bones
Ref.: Read the text below
Sol :
§ Type of fracture : Transverse fractures unite slowly compared to oblique or spiral fractures.
§ Type of bone : Fracture at the cancellous ends of bone unites better than those in the mid shaft of long bones where cancellous
bone is
minimal.
§ General Causes :
§ Fractures in children unite very rapidly whereas delayed union is common in the aged.
Your Answer. a
Correct Answer. a
(200). Wilson fracture is seen in :
a. Metatarsals
b. Calcanium
c. Scaphoid
d. Volar plate fracture
Solution. Ans-200: (d) Volar plate fracture
Ref.: Read the text below
Sol :
§ Volar plate fractures are the result of hyperextension. This injury typically involve the proximal interphalangeal joint of the fingers.
§ At this location, the volar plate (a dense fibrous band) forms a portion of the capsule. Typically, there is a small fragment of bone
avulsed from the volar aspect of the base of the proximal phalanx.
§ If not repaired, this can lead to instability.
A Wilson fracture refers to a volar plate injury to the middle phalanx of a finger.
Your Answer. b
Correct Answer. d
Test Answer
1.(b) 2.(b) 3.(b) 4.(d) 5.(b) 6.(d) 7.(d) 8.(b) 9.(a) 10.(c)
11.(b) 12.(b) 13.(b) 14.(b) 15.(a) 16.(d) 17.(a) 18.(d) 19.(b) 20.(a)
21.(a) 22.(d) 23.(c) 24.(b) 25.(d) 26.(a) 27.(b) 28.(b) 29.(c) 30.(a)
31.(c) 32.(a) 33.(d) 34.(b) 35.(a) 36.(a) 37.(d) 38.(c) 39.(b) 40.(c)
41.(c) 42.(d) 43.(a) 44.(d) 45.(c) 46.(c) 47.(b) 48.(c) 49.(d) 50.(c)
51.(b) 52.(b) 53.(c) 54.(c) 55.(d) 56.(c) 57.(d) 58.(a) 59.(c) 60.(c)
61.(d) 62.(a) 63.(b) 64.(d) 65.(c) 66.(a) 67.(b) 68.(c) 69.(c) 70.(b)
71.(d) 72.(a) 73.(d) 74.(b) 75.(a) 76.(d) 77.(b) 78.(a) 79.(c) 80.(a)
81.(d) 82.(c) 83.(c) 84.(a) 85.(b) 86.(c) 87.(b) 88.(b) 89.(c) 90.(c)
91.(c) 92.(a) 93.(a) 94.(c) 95.(a) 96.(c) 97.(a) 98.(b) 99.(b) 100.(a)
101.(d) 102.(b) 103.(d) 104.(b) 105.(b) 106.(c) 107.(c) 108.(a) 109.(c) 110.(a)
111.(d) 112.(b) 113.(d) 114.(b) 115.(d) 116.(c) 117.(d) 118.(b) 119.(a) 120.(a)
121.(d) 122.(d) 123.(d) 124.(b) 125.(a) 126.(c) 127.(b) 128.(b) 129.(d) 130.(a)
131.(c) 132.(d) 133.(c) 134.(c) 135.(d) 136.(d) 137.(a) 138.(b) 139.(a) 140.(b)
141.(b) 142.(b) 143.(d) 144.(b) 145.(a) 146.(a) 147.(a) 148.(c) 149.(b) 150.(d)
151.(d) 152.(a) 153.(a) 154.(d) 155.(c) 156.(c) 157.(a) 158.(b) 159.(d) 160.(a)
Copyright © 2014 Delhi Academy of Medical Sciences, All Rights Reserved. 146/146
161.(a) 162.(a) 163.(b) 164.(a) 165.(d) 166.(a) 167.(a) 168.(a) 169.(d) 170.(a)
171.(b) 172.(a) 173.(c) 174.(a) 175.(b) 176.(c) 177.(b) 178.(d) 179.(b) 180.(a)
181.(d) 182.(a) 183.(d) 184.(c) 185.(d) 186.(d) 187.(a) 188.(b) 189.(b) 190.(a)
191.(a) 192.(b) 193.(d) 194.(c) 195.(a) 196.(d) 197.(d) 198.(a) 199.(a) 200.(d)

You might also like